U Washington Bioethic Questions

अब Quizwiz के साथ अपने होमवर्क और परीक्षाओं को एस करें!

Robert Buckman's Six Step Protocol for Breaking Bad News

1.Getting started. The physical setting ought to be private, with both physician and patient comfortably seated. You should ask the patient who else ought to be present, and let the patient decide--studies show that different patients have widely varying views on what they would want. It is helpful to start with a question like, "How are you feeling right now?" to indicate to the patient that this conversation will be a two-way affair. 2. Finding out how much the patient knows. By asking a question such as, "What have you already been told about your illness?" you can begin to understand what the patient has already been told ("I have lung cancer, and I need surgery"), or how much the patient understood about what's been said ("the doctor said something about a spot on my chest x-ray"), the patients level of technical sophistication ("I've got a T2N0 adenocarcinoma"), and the patient's emotional state ("I've been so worried I might have cancer that I haven't slept for a week"). 3. Finding out how much the patient wants to know. It is useful to ask patients what level of detail you should cover. For instance, you can say, "Some patients want me to cover every medical detail, but other patients want only the big picture--what would you prefer now?" This establishes that there is no right answer, and that different patients have different styles. Also this question establishes that a patient may ask for something different during the next conversation.. 4. Sharing the information. Decide on the agenda before you sit down with the patient, so that you have the relevant information at hand. The topics to consider in planning an agenda are: diagnosis, treatment, prognosis, and support or coping. However, an appropriate agenda will usually focus on one or two topics. For a patient on a medicine service whose biopsy just showed lung cancer, the agenda might be: a) disclose diagnosis of lung cancer; b) discuss the process of workup and formulation of treatment options ("We will have the cancer doctors see you this afternoon to see whether other tests would be helpful to outline your treatment options"). Give the information in small chunks, and be sure to stop between each chunk to ask the patient if he or she understands ("I'm going to stop for a minute to see if you have questions"). Long lectures are overwhelming and confusing. Remember to translate medical terms into English, and don't try to teach pathophysiology. 5. Responding to the patient's feelings. If you don't understand the patient's reaction, you will leave a lot of unfinished business, and you will miss an opportunity to be a caring physician. Learning to identify and acknowledge a patient's reaction is something that definitely improves with experience, if you're attentive, but you can also simply ask ("Could you tell me a bit about what you are feeling?"). 6. Planning and follow-through. At this point you need to synthesize the patient's concerns and the medical issues into a concrete plan that can be carried out in the patient's system of health care. Outline a step-by-step plan, explain it to the patient, and contract about the next step. Be explicit about your next contact with the patient ("I'll see you in clinic in 2 weeks") or the fact that you won't see the patient ("I'm going to be rotating off service, so you will see Dr. Back in clinic"). Give the patient a phone number or a way to contact the relevant medical caregiver if something arises before the next planned contact.

What role does language play in discussions of aid-in-dying?

A variety of terms have historically been used to describe when a terminally ill patient uses a lethal dose of medication for the purpose of ending his or her life (or having control over the timing of death). Prior to the passage of the Oregon Death with Dignity Act in 1996, the term most often used was "physician-assisted suicide" (PAS). Those who use this term feel that it is an accurate reflection of the relationship between doctor and patient and refer to the etymological roots of suicide as "auto-killing" or "self-killing." The use of this term ties the role of the physician to one that aids the patient in killing him or herself. However, implicit in the understanding of the word suicide is the notion of a premature death that is being hastened out of despair, therefore when mental illness impairs judgment, intervention to stop a suicide is ethically warranted because the person seeking suicide has lost his ability to carefully weigh the benefits and burdens of continued life. Generally speaking, persons who are suicidal are treated as though their decision-making capacity is compromised and health care providers often intervene and provide life-sustaining treatments (including involuntary psychiatric treatment) over the objections of the patient. Therefore, some people, including several national professional organizations*, object to the term suicide for the choice of a terminally ill patient to hasten death, because of the associations between suicide and mental illness. They argue that, unlike the patients with impaired judgment who request suicide, terminally ill patients who request medication under the Act have the capacity to make a rational, autonomous decision to end their lives. The term "physician aid-in-dying" is used to describe the practice authorized under the Washington, Oregon and Vermont Death with Dignity Acts and is meant to reflect the requirement that eligible persons must be decisionally competent and have a limited life expectancy of about 6 months or less. In this context, the term is meant to reflect that physicians provide assistance to patients who are otherwise going to die, and who seek help to control the timing and circumstances of their death in the face of end-of-life suffering they deem intolerable. While this term evades the mental health connotations associated with the word suicide, people who object to the use of "physician aid in dying" suggest that it could include other practices that are clearly outside the legal bounds of the three states' Death with Dignity Acts, e.g. a patient who receives assistance in ingesting the medication, which would constitute euthanasia (see below). Here we use the term physician aid-in-dying to reflect the practice that is legal under the Washington Death with Dignity Act. It is important to note that both terms, "physician assisted suicide" and "physician aid-in-dying" are value-laden and may reflect the speaker or writer's political or ideological support for or objection to the practice. Recent research has detailed the need for open and honest discussion on end of life issues. This discussion should supersede any debate over the use of particular terms or language. Acknowledging the power of both historic and contemporary terminology, will help flesh out both sides of this sensitive and powerful debate. *The Oregon Department of Public Health, American Public Health Association, American Psychological Association, American Academy of Hospice and Palliative Care, American Medical Women's Association, and the American Medical Student Association have adopted the term patient directed dying or physician aid-in-dying and have rejected the term physician-assisted suicide. Neither term is used in Oregon, Washington or Vermont Death with Dignity laws.

What is paternalism and who is responsible for health?

Cases where harm to others is absent or less easily established stir much more debate because they raise the specter of paternalism. Paternalism occurs when the state or an individual interferes with the preferences of a person for her own benefit (e.g., to promote her health) (Childress et al. 2002). Indeed, public health action often seeks to influence human behavior that arguably does no harm to others. Because chronic diseases now account for the majority of deaths, and personal behaviors make a nontrivial contribution to their onset and progression (McGinnis et al. 2002), much public health research and action seeks to change behaviors whose ill effects are felt primarily by those who engage in them. The least controversial tactics for promoting behavior change are educational in nature-providing people with information, whether it be the calorie content of foods, the health effects of certain behaviors, and so on. Educational campaigns are not coercive, as individuals can choose to heed them or not, and they can be argued to support more informed and thus autonomous citizens. Even Mill would likely have endorsed (non-controversial) public education campaigns for this reason (Nuffield Bioethics Council 2007). Educational strategies, however, often fail to produce behavior change, particularly among socially disadvantaged groups. Take obesity, which disproportionately affects minority and low socioeconomic groups. Education initiatives are most effective for individuals who have the time, interest, motivation, and resources to act on the information and opportunities made available. People who work long hours or double shifts, or who lack access to affordable gyms or safe spaces to walk or exercise, for example, may not have the time, energy, or financial resources to change habits that are embedded in ways of life. Thus, strategies based on education, information, and individual choice can reproduce or even exacerbate health disparities between more and less advantaged groups (Link, Phelan 1995; Link 2008). To address this challenge, social and material conditions may be remade to support health and healthy choices. In the case of obesity, such measures might include regulating food production (e.g., government choices about which foods to subsidize, banning excess sodium in processed foods), prohibiting the sale of soda and other sugary beverages in schools, and planning communities that will promote walking, recreation, and otherwise positive social connections. This structural approach to public health is, however, often criticized in political contexts such as the United States, where personal responsibility (for health and other goods) and individualism are highly valued. Debate over responsibility for health is not new (Reiser 1985) but it has intensified in the last few decades. One prominent way this debate has played out in U.S. health policy is the increased use of incentives and disincentives to promote the adoption of health-enhancing behaviors in both the private and public sectors. Private employers and state governments have ramped up their wellness programs, offering employees a variety of goods to participate in health screenings, on-site exercise programs, among other activities. While most programs use rewards in some form, the use of penalties is also on the upswing. A high-profile and much criticized example is West Virginia's Medicaid program, which now requires beneficiaries to sign a personal responsibility agreement in order to receive an "enhanced" package of services (Blacksher 2008). Critics oppose such programs on numerous grounds. One common refrain is that penalties 'blame the victim'. Responsibility lies not with these individuals, critics charge, but rather with social institutions and society at large. This claim of social responsibility for health is a critical plank in a larger argument that health (and/or the social determinants thereof) should be subject to considerations of social justice. Agents of public health often claim that health inequalities associated with social class, race, and ethnicity constitute a social injustice because they are caused, at least in part, by socially controllable factors. As Wikler points out, "The locus of blame is key, for if blame is placed on the individual, social structure is exculpated, and the resulting suffering and premature death will not be counted as a social injustice" (Wikler 2006).

How should I work with hospital chaplains?

It is heartening to know that the physician is not alone in relating to the spiritual needs of the patient, but can enjoy the team work of well trained hospital chaplains who are prepared to help when the spiritual needs of the patient are outside the competence of the physician. Most of today's hospital chaplains are board certified and have undergone specialized training in listening to and talking with patients. Rev. Stephen King, PhD, Director of Chaplaincy at Seattle Cancer Care Alliance describes the requirements for chaplains as follows: Board Certification Objective Requirements: 72 semester hours/108 quarter hours Masters in theological studies 1 fulltime year equivalent in clinical pastoral education (CPE) (ACPE residency) Ordained or commissioned by a religious/spiritual tradition (accountability) [e.g., Christian, Jewish, Muslim, Hindu, Buddhist, Sufi, Sikh, Interfaith] Endorsed by a religious/spiritual tradition for chaplaincy (accountability) (Rev. Stephen King, personal communication, Need date) Chaplains play an important role in a team approach to caring for patients. The onset of serious illness or accident often induces spiritual reflection as patients wonder, "what is the meaning of my life now?" Others ponder questions of causation, "why did this happen to me?" As mentioned earlier, some patients in the midst of a health crisis may also face a religious struggle or feel angry with God for allowing this to happen. Still others are concerned as to whether the physician's recommendations for treatment are permissible in the patient's faith community. Practical questions concerning the permissibility of procedures such as an autopsy, in vitro fertilization, pregnancy termination, blood transfusion, organ donation, the removal of life supports such as ventilators, dialysis, or artificially administered nutrition and hydration, or employment of the Death with Dignity Act, arise regularly for persons of faith. In many cases, the chaplain will have specialized knowledge of how medical procedures are viewed by various religious bodies. In each case, the chaplain will first attempt to elicit the patient's current understanding or belief about the permissibility of the procedure in question. The chaplain is also prepared to respond to patients experiencing religious struggle through expert listening and communication skills. The chaplain is a helpful resource in providing or arranging for rituals that are important to patients under particular circumstances. Some patients may wish to hear the assurances of Scripture, others may want the chaplain to lead them in prayer, and still others may wish for the sacraments of communion, baptism, anointing, (formerly, the last rites), depending upon their faith system. The chaplain may provide these direct services for the patient, or may act as liaison with the patient's clergy person. In one case, a surgeon called for the chaplain to consult with a patient who was inexplicably refusing a life-saving surgical procedure. The chaplain gently probed the patient's story in an empathic manner, leading the patient to "confess" to a belief that her current illness was God's punishment for a previous sin. The ensuing discussion revolved around notions of God's forgiveness and the patient's request for prayer. In this case, the chaplain became the "embodiment" of God's forgiveness as he heard the patient's confession, provided reassurance of God's forgiving nature, and offered a prayer acknowledging her penitence and desire for forgiveness and healing. The conference with the chaplain opened the door for this patient to accept the care plan that she had refused earlier. In another case, a neonatologist summoned the chaplain to the NICU when it became apparent that a newly born premature infant was not going to live and the parents were distraught at the notion that their baby would die without the sacrament of baptism. In this case, the chaplain was able to discuss the parent's beliefs, to reassure them that their needs could be met, and to provide a baptism service with the parents, the neonatologist and the primary nurse in attendance before the baby died. The chaplain also notified the parent's home-town pastor and helped make arrangements for the parents to be followed back home in their grieving process after the funeral. Sometimes, in the fast moving delivery of health care, the chaplain, by his or her job description, is the only one on the team with sufficient time to follow up on these important patient needs and concerns.

Why is it important to attend to spirituality in medicine?

Religion and spiritual beliefs play an important role for many patients. When illness threatens the health, and possibly the life of an individual, that person is likely to come to the physician with both physical symptoms and spiritual issues in mind. An article in the Journal of Religion and Health claims that through these two channels, medicine and religion, humans grapple with common issues of infirmity, suffering, loneliness, despair, and death, while searching for hope, meaning, and personal value in the crisis of illness (Vanderpool & Levin, 1990). Definitions: Religion is generally understood as a set of beliefs, rituals and practices, usually embodied within an institution or an organization. Spirituality, on the other hand, is commonly thought of as a search for what is sacred in life, one's deepest values, along with a relationship with God, or a higher power, that transcends the self. Persons may hold powerful spiritual beliefs, and may or may not be active in any institutional religion. Spirituality can be defined as "a belief system focusing on intangible elements that impart vitality and meaning to life's events" (Maugans, 1996). Many in the baby-boom generation who claim not to be religious, admit to a sense of "reverence" for life, similar to the concept championed by theologian-philosopher Albert Schweitzer. Many physicians and nurses have intuitive and anecdotal impressions that the beliefs and religious practices of patients have a profound effect upon their existential experiences with illness and the threat of dying. Recent research supports this notion. When patients face a terminal illness, religious and spiritual factors often figure into their coping strategies and influence important decisions such as the employment of advance directives, the living will and the Durable Power of Attorney for Health Care. Considerations of the meaning, purpose and value of human life are used to make choices about the desirability of CPR and aggressive life-support, or whether and when to forego life support and accept death as appropriate and natural under the circumstances (Puchalski et al., 2009; McCormick et al., 2012; Ai, 2008). Many are comforted in the face of a health-crisis with an inner calm that is founded on their deep trust in God's loving care for them in all situations. On the other hand, Pargament's research reveals that some patients in similar circumstances are involved in religious struggle that may have deleterious effects upon their health outcomes (Pargament et al., 2001). He identifies specific forms of religious struggle that are predictive of mortality. Patients who feel alienated from God, unloved by God, or punished by God, or attribute their illness to the work of the devil were associated with a 19% to 28% increased risk of dying during the 2 year follow up period (ibid). A study of religious coping in patients undergoing autologous stem cell transplants also suggests that religious struggle may contribute to adverse changes in health outcomes for transplant patients (Sherman et al., 2009). Referral of these patients to the chaplain, or appropriate clergy, to help them work through these issues may ultimately improve clinical outcomes (Pargament, et al., 2001). Further, the Joint Commission mandates that healthcare institutions ensure that patients' spiritual beliefs and practices are assessed and accommodated (Joint Commission on the Accreditation of Healthcare Organizations, 2003). Handzo, a chaplain, and Koenig, a physician suggest that the physician's role (as a generalist in spirituality) is to briefly screen patient's spiritual needs as they relate to health care and to refer to the chaplain (a specialist in pastoral care), as appropriate (Handzo & Koenig, 2004).

Association of American Medical Colleges (AAMC) Report III- Contemporary Issues in Medicine: Communication in Medicine Medical School Objectives Project (MSOP III) Learning Objectives

With regard to spirituality and cultural issues, before graduation students will have demonstrated to the satisfaction of the faculty: MSOP III The ability to elicit a spiritual history The ability to obtain a cultural history that elicits the patient's cultural identity, experiences and explanations of illness, self-selected health practices, culturally relevant interpretations of social stress factors, and availability of culturally relevant support systems An understanding that the spiritual dimension of people's lives is an avenue for compassionate care giving The ability to apply the understanding of a patient's spirituality and cultural beliefs and behaviors to appropriate clinical contexts (e.g., in prevention, case formulation, treatment planning, challenging clinical situations) Knowledge of research data on the impact of spirituality on health and on health care outcomes, and on the impact of patients' cultural identity, beliefs, and practices on their health, access to and interactions with health care providers, and health outcomes An understanding of, and respect for, the role of clergy and other spiritual leaders, and culturally-based healers and care providers, and how to communicate and/or collaborate with them on behalf of patients' physical and/or spiritual needs An understanding of their own spirituality and how it can be nurtured as part of their professional growth, promotion of their well-being, and the basis of their calling as a physician. (Association of American Medical Colleges, 1999) Beyond the four years of medical school, residency programs, particularly those with a primary care focus and a palliative care focus, are incorporating education in spirituality training residents. In addition, Continuing Medical Education (CME) events are now offered to practicing physicians through a series of annual conferences on "Spirituality in Medicine," the first of which was hosted by Harvard Medical School with Herbert Benson, MD, as facilitator. Dr. Benson and Dr. Christina Puchalski combined efforts as co-directors of this conference for several years. Since 2008, Dr. Puchalski has directed an annual Spirituality and Health Care Summer Institute sponsored by the George Washington Institute for Spirituality and Health (GWISH) in Washington D.C. Summary: Patients facing serious illness, accident, or death often experience a crisis of meaning. Spirituality is often defined as "the search for meaning." Spirituality may, or may not be accompanied by a particular religion. Some patients are profoundly comforted by their spiritual beliefs. Others may encounter religious struggle or negative ways of coping with illness. It is important for patients that their cultural, spiritual, and religious beliefs be recognized and integrated in the development of a plan of care and in decisions that are made concerning end-of-life care. Respect for patient values and beliefs requires competent communication skills in health care professionals. In recent years, considerable effort has been made in professional training to foster patient centered communication that is cognizant and respectful of patients' cultural and spiritual values and how these may be incorporated into optimal patient care. The American Association of Medical Colleges (AAMC) has developed medical school objectives (MSO) related to spirituality and culture that every student should achieve before graduating. Residency training programs and continuing medical education programs foster continued learning after medical school. However, there is room for improvement. In a recent survey of NW physicians, only 17% of responding physicians reported routinely inquiring about religion/spirituality with new patients, while in a crisis situation, 49% reported inquiry into the patient's religion/spirituality. 83% of respondents agreed that doctors should refer to chaplains (King, et al., 2013). New resources are available for educators such as those developed by the George Washington Institute for Spirituality and Health (GWISH), including on-line materials that are easily accessible to both students and faculty. Research into the relationship between religion/spirituality and health outcomes and patient well-being is burgeoning. Health care professionals ought not to neglect their own psychological and spiritual well-being. Health care professionals work in an intense and stressful environment, frequently exposed to the suffering of others and to companying with the dying. Such work requires that we stay in touch with our own feelings and that which provides meaning and value within our own lives, while working in a profession dedicated to the care of others.

Advance care planning is a process to help patients with decision-making capacity guide future health care decisions in the event that they become unable to participate directly in their care. The process, when accomplished comprehensively, involves four steps:

(1) thinking through one's relevant values and preferences, (2) talking about one's values and preferences with one's spokesperson, close family members and health care providers, (3) documenting them with an advance directive, and (4) reviewing them periodically and updating them as needed.

Is a psychiatry consult required to determine decision making capacity?

A psychiatry consult is not required, but can be helpful in some cases. Psychiatrists are trained in interviewing people about very personal, sensitive issues, and thus can be helpful when patients are facing difficult choices with fears or concerns that are difficult to talk about. Similarly, if decision making capacity is clouded by mental illness, a psychiatrist's skill at diagnosis and potential treatment of such disorders can be helpful.

What is my responsibility when a patient endangers her health by refusing a treatment?

Adults have a moral and legal right to make decisions about their own health care, including the right to refuse treatments that may be life-saving. The physician has a responsibility to make sure that the patient understands the possible and probable outcomes of refusing the proposed treatment. The physician should attempt to understand the basis for the patient's refusal and address those concerns and any misperceptions the patient may have. In some cases, enlisting the aid of a leader in the patient's cultural or religious community may be helpful.

How do mistakes occur?

All physicians make mistakes, and most mistakes are not the result of negligence. A physician may make a mistake because of an incomplete knowledge base, an error in perception or judgment, or a lapse in attention. Making decisions on the basis of inaccurate or incomplete data may lead to a mistake. The environment in which physicians practice may also contribute to errors. Lack of sleep, pressures to see patients in short periods of time, and distractions may all impair an individual's ability to avoid mistakes.

You are a 25-year-old female medical student doing a rotation in an HIV clinic. Sara is a 30-year-old woman with advanced HIV who dropped out of college after she found that she contracted HIV from her husband, who has hemophilia. In talking to Sara, it turns out you share a number of things--you are from the same part of Montana originally, also have young children, and like to cook. Later in the visit, when you suggest that she will need some blood tests, she gets very angry and says, "What would you know about this?" What happened?

Although the protocol for breaking bad news is helpful, it doesn't cover everything. There are instances when you may provoke a reaction from a patient because you remind them of someone else--or, as in this case, themselves. In these instances it can be helpful to step back, get another perspective (perhaps from someone in clinic who has known Sara), and try not to take this reaction too personally--even though it is likely that Sara will know how to really bother you.

What if a patient changes her mind about advance directives?

As long as a patient remains able to participate in medical decisions, both documents are revocable. Informed decisions by patients with decision-making capacity always supersede a written directive.

Does the cause of arrest matter?

Because arrests in the OR are often due to hemorrhage or medication effects, rather than the patient's underlying disease, physicians may feel that their actions "caused" the arrest, and they are ethically obliged to resuscitate the patient, even if the patient has clearly expressed wishes to the contrary. But competent patients, or their appropriate surrogates, have the right to refuse medical procedures and care, even if the care is to counteract the effects of previous medical intervention.

Your 36-year-old patient has just tested positive for HIV. He asks that you not inform his wife of the results and claims he is not ready to tell her yet. What is your role legally? What would you say to your patient?

Because the patient's wife is at serious risk for being infected with HIV, you have a duty to ensure that she knows of the risk. While public health law requires reporting both your patient and any known sexual partners to local health officers, it is generally advisable to encourage the patient to share this information with his wife on his own, giving him a bit more time if necessary.

Mr. D is a 64-year-old man with multiple complex medical problems including uncontrolled diabetes, untreated depression, coronary artery disease (status post myocardial infarction with multiple stents), painful peripheral vascular disease, hypertension, hyperlipidemia, chronic hepatitis C, spinal stenosis and ongoing 2 pack per day tobacco use. He is on a long list of 16 medications and insulin, though does not take any of his medications with regularity. Mr. D returns to clinic and all of the above medical problems are not well controlled. He has not been taking medication or following up with specialty care as advised. He continues to smoke and has not improved his extremely sedentary activity level. He requests that something more be done.

Be aware of your own feelings. It is frustrating to devote time and compassion to patients when they do not take your advice. Still, you provide essential service to the patient by listening to his concerns and providing an opportunity for care. Prepare for the visit by acknowledging your frustrations before seeing the patient and strategize about how you will make the most of his visit. Set your own goal for the visit. Also ask him why he is coming to see you. If it is unrealistic to expect Mr. D to comply with all his medical therapies, if he feels respected and heard, he will be more likely to heed your advice. Consider asking which of his medical problems is bothering him most and negotiate with him on ways that he can work toward mutually agreed upon goals. Explore causes of his non-compliance and problem solve together toward a more coherent medical plan. Emphasize that the two of you work together to improve his health and establish one or two goals for his next visit (e.g. better diabetes control). Plan to maintain a patient and respectful demeanor even though you might actually be feeling frustrated. Enlist other providers (pharmacist, nurse, counselor or social worker) to help.

How does informed consent apply to children?

Children do not have the decision-making capacity to provide informed consent. Since consent, by definition, is given for an intervention for oneself, parents cannot provide informed consent on behalf of their children. Instead they can provide informed permission for treatment. For older children and adolescents, assent should always be sought in addition to the authorization of legal surrogates. Adolescents and mature minors are legally and ethically authorized to provide informed consent if they are emancipated, and in many states, including Washington, they may provide consent for matters regarding sexual and reproductive health, mental health, and substance abuse. See your state's legislation regarding mature minors and consent laws. The primary responsibility of the physician is the well-being of the child. Therefore, if the parental decision places the child at risk of harm then further action may be indicated. When there are differences in opinion between the parents and physicians that cannot be resolved ethics consultation may be pursued, and legal avenues may be pursued when all other means have failed. Children should be included in decision-making at a developmentally appropriate level and assent should be sought when possible.

When should communicable diseases be reported to public health authorities?

Each state has specific statutes that identify specific diseases with public health implications, such as communicable diseases, which require reporting. If you are not sure whether reporting is required or advised, contact your local public health authority for guidance. Beyond this legal requirement lies the ethical question of when it is justified to potentially violate confidentiality to protect the public's health. It is ethically justified to disclose a diagnosis to public health authorities if the risk to the public has the following features: the probability of risk is high the magnitude of risk is serious the risk relates to an identifiable individual or group Case 1 below discusses this question in the context of a patient with sexually transmitted disease.

Components of ethically valid informed consent for research.

Given the appreciation that informed consent is at the crux of human subjects protection, it is not surprising that the regulations reflect extensively upon the necessary elements of the consent document itself as well as on the informed consent process. For an informed consent to be ethically valid, the following components must be present: Disclosure: The informed consent document must make clear that the study is a research study, and not clinical therapy. The potential participant must be informed as fully as possible of the nature and purpose of the research, the procedures to be used, the expected benefits to the participant and/or society, the potential of reasonably foreseeable risks, stresses, and discomforts, and alternatives to participating in the research. There should also be a statement that describes procedures in place to ensure the confidentiality of data and anonymity of the participant. The informed consent document must also disclose what compensation and medical treatment are available in the case of a research-related injury. The document should make it clear whom to contact with questions about the research study, research subjects' rights, and in case of injury. Understanding: The participant must understand what has been explained and must be given the opportunity to ask questions and have them answered by someone fully conversant in the study particulars. The informed consent document must be written in lay language, avoiding any technical jargon. The potential participant must be able to read and/or understand the language in which the consent form is written. Consent forms for multinational research must be translated into the respective language for each participating country and back-translated to verify accuracy. Voluntariness: The participant's consent to participate in the research must be voluntary, free of any coercion or inflated promise of benefits from participation. Care should be taken that the consent form is administered by someone who does not hold authority over the participant. Ideally, the potential participant is given the opportunity to discuss their participation in the study with family, trusted friends, or their physician before reaching a decision. Competence: The participant must be competent to give consent. If the participant is not competent due to mental status, disease, or emergency, a designated surrogate may provide consent if it is in the participant's best interest to participate. In certain emergency cases, consent may be waived due to the lack of competence of the participant and absence of an appropriate surrogate. In the event that there is a question about competence, mental status exams may be administered. Consent: The potential human subject must authorize his/her participation in the research study, preferably in writing. If there is no need to collect personally identifiable information, and a signature on the consent form would be the only thing linking the subject to the study, an oral or implicit consent may be more appropriate. Children who cannot read or write should still signal their willingness to participate by an affirmative act (for example, nodding their head). Consent by minors is referred to as assent. Exculpatory language: No informed consent may contain any exculpatory language by which the participant waives any legal rights or releases the investigator or sponsor from liability for negligence.

What goals should I have in mind when working towards a decent death for my patient?

I have several working clinical goals when I am caring for someone near the end of life. I work towards: Control of pain and other physical symptoms. The physical aspects of care are a prerequisite for everything that follows. Involvement of people important to the patient. Death is not usually an individual experience; it occurs within a social context of family, significant others, friends, and caregivers. A degree of acceptance by the patient. Acceptance doesn't mean that the patient likes what is going on, and it doesn't mean that a patient has no hopes--it just means that he can be realistic about the situation. A medical understanding of the patient's disease. Most patients, families, and caregivers come to physicians in order to learn something about what is happening medically, and it is important to recognize their need for information. A process of care that guides patient understanding and decision making. One great physician does not equal great care--it takes a coordinated system of providers.

When is it justifiable to discontinue life-sustaining treatments?

If the patient has the ability to make decisions, fully understands the consequences of their decision, and states they no longer want a treatment, it is justifiable to withdraw the treatment. Treatment withdrawal is also justifiable if the treatment no longer offers benefit to the patient.

What should occur if the patient cannot give informed consent?

If the patient is determined to be incapacitated/incompetent to make health care decisions, a surrogate decision maker must speak for her. There is a specific hierarchy of appropriate decision makers defined by state law (also see the DNR topic page). If no appropriate surrogate decision maker is available, the physicians are expected to act in the best interest of the patient until a surrogate is found or appointed. In rare circumstances, when no surrogate can be identified, a guardian ad litem may have to be appointed by the court. Confer with social work and risk management if you have trouble finding a legal surrogate for the patient.

I'm noticing what looks like addictive behavior in one of my classmates. What should I do?

Impaired students become impaired physicians. You are entering a profession that carries an obligation to its members for self-regulation. As a student, your classmate has an opportunity to seek help before serious harm comes to himself or herself, or to one of his or her patients. Once licensed, you will have a legal obligation to report colleagues to the medical board if they are "unable to practice medicine with reasonable skill and safety to patients by reason of illness, drunkenness, excessive use of drugs, narcotics, chemicals, or any other type of material, or as a result of any mental or physical conditions (Revised Code of Washington 18.72.165)." The UW Counseling Office or the Assistant Dean for Student Affairs can help you arrange an intervention if you have these concerns.

Your patient has been suffering from chronic low back pain for many years now. She voices her frustration with the various treatment modalities that you have been trying and says she is considering getting acupuncture. How do you respond?

In this case, there are few clearly effective treatments for the medical condition. Hence, complementary approaches may be a reasonable recommendation, assuming they are not harmful. While you may know little about acupuncture yourself, you may encourage your patient to consult with local experts to inform herself about what acupuncture can offer. Encourage your patient to stay in contact with you and inform you about both the perceived benefits and any side effects she may experience.

What is informed consent?

Informed consent is the process by which the treating health care provider discloses appropriate information to a competent patient so that the patient may make a voluntary choice to accept or refuse treatment. (Appelbaum, 2007)1 It originates from the legal and ethical right the patient has to direct what happens to her body and from the ethical duty of the physician to involve the patient in her health care.

By disclosing a mistake to my patient, do I risk having a malpractice suit filed against me?

It has been shown that patients are less likely to consider litigation when a physician has been honest with them about mistakes. Many lawsuits are initiated because a patient does not feel they have been told the truth. Litigation is often used as a means of forcing an open and honest discussion that the patient feels they have not been granted. Furthermore, juries look more favorably on physicians who have been honest from the beginning than those who give the appearance of having been dishonest.

Is it justifiable to withhold or withdraw care because of costs?

It is rarely justifiable to discontinue life-sustaining treatment for cost reasons alone. While we should always try to avoid costly treatments that offer little or no benefit, our obligation to the patient outweighs our obligation to save money for health care institutions. There are rare situations in which costs expended on one terminally ill patient could be clearly better used on another, more viable patient. For instance, a terminally ill patient with metastatic cancer and septic shock is in the last ICU bed. Another patient, young and previously healthy, now with a self-limited but life-threatening illness, is in the emergency room. In such cases, it may be justifiable to withdraw ICU treatment from the terminally ill patient in favor of the more viable one. Even so, such decisions must be carefully considered, and made with the full knowledge of patients and their surrogate decision makers.

When and where should I initiate advance care planning?

It is unknown when or where advance care planning should ideally occur. It is generally thought that this should occur initially in the outpatient setting, and then be reviewed upon admission to and discharge from inpatient settings. It is also recommended that whenever there is a significant change in a patient's social or health status, the patient's views about advance care planning should be reviewed.

What is the difference between a futile intervention and an experimental intervention?

Making a judgment of futility requires solid empirical evidence documenting the outcome of an intervention for different groups of patients. Futility establishes the negative determination that the evidence shows no significant likelihood of conferring a significant benefit. By contrast, treatments are considered experimental when empirical evidence is lacking and the effects of an intervention are unknown.

What role should the physician's personal feelings and beliefs play in the physician-patient relationship?

Occasionally, a physician may face requests for services, such as contraception or abortion, which raise a conflict for the physician. Physicians do not have to provide medical services in opposition to their personal beliefs. In addition, a nonjudgmental discussion with a patient regarding her need for the service and alternative forms of therapy is acceptable. However, it is never appropriate to proselytize. While the physician may decline to provide the requested service, the patient must be treated as a respected, autonomous individual. Where appropriate, the patient should be provided with information about how to obtain the desired service.

When can confidentiality be breached?

Overriding concerns can lead to the need to breach confidentiality in certain circumstances. Exception 1: Concern for the safety of other specific persons Access to medical information and records by third parties is legally restricted. Yet, at the same time, clinicians have a duty to protect identifiable individuals from any serious, credible threat of harm if they have information that could prevent the harm. The determining factor is whether there is good reason to believe specific individuals (or groups) are placed in serious danger depending on the medical information at hand. An example is homicidal ideation, when the patient shares a specific plan with a physician or psychotherapist to harm a particular individual. The California Tarasoff case exemplifies the challenges providers face in protecting confidentiality. In that case a graduate student, Prosinjit Podder, disclosed to a counselor affiliated with Berkeley University that he intended to obtain a gun and shoot Tatiana Tarasoff. Dr. Moore, the psychologist, found Podder's threat credible. Dr. Moore then faced dual obligations: protect Tatiana Tarasoff from harm and protect Mr. Podder's confidentiality. Dr. Moore sent a letter to campus police about the threat. They spoke to Mr. Podder, told him to stay away from Tatiana, but determined he was not a danger to her. He later stalked, stabbed and killed Tatiana. Tatiana's parents sued campus police and the university's health service for failure to warn Tatiana. The lower court refused to hear the case, claiming that Dr. Moore had an obligation to protect Podder's confidentiality. Tarasoff's parents appealed and the California Supreme Court ruled that, "the discharge of this duty may require the therapist to take one or more of various steps. Thus, it may call for him to warn the intended victim, to notify the police, or to take whatever steps are reasonably necessary under the circumstances." (Ref. Tarasoff case). The implication of this ruling is that a duty to warn third parties of imminent threats trumps a duty to protect patient confidentiality, however, it is usually difficult for a therapist or health care provider to accurately ascertain the seriousness and imminence of a threat. Tarasoff has subsequently been interpreted to endorse the provider's duty to warn when a patient threatens an identifiable victim. Ethically, most would agree that a duty to warn an innocent victim of imminent harm overrides a duty to confidentiality, but these cases are rare and judgment calls of this sort are highly subjective. Hence, the duty to maintain confidentiality is critical, but may be overridden in rare and specific circumstances. Exception 2: Legal requirements to report certain conditions or circumstances State law requires the report of certain communicable/infectious diseases to the public health authorities. In these cases, the duty to protect public health outweighs the duty to maintain a patient's confidence. From a legal perspective, the State has an interest in protecting public health that outweighs individual liberties in certain cases. For example, reportable diseases in Washington State include (but are not limited to): measles, rabies, anthrax, botulism, sexually transmitted diseases, and tuberculosis. Suspected cases of child, dependent adult, and elder abuse are reportable, as are gunshot wounds. Local municipal code and institutional policies can vary regarding what is reportable and standards of evidence required. Stay informed about your state and local policies, as well as institutional policies, governing exceptions of patient confidentiality. A Test for Breach of Confidentiality In situations where you believe an ethical or legal exception to confidentiality exists, ask yourself the following question: will lack of this specific patient information put another person or group you can identify at high risk of serious harm? If the answer to this question is no, it is unlikely that an exception to confidentiality is ethically (or legally) warranted. The permissibility of breaching confidentiality depends on the details of each case. If a breach is being contemplated, it is advisable to seek legal advice before disclosure.

What if maternal decisions seem to be based on unusual beliefs?

Parents are granted wide discretion in making decisions about their children's lives. However, when the exercise of certain beliefs would disadvantage the child's health in a serious way, there are limits in exercising this discretion (for further discussion, see the pediatric portion of the Cross-Cultural Issues topic page).

The Role of Patient Autonomy

Since the original inception of DNAR orders, respecting the rights of adult patients and their surrogates to make medical decisions, otherwise known as respect for autonomy or respect for persons, has been emphasized. This concept is reinforced legally in the Patient Self Determination Act of 1991, which requires hospitals to respect the adult patient's right to make an advanced care directive and clarify wishes for end-of-life care. In general, an emphasis on improving communication with patients and families is preferred over physicians making unilateral decisions based on appeals to medical futility regarding the resuscitation status of their patients. See below.

What about "slow codes" or "show codes"?

Slow codes and show codes are forms of "symbolic resuscitation." A "slow code" is an act performed by the health care providers that resembles CPR yet is not the full effort of resuscitation while a "show code" is a short and vigorous resuscitation performed to benefit the family while minimizing harm to the patient (Frader et al., 2010). Slow and show codes are ethically problematic. In general, performing slow and show codes undermines the rights of patients to be involved in clinical decisions, is deceptive, and violates the trust that patients have in health care providers.

I had a long talk with the patient yesterday, and today the nurse took me aside to say that the patient doesn't understand what's going on! What's the problem?

Sometimes patients ask the same question of different caregivers, sometimes they just didn't remember it all, and sometimes they need to go over something more than once because of their emotional distress, the technical nature of the medical interventions involved, or their concerns were not recognized and addressed.

A 22-year-old woman is admitted to the hospital with a headache, stiff neck and photophobia but an intact mental status. Lab test reveal cryptococcal meningitis, an infection commonly associated with HIV infection. When given the diagnosis, she adamantly refuses to be tested for HIV. Should she be tested anyway by the medical staff?

Testing for HIV, as for any other medical procedure should be done only with the informed consent of the patient. Testing without consent is unethical in this setting. The physician's role in the care of this patient is ongoing support, education and guidance about her various options for care.

Is it ever appropriate to do a procedure for the first time without supervision

The "see one, do one, teach one" model of medical training has become something of a urban legend. However, on a busy service, you will be probably be asked to "go consent Mr. Jones" or "just start a line on Mrs. Smith." If you have never done either of these activities before, it is your responsibility to ask for appropriate supervision before beginning the procedure. Emphasize your interest in learning the new skill as well as your interest in learning it under the best conditions possible.

Is there a resource that summarizes the various state laws regarding adolescent consent?

The Guttmacher Institute publishes a summary of the adolescent consent laws that can be found at: www.guttmacher.org/statecenter/spibs/spib_OMCL.pdf

How should I advise a patient if he believes that some family members will disagree with his wishes?

The patient should be informed that the best way to prevent disagreements is to communicate with everyone ahead of time to let them know who has been chosen as a spokesperson and what kind of approach to health care he wants.

A patient tells his family that he would never want to be "kept alive like a vegetable". What is meant by the term "vegetable"?

The use of this expression is as vague as saying, "I don't want any heroics or extraordinary treatments" or, "Pull the plug if I'm ever in ...." If these types of comments in advance care planning discussions are not clarified, they are not helpful. For some patients being a "vegetable" means being in a coma, for others it means not being able to read. Gently discuss the patient's hopes, fears, and specific preferences in order to clarify the meaning of the term and the patient's preferences for end-of-life care.

What is "medical futility"?

"Medical futility" refers to interventions that are unlikely to produce any significant benefit for the patient. Two kinds of medical futility are often distinguished: Quantitative futility, where the likelihood that an intervention will benefit the patient is exceedingly poor, and Qualitative futility, where the quality of benefit an intervention will produce is exceedingly poor. Both quantitative and qualitative futility refer to the prospect that a specific treatment will benefit (not simply have a physiological effect) on the patient. Futility does not apply to treatments globally, to a patient, or to a general medical situation. Instead, it refers to a particular intervention at a particular time, for a specific patient. For example, rather than stating, "It is futile to continue to treat this patient," one would state, "CPR would be medically futile for this patient."

What are the goals and expected outcomes of advance care planning?

The goals of advance care planning are four-fold. These goals reflect respect for the principles of patient autonomy (right to self-determination in light of personal interests including goals, preferences, and concerns for one's family), beneficence (promoting good) and non-maleficence (avoiding harm). In the event of decisional incapacity, they are to: Maximize the likelihood that medical care serves the patient's goals, Minimize the likelihood of over- or under-treatment, Reduce the likelihood of conflicts between a patient's spokesperson, family members and health care providers, and Minimize the burden of decision making on the spokesperson and/or family members. As a health care provider who engages her patients in advance care planning, you can expect to better understand your patient's views about who they want you to communicate with when health care decisions are required and they can no longer participate in decisions, their goals for medical treatment and care under situations in which they cannot participate in decisions, their preferred approach to end-of-life care, and their hopes (e.g., to live longer, to have quality relationships with loved ones), as well as their fears about medical interventions (e.g., permanent loss of cognitive functioning, loss of dignity). During the advance care planning process the provider may want to discuss patient preferences for treatment and care if a patient has an illness with known possible sequelae. For example, someone with mild dementia may be able to state if she would want to receive antibiotics and be hospitalized if she developed pneumonia at a time when she could no longer provide self-care or recognize family members. In addition, you can expect to have a discussion that clarifies misunderstandings. For example, patients often think that cardiopulmonary resuscitation is successful 80% of the time, that mechanical ventilation is a fancy word for nasal prongs, and that coma patients still have the ability to enjoy life. These discussions also may reveal patients' treatment preferences that fall outside standard practices in medicine (e.g., herbal remedies, medically futile treatments). When these occur, they provide opportunities to negotiate about culturally sensitive approaches to health care and establish limits to patient autonomy based on professional standards. You can also expect to have better communication with patient's family members or loved ones because there should be enhanced shared understanding of patient values and wishes. Thus, you can expect to have fewer conflicts with family members about the approach to end-of-life care. Advance directives will not be the outcome of most advance care planning. However, advance care planning may lead to completion of a Physician Order for Life Sustaining Treatment (POLST) form, an order that reflects the patient's goals and wishes for inpatient and outpatient medical care. (See Advance Directives and Additional Readings below.) At a minimum, patients should become familiar with the concept and rationale for advance care planning. Some patients will want to mull things over, whereas others will want to discuss the topic with their close friends or family and health care providers. Fewer will be ready to sign documents and even fewer will be interested in personalizing their advance directives (e.g., attaching additional information, such as worksheets that characterize values and preferences) so that they are clear and contain pertinent information with clinical relevance.

After a goal oriented conversation, Mrs. W continues to request to be fully resuscitated in the setting of cardiopulmonary arrest. However, several days later, despite a worsening clinical condition, Mrs. W still requests to be a "full code." Your intern suggests that you sign her out as a "slow code." Should you do this?

"Slow codes" are deceitful, and therefore are not ethically justifiable. During slow codes, health care providers act in such a manner that provides families with the perception that they respect patients' decisions yet they knowingly do not provide a full resuscitative effort. This approach has the potential to disrupt the patient-physician relationship. Rather than acting in a deceitful manner, ongoing conversations regarding Mrs. W's goals while remaining transparent regarding the limitations of medicine is essential to develop a mutual and shared care plan between the medical providers and Mrs. W.

What is a difficult patient-clinician relationship?

A difficult patient-clinician relationship, occurring in approximately 15% of adult patient encounters (Krebs et al., 2006) arises when physicians encounter patients with complex, often chronic medical issues (such as chronic pain, and/or mental illness) that are influenced or exacerbated by social factors (such as poverty, abusive relationships, addiction). Previous experience with similar patients along with the social and economic disparities between the physician and patient may make the physician uncomfortable. This may lead the physician to be guarded or distant which the patient may interpret as distrust. Likewise, the therapies the doctor recommends often entail behavioral changes that the patient is unwilling or unable to make, yet the patient continues to seek the clinician's advice and treatment. The physician may become frustrated or angry because his advice is not heeded, because the diagnosis or treatment is unclear or ineffective, or because the patient is rude, seemingly ungrateful, or transgresses boundaries in the clinician-patient relationship (e.g. comes to the clinic when she does not have an appointment). The physician in turn may feel that treating the patient is futile, burdensome, and disruptive to other patients and staff. Clinicians may become angry, and avoid or sometimes 'punish' the patient.

I'm not sure how I feel about "using" vulnerable patients as teaching patients. Are we taking unfair advantage of people?

A necessary part of learning to be a physician, "practicing" on people sometimes feels uncomfortable. You can keep a few things in mind to minimize the discomfort you might feel. First, as with all your future patients, treat them with respect and ask permission before doing any observations, tests, or procedures. Second, remember that it is a privilege to learn medicine. When appropriate, convey your gratitude to the patients, acknowledging the crucial role they play in your education. Listen to your instincts as well. Sometimes it may not be appropriate to do an unnecessary duplicate examination or, for example, try more than three times to start an IV line in a patient. If the patient is uncomfortable with your presence, you must respect that and ask a more senior person on your team to complete the procedure or the exam. Unfortunately, you may notice a difference in how some housestaff or attending physicians treat patients from different socioeconomic classes. It is your responsibility to attend to these patients needs with respect and compassion. The homeless man in the ER could be very lucky to have you be the one to stitch his lacerations if you are the one who will be gentle and kind. Sometimes you can put a patient at ease if you convey that you are the member of the team with the most time and attention at the moment.

Can a patient demand that I provide them with a form of treatment that I am uncomfortable providing?

A physician is not morally obligated to provide treatment modalities that they do not believe offer a benefit to the patient or which may harm the patient. Physicians should also not offer treatments that they do not feel competent to provide or prescribe. However, it is important to take the patient's request seriously, consider accommodating requests that will not harm the patient or others, and attempt to formulate a plan that would be acceptable to both the physician and patient.

What if I see someone else make a mistake?

A physician may witness another health care provider making a major error. This places the physician in an awkward and difficult position. Nonetheless, the observing physician has some obligation to see that the truth is revealed to the patient. This should be done in the least intrusive way. If the other health care provider does not reveal the error to the patient, the physician should encourage her to disclose her mistake to the patient. Should the health care provider refuse to disclose the error to the patient, the physician will need to decide whether the error was serious enough to justify taking the case to a supervisor or the medical staff office, or directly telling the patient. The observing physician also has an obligation to clarify the facts of the case and be absolutely certain that a serious mistake has been made before taking the case beyond the health care worker involved.

Is it justifiable to deceive a patient with a placebo?

A placebo is any substance given to a patient with the knowledge that it has no specific clinical effect, yet with the suggestion to the patient that it will provide some benefit. The placebo effect is powerful, in many cases providing measurable improvement in symptoms in 20-30% of patients. In general, the deceptive use of placebos is not ethically justifiable. Specific exceptions should be rare and only considered if the following conditions are present: the condition is known to have a high placebo response rate the alternatives are ineffective and/or risky the patient has a strong need for some prescription

Your patient with cryptococcal meningitis eventually agrees to be tested for HIV and her test comes back positive. Due to her opportunistic infection she receives the diagnosis of AIDS. Should she be reported to the department of public health?

AIDS is a currently a reportable diagnosis in all 50 states of the union. Her diagnosis should be reported to the department of public health. Notably, HIV positivity without the diagnosis of AIDS is not reportable in all states. Currently, 30 of 50 states requires reporting of a positive test. It is important to find out the local states laws where you are practicing to know how to approach this problem. (See also Confidentiality.)

Regulatory requirement to administer informed consent.

According to the regulations at 45 CFR 46.116: "no investigator may involve a human being as a subject in research covered by this policy unless the investigator has obtained the legally effective informed consent of the subject or the subject's legally authorized representative."

How is advance care planning different from advance directives?

Advance care planning is the process as outlined above. Advance directives usually are the written documents designed to allow competent patients the opportunity to guide future health care decisions in the event that they are unable to participate directly in medical decision making. Verbal directives may be ethically valid, but most patients and health care providers prefer written, official documentation. If official forms are not used, health care providers should document the result of their advance care planning conversations in a medical record progress note.

Do different standards apply to withholding and withdrawing care?

Many clinicians feel that it is easier to not start (withhold) a treatment, such as mechanical ventilation, than to stop (withdraw) it. While there is a natural tendency to believe this, there is no ethical distinction between withholding and withdrawing treatment. In numerous legal cases, courts have found that it is equally justifiable to withdraw as to withhold life-sustaining treatments. Also, most bioethicists, including the President's Commission, are of the same opinion.

When can parental authority to make medical decisions for their children be challenged?

Medical caretakers have an ethical and legal duty to advocate for the best interests of the child when parental decisions are potentially dangerous to the child's health, imprudent, neglectful, or abusive. As a general rule, medical caretakers and others should challenge parental decisions when those decisions place the child at significant risk of serious harm. When satisfactory resolution cannot be attained through respectful discussion and ethics consultation, seeking involvement of a State child protection agency or a court order might be necessary.

How should I take a "spiritual history"?

Medical students are usually introduced to the concept of spiritual inquiry in courses such as "Introduction to Clinical Medicine." Medical students learn the various components of the doctor-patient interview, often beginning with topics such as the chief complaint, a history of the present illness, a psycho-social history which includes questions about religion and spirituality, and a review of organ systems. Students-in-training are often hesitant to ask questions that they regard as intrusive into the personal life of the patient until they understand there are valid reasons for asking about sexual practices, alcohol, the use of tobacco, guns, or non-prescription drugs. Religious belief and practice often fall into that "personal" category that students-in-training sometimes avoid, yet when valid reasons are offered by teachers and mentors for obtaining a spiritual history, students readily learn to incorporate this line of questioning into the patient interview. Often, the spiritual history can be incorporated into what we may now want to call the "bio-psycho-social-spiritual" patient history. Students are taught to make a transition by simply stating something like the following: "As physicians, (or, as physicians-in-training,) we have discovered that many of our patients have spiritual or religious beliefs that have a bearing on their perceptions of illness and their preferred modes of treatment. If you are comfortable discussing this with me, I would like to hear from you of any beliefs or practices that you would want me to know about as your care giver." If the patient responds affirmatively, follow-up questions can be used to elicit additional information. If the patient says "no" or "none" it is a clear signal to move on to the next topic, although it is often productive to ask before leaving this topic if other family members have spiritual beliefs or practices in order to better understand the family context and anticipate concerns of the immediate family. From years as a clinical tutor, I have observed students returning to my office to de-brief a recent patient interview with a sense of excitement and gratification in discovering that this line of questioning opened a discussion that disclosed the patient's faith in God as a major comforting factor in the face of a life-threatening illness. One patient-family described gratitude for their church community who brought meals to their home in a period when one parent was at work and the other was at the hospital with a sick child, leaving no one to cook for the other siblings. Others spoke of a visit from a priest, a rabbi, or a minister during their hospitalization as a major source of comfort and reassurance. One patient, self-described as a "non-church-goer," described his initial surprise at a visit from the hospital chaplain which turned into gratitude as he found in the chaplain a skilled listener with a deep sense of caring to whom he could pour out his feelings about being sick, away from home, separated from his family, frightened by the prospect of invasive diagnostic procedures and the possibility of a painful treatment regimen. An older woman dying of metastatic cancer, whose adult children and grandchildren lived on the east coast, commented that although she was separated from her family as death drew near, she was sustained by the belief that "I will never be separated from the love of God, even in the moment of death." An elderly patient with advanced COPD and heart disease explained that he would not seek aggressive treatment, but as a man of faith, was at peace with his imminent death. Most patients will be grateful for an inquiry about their spirituality and for the physician's respect for their spiritual as well as their physical being. Some find it helpful to have a clear approach or structure in mind when opening a discussion on spirituality with a patient or taking a spiritual history. A group at Brown University School of Medicine has developed a teaching tool to help begin the process of incorporating a spiritual assessment into the patient interview which they call the HOPE questions: H: Sources of hope, meaning, comfort, strength, peace, love and connection. O: Organized religion P: Personal spirituality and practices E: Effects on medical care and end-of-life issues (Anadarajah & Hight, 2001) So, for example, one might open this line of inquiry by stating that many patients have religious or spiritual beliefs that affect their choices regarding medical care, and ask, "I'm wondering, (H) Where do you find comfort or hope in this time of illness? When things are tough, what keeps you going? (O) Does organized religion have a place in your life, or in your family's life? (P) Are there spiritual practices or beliefs that are important to you personally? (E) Are there ways that your personal beliefs affect your health care choices or might provide guidance as we discuss decisions about your care near the end of your life?" (ibid) One possible advantage of the HOPE questions is the fact that they begin with open-ended questions related to one's support systems and are inclusive of those who may be nontraditional in their spirituality (ibid). As the interviewer's skills develop it will become easier and more natural to recognize both verbal and nonverbal cues of the patient and to follow up appropriately.

What rules guide rationing decisions?

Often scarcity can be alleviated by improved efficiency or expanded investment. However, if these practical solutions cannot solve the problem, a "rationing" decision must be made. Rationing means the distribution of any needed thing or procedure that is in short supply to those who need it in accord with a set of rules that assure fair distribution. The reasons for shortage can be many. For example, there are many more patients with end stage cardiac disease or liver disease than there are cadaver organs available; expensive equipment may be lacking in a particular region; tertiary care hospital beds may be limited; a particular medication may be extremely costly; few personnel might be trained for a certain technical procedure, insurance coverage is unavailable or of prohibitive cost. Every physician rations his or her own time available to provide medical services. For the most part, this personal rationing is done by rules of common sense: I will take only as many patients as I can care for competently; I will assure that my attendance is sufficient to guarantee high quality medical care to my patients, etc. For other kinds of rationing, for instance rationing of ICU beds, these rules of thumb are not enough. More articulate principles are required. In one highly publicized instance of resource allocation, the Seattle Artificial Kidney Center appointed a committee to decide who would receive dialysis treatments, in 1962 a rare and expensive resource. "Likelihood of medical benefit" was the first criteria used to determine eligibility. Even so, many more patients required dialysis than there were machines available. The committee turned to "social worth" criteria that is, they tried to weigh the anticipated contributions the patients would make to society were their lives saved. This sort of evaluation proved very difficult and troubling, since it led to highly discriminatory judgments, such as "Popular" people over unpopular, school graduates over the uneducated, devout over unreligious, etc. Many bioethicists argued that a lottery or a "first-come, first-served" criteria would have been more equitable and ethically justifiable. One of the most serious medical shortages, organs for transplantation, has been organized into a national system with criteria that strive for fairness. The criteria attempt to match available organs with recipients on presumed "objective" grounds, such as tissue type, body size, time on waiting list, seriousness of need. However, even in this system, it is obvious that such a criterion as "serious need" can be used in a manipulative way. Still, this system is preferable to the subjective use of criteria of social worth and status that would unfairly skew the distribution of organs.

Is professionalism compatible with the restrictions sometimes placed on physician's judgments in managed care?

One of the principal attributes of professionalism is independent judgment about technical matters relevant to the expertise of the profession. The purpose of this independent judgment is to assure that general technical knowledge is appropriately applied to particular cases. Today, many physicians work in managed care situations that require them to abide by policies and rules regarding forms of treatment, time spent with patients, use of pharmaceuticals, etc. In principle, such restrictions should be designed to enhance and improve professional judgment, not limit it. For example, requiring consultation is ethically obligatory in doubtful clinical situations; penalizing consultation for financial reasons would be ethically wrong. Also, requiring physicians to adhere to practice guidelines and to consult outcome studies may improve professional judgment; requiring blind adherence to those guidelines may be a barrier to the exercise of professional judgment. The presence of rules, policies and guidelines in managed care settings requires the physicians who work in these settings to make such judgments and to express their reasoned criticism of any that force the physician to violate the principles of professionalism.

What happens when physicians and patients disagree?

One third to one half of patients will fail to follow a physician's treatment recommendations. Labeling such patients "noncompliant" implicitly supports an attitude of paternalism, in which the physician knows best (see: Difficult Patient Encounters). Patients filter physician instructions through their existing belief system and competing demands; they decide whether the recommended actions are possible or desirable in the context of their everyday lives. Compliance can be improved by using shared decision making. For example, physicians can say, "I know it will be hard to stay in bed for the remainder of your pregnancy. Let's talk about what problems it will create and try to solve them together." Or, "I can give you a medication to help with your symptoms, but I also suspect the symptoms will go away if you wait a little longer. Would you prefer to try the medication, or to wait?" Or, "I understand that you are not ready to consider counseling yet. Would you be willing to take this information and find out when the next support group meets?" Or, "Sometimes it's difficult to take medications, even though you know they are important. What will make it easier for you to take this medication?" Competent patients have a right to refuse medical intervention. Dilemmas may arise when a patient refuses medical intervention but does not withdraw from the role of being a patient. For instance, an intrapartum patient, with a complete placenta previa, who refuses to undergo a cesarean delivery, often does not present the option for the physician to withdraw from participation in her care (see: Maternal-Fetal Conflict). In most cases, choices of competent patients must be respected when the patient cannot be persuaded to change them.

Can parents refuse to provide their children with necessary medical treatment on the basis of their beliefs?

Parents have legal and moral authority to make health care decisions for their children, as long as those decisions do not pose a significant risk of serious harm to the child's health. Parents should not be permitted to deny their children medical care when that medical care is likely to prevent substantial harm or suffering. If necessary, the physician may need to pursue a court order or seek the involvement of child protective services in order to provide treatment against the wishes of the parents. Nevertheless, the physician must always take care to show respect for the family's beliefs and a willingness to discuss reasonable alternatives with the family.

Who has the authority to make decisions for children?

Parents have the responsibility and authority to make medical decisions on behalf of their children. This includes the right to refuse or discontinue treatments, even those that may be life-sustaining. However, parental decision-making should be guided by the best interests of the child. Decisions that are clearly not in a child's best interest can and should be challenged.

What kinds of treatment can parents choose not to provide to their children?

Parents have the right to refuse medical treatments when doing so does not place the child at significant risk of substantial harm or suffering. For example, parents have the right to refuse routine immunizations for their children on religious or cultural grounds.

What is physician aid-in-dying?

Physician aid-in-dying (PAD) refers to a practice in which a physician provides a competent, terminally ill patient with a prescription for a lethal dose of medication, upon the patient's request, which the patient intends to use to end his or her own life. (For related discussion, see also End of Life Issues.)

Do physicians have an ethical duty to disclose information about medical mistakes to their patients?

Physicians have an obligation to be truthful with their patients. That duty includes situations in which a patient suffers serious consequences because of a physician's mistake or erroneous judgment. The fiduciary nature of the relationship between a physician and patient requires that a physician deal honestly with his patient and act in her best interest.

How does public health balance the interests of individuals with the interests of the broader public?

Public health activities routinely bring some of these moral considerations into conflict. One major area of discourse and debate concerns the power of public health as an agent of the state to restrict individual choice in efforts to prevent disease and promote health. Many public health activities try to influence individual actions, though they may do so in more or less restrictive ways. Public health policy may eliminate choice altogether through, for example, compulsory quarantine of patients with infectious disease; restrict choice by, for example, banning smoking in public places or fluoridating public water supplies; guide choice through disincentives (e.g., taxes on health-harming goods, such as sugary beverages) and incentives (e.g., tax breaks on health-promoting goods); or inform choice through, for example, food labeling or media campaigns (Nuffield Bioethics Council, see chapter 3). The analysis of which of these actions are or are not ethically and politically justifiable is often informed by the harm principle, originally articulated by John Stuart Mill in his essay On Liberty. Mill argues "the only purpose for which power can be rightfully exercised over any member of a civilized community against his will, is to prevent harm to others. His own good, either physical or moral, is not a sufficient warrant" (Mill 1989). Mill qualifies the principle in several ways, for example, making exceptions for children and those cared for by others. But, in general the principle justifies state interference with individual liberty only when individual actions pose serious harm to others. The principle thus presupposes that behavior affecting only the self ("self-regarding behavior") can be clearly distinguished from that affecting others ("other-regarding behavior"), which may be difficult in a public health context. Take, for example, the obesity epidemic in the United States, which has the highest rate among Organisation for Economic Cooperation and Development countries (OECD 2012). It has been argued that individual choices that contribute to overweight and obesity constitute an economic harm to others who share in the costs of the chooser's health care (Pearson, Lieber 2009). Or, consider the dietary choices of an adult who is also a parent of small children. Parents' meal planning, purchase of groceries, cooking, and direct feeding of a very young child has other-regarding, sometimes deleterious, consequences for children (Blacksher 2008). Such examples suggest that apparently self-regarding choices related to diet might be other-regarding in significant respects. Nonetheless, the distinction between self- and other-regarding behavior has been invoked persuasively in clear cases, for example, to justify the quarantine of individuals with infectious disease that can easily be transmitted or bans on smoking in public spaces to protect others from the harmful effects of second-hand smoke.

What is known about the efficacy of CAM?

Research to evaluate the efficacy of CAM is ongoing. Federal funding for CAM research has increased over the past two decades, but still falls short of allocations made to other areas of inquiry. To date, clinical trials have demonstrated some efficacy of CAM therapies in the amelioration of back pain, upper respiratory infections, and diabetes. Medicinal mushrooms are being studied in cancer patients with promising results. In vitro studies provide exciting data on the induction of apoptosis in cancer cells by botanical compounds, modulation of immune function by herbal products or mushrooms, and the mitigation of cellular oxidative stress by botanical extracts. The Cochrane Reviews provide useful summaries of extant CAM clinical trials and all entries articulate the need for more research in the area. There are many methodological issues that make the design of CAM research studies challenging. Following are a few of many examples. Botanical medicine derives from folk traditions in which plants, either the whole plant or isolated parts of it, are harvested and prepared according to learned wisdom. Botanical extracts are complex mixtures of molecular constituents that are difficult to standardize and characterize, an issue further complicated by the timing of harvest, method of extraction, and growing conditions. The western science paradigm of pharmaceutical development and testing (a single molecule delivered in pure form at relatively high concentrations) does not fit the realities of botanical therapies. Whole system CAM practices treat the individual patient in a very individualized way, generally using multiple interventions simultaneously. Such an approach does not lend itself to validation by the RCT approach. More appropriate to CAM clinical trials may be the "n-of-1" design. Using this approach, an individual serves as their own control, and the intervention or placebo is each administered multiple times, in random order, with outcome measures taken during each phase. Informative though they can be, these studies are laborious, expensive, and time-consuming for the subject, thus increasing attrition rates. In designing an RCT, it is often difficult to define an appropriate control group. In acupuncture studies, for example, debate continues about what constitutes sham needling: is it pressure without penetration, or is it needling at a point known to be ineffective, or is it a time and attention control? In botanical medicine studies on compounds with a strong or recognizable taste (for example, garlic), how does one come up with true placebo? NCCAM addresses some of these concerns in its current statement of funding priorities , asserting the need for pharmacokinetic studies of biologically based interventions, the optimization of dosing regimens (across modalities) and control groups, and the refinement and validation of outcome measures. Additionally, studies on safety and efficacy and mechanistic studies are being encouraged.

Why is it important to respect what appear to me to be idiosyncratic beliefs?

Respecting the beliefs and values of your patient is an important part of establishing an effective therapeutic relationship. Failure to take those beliefs seriously can undermine the patient's ability to trust you as her physician. It may also encourage persons with non-mainstream cultural or religious beliefs to avoid seeking medical care when they need it.

Do survivors and families think that aggressive care for very low birth weight infants is good?

Self-assessment research of a large group of adolescents, former extra-low birth weight infants, demonstrates that their view of their quality of life was "quite satisfactory". Indeed, when compared to a comparable age group of normal controls, there were few differences between the two groups in their perception of their quality of life. Similar studies have found comparable quality of life assessment with adolescents with and without cerebral palsy, a known complication of prematurity. Physicians judge former premature infants' quality of life lower than former premature infants and their families judge themselves.

A 32 year old woman was admitted to the Trauma Intensive Care Unit following a motor vehicle accident; she had multiple injuries and fractures, with several complications which continued to develop over the first couple of weeks. The patient rapidly developed Adult Respiratory Distress Syndrome, was on a ventilator, and was continuously sedated. Shortly after the patient's admission, her parents were contacted and remained vigilant at her bedside. The parents reported that the patient was one month away from having her divorce finalized. The patient's husband was reportedly physically and emotionally abusive to her throughout their five years of marriage. The parents had not notified this man of the patient's hospitalization, and reported that a visit by him would be distressing to the patient if she were aware of it. The patient's soon to be ex-husband is her legal next of kin.

Some key clinical ethics and legal issues raised by Case 2 are informed consent and surrogate decision-making. While the details of each case will determine the advice provided, this case raises a number of issues. Specific clinical ethics and legal issues: As mentioned above,23 implied consent is permitted by law for provision of "emergency" medical treatment. However the relevant state's law does not define the term "emergency."24 Each institution should have a policy that defines emergency in accordance with state law and lays out institutional documentation requirements so that providers are guided in their decision-making.25 Thus, if a medical emergency exists and implied consent is relied on by the health care providers, it should be documented in the patient's medical record in accordance with legal and institutional standards. The patient may have provided her own consent to treatment either at the time of her admission or earlier in her hospitalization. At that time, she may have expressed her ongoing wishes for care. The patient's own previous statements/consent may therefore be the basis for continued consent for her ongoing care. If there is a need for informed consent for a new treatment decision on behalf of the patient, the patient's previously expressed wishes may still be relevant to her legally authorized surrogate decision-maker and her treatment plan. If the patient already filed for divorce, it is likely that there is a temporary court order in effect and this order may affirmatively remove the patient's estranged husband from making medical decisions for her. Also, divorce paperwork may have mutual restraining orders which prevent both spouses from contacting each other. The patient's parents should be asked to provide the name of her divorce attorney. Hospital staff may contact the patient's attorney to request information and to obtain copies of the relevant legal papers, which can then be placed in the legal section of the patient's medical record. Obtaining information is not a violation of patient confidentiality. It is also permissible for an attorney to provide information that is contained in public records, such as documents filed with a court. With the husband thus removed as her surrogate decision-maker, it appears the patient's parents would become the highest level class of surrogate decision-maker and could provide informed consent for her care if the patient is unable to do so. Even if the patient's husband remains as her legal surrogate decision-maker, his decisions on the patient's behalf are constrained by clinical ethics and legal standards. First, a surrogate is legally required to provide "substituted judgment" on behalf of the patient. This means that the surrogate must act in accordance with the patient's wishes. If substituted judgment isn't possible (i.e., unknown what the patient would want under the current medical circumstances), then the law requires the surrogate to act in the patient's "best interest." Since the medical team has significant input about what would medically be in the patient's best interest, a decision by a surrogate which doesn't adhere to this standard should not be automatically followed and may need to be reviewed by a clinical ethics consultant or committee, risk management, or legal counsel. The patient's husband may be willing to waive his role as surrogate decision-maker. If this occurs, then he would agree to remove himself from the list of potential surrogate decision-makers and the next highest level surrogate decision-maker(s) would be contacted as necessary to provide informed consent for the patient. Another option may be for the patient's parents to file in court to become the patient's legal guardians for health care decision-making

What if I'm not sure if the patient is competent?

Sometimes the patient is awake, alert, and conversant, but their decisions seem questionable or irrational. First, it is important to distinguish an irrational decision from simple disagreement. If you feel strongly that a certain course of action is "what's best" for the patient, it can seem irrational for them to disagree. In these situations, it is critical to talk with the patient and find out why they disagree. Patients are presumed to be "competent" to make a treatment decisions. Often it's better to say they have "decision making capacity" to avoid confusion with legal determinations of competence. In the courts, someone's competence is evaluated in a formal, standardized way. These court decisions do not necessarily imply anything about capacity for making treatment decisions. For example, an elderly grandfather may be found incompetent to manage a large estate, but may still have intact capacity to make treatment decisions. In general, the capacity to make treatment decisions, including to withhold or withdraw treatment, is considered intact if the patient: understands the clinical information presented appreciates his/her situation, including consequences with treatment refusal is able to display reason in deliberating about their choices is able to clearly communicate their choice. If the patient does not meet these criteria, then their decision to refuse treatment should be questioned, and handled in much the same way as discussed for the clearly incompetent patient. When in doubt, an ethics consultation may prove helpful.

B.R. is a term female infant from an unexpected pregnancy. She has Down syndrome (Trisomy 21) and also has a complex cardiac lesion that will require at least two major surgical procedures during early infancy for her to have a chance to survive beyond childhood. B.R.'s parents, ages 44 and 45, have three other children, all in college. They have considerable ambivalence as to what to do: continue to pursue potentially beneficial though burdensome and costly treatments, or forego such treatments in favor of a more conservative approach. What are the issues involved?

That the child has Down syndrome should not be a factor in the decision-making process. Moreover, it is also not appropriate to allow financial issues to play a major role. The parents, who are the decision-makers, should be apprised of the medical facts (types of surgical interventions required, chances for success). They should also be given a good understanding of the amount of suffering the child will experience during aggressive intervention efforts. They should then come to a decision based on the child's best interests.

How can we approach spirituality in medicine with physicians-in-training?

The UW School of Medicine was an early leader among medical schools in addressing the topic of patient-spirituality. In an elective course, originating in Spring, 1998, "Spirituality in Health Care," the range of topics goes beyond simply teaching spiritual history taking. Students are encouraged to practice self-care in order to remain healthy as providers for others, and to give intentional consideration to their deep values and their own spirituality as components of their spiritual well-being. The purpose of this interdisciplinary course is to provide an opportunity for interactive learning about relationships between spirituality, ethics and health care. Some of the goals of the class are as follows: To heighten student awareness of ways in which their own faith system provides resources for encounters with illness, suffering and death. To foster student understanding, respect and appreciation for the individuality and diversity of patients' beliefs, values, spirituality and culture regarding illness, its meaning, cause, treatment, and outcome. To strengthen students in their commitment to relationship-centered medicine that emphasizes care of the suffering person rather than attention simply to the pathophysiology of disease, and recognizes the physician as a dynamic component of that relationship. To facilitate students in recognizing the role of the hospital chaplain and the patient's clergy as partners in the health care team in providing care for the patient. To encourage students in developing and maintaining a program of physical, emotional and spiritual self-care, which includes attention to the purpose and meaning of their lives and work. (McCormick, 2011) Until recently, there were all too few medical schools that offered formal courses in spirituality in medicine for medical students and residents. This situation is changing. When the University of Washington School of Medicine created the elective, "Spirituality in Medicine" (1998) there were few other such courses available in medical schools. In 2004, JAMA's curricular survey showed that: "in 1994, only 17 of the 126 accredited US medical schools offered courses on spirituality in medicine. By 1998, this number had increased to 39, and by 2004, to 84 schools" (Fortin & Barnett, 2004).

An otherwise healthy 54-year-old man presents for radical retropubic prostatectomy, and expresses interest to his anesthesiologist in having postoperative epidural narcotic pain management. The anesthesiologist believes it provides superior pain control, but is informed by the surgeon that the patient "is not to have an epidural." Is the anesthesiologist obliged to "take an order" from the surgeon? Should the anesthesiologist provide the anesthetic he feels is best, regardless of the surgeon's input?

The answer to both questions is no. Anesthesiologists have special knowledge and training which are not shared by the surgeon with regard to the safe administration of anesthesia. They also have direct obligations to the patient to provide safe medical care which is as far as possible in keeping with the patient's wishes. When medical issues of safety, or specific patient goals are in conflict with the surgeon's desires, the anesthesiologist is first ethically obliged to provide the best care to the patient. But the anesthesiologist would be incorrect to proceed at this point without some discussion with the surgeon, for at least two reasons. First, ignoring the surgeon's communication is disrespectful. Second, the surgeon may have valuable information to impart, such as "my patients achieve very good pain control with intravenous and oral medication, and end up being discharged two days sooner than epidural patients, because they do not require prolonged urinary catheterization from epidural-associated urinary retention." This dialogue between team members can result in improved team relations, and better care for the patient.

Who is the decision-maker regarding the nature of medical care administered to a newborn infant?

The biological parents (or parent) have authority regarding the decisions for their child (known as parental authority). This is a universally granted right regardless of the parents' age or other contextual features, unless the parents are declared not competent or otherwise unfit to serve as the child's proxy. There are some limits to such authority. For example, when the discussion occurs in the context of delivery room management of extreme prematurity, usually parents wishes (whatever they may be) are followed at 23 and for the most part 24 weeks gestation. Whereas at 22 weeks most neonatologists are reluctant to aggressively resuscitate and at 25 weeks reluctant to withhold such care. (See "Resuscitation in the Gray Zone of Viability: Determining Physicians Preferences and Predicting Infant Outcomes." Pediatrics. 2007;120;519-526. (See also Parental Decision Making).

What is the accepted legal and ethical basis for decision-making regarding the nature of medical care in the newborn infant?

The child's best interest is legally and ethically primary and should be weighed over the family's well-being or societal concerns (using the Best Interest Standard). However, there is considerable discussion in the literature about the utility of applying this standard to neonates based on their present or future best interest. Also, much discussion is placed on the need to include the family's interests when making life and death decisions regarding severely compromised infants. In the interest of justice, societal concerns about excessive cost for aggressive care should be addressed at the policy level, rather than on an individual or case-by-case basis.

An 18-month-old child presents to the clinic with a runny nose. Since she is otherwise well, the immunizations due at 18 months are administered. After she and her mother leave the clinic, you realize that the patient was in the clinic the week before and had also received immunizations then. Should you tell the parents about your mistake?

The error is a trivial one. Aside from the discomfort of the unnecessary immunization, no harm has resulted. Nonetheless, an open and honest approach to errors is most appropriate. While the parents may be angry initially about the unnecessary injection, they will appreciate your candor. On the other hand, should they discover the error and believe you have been dishonest, their loss of trust will be significant.

Neonatal ICU issues

The evolution of aggressive treatment of the newborn infant over the past 45 years has been associated with a dramatic reduction in mortality for virtually all major disease categories in the newborn period. Such care is costly; often causes suffering; and sometimes can result in considerable long-term morbidity. The following quotes provide a perspective: "Neonatal intensive care is responsible for the survival of a significant number of infants who formerly would not have survived. This increased survival has been accomplished with an acceptable level of burden and without substantially increasing the population of handicapped children." - A proponent "Neonatal intensive is a good example of medicine out of control. There is inappropriate use of technology by health care professional who are out of touch with patients and their families. The benefits of increa

What are the ethical obligations of physicians when a health care provider judges an intervention is futile?

The goal of medicine is to help the sick. Physicians have no obligation to offer treatments that do not benefit patients. Futile interventions may increase a patient's pain and discomfort in the final days and weeks of life; give patients and family false hope; delay palliative and comfort care; and expend finite medical resources. However, determining which interventions are beneficial to a patient can be difficult, since the patient or surrogate might see an intervention as beneficial while the physician does not. Physicians should follow professional standards, and should consider empirical studies and their own clinical experience when making futility judgments. They should also show sensitivity to patients and families in carrying out decisions to withhold or withdraw futile interventions. Although the ethical requirement to respect patient autonomy entitles a patient to choose from among medically acceptable treatment options (or to reject all options), it does not entitle patients to receive whatever treatments they ask for. Instead, the obligations of physicians are limited to offering treatments that are consistent with professional standards of care and that confer benefit to the patient.

What is the difference between a profession and a business?

The line between a business and a profession is not entirely clear, since professionals may engage in business and make a living by it. However, one crucial difference distinguishes them: professionals have a fiduciary duty toward those they serve. This means that professionals have a particularly stringent duty to assure that their decisions and actions serve the welfare of their patients or clients, even at some cost to themselves. Professions have codes of ethics which specify the obligations arising from this fiduciary duty. Ethical problems often occur when there appears to be a conflict between these obligations or between fiduciary duties and personal goals.

Mr. S is a 73-year-old man, with a history of severe coronary artery disease, peripheral vascular disease, and stroke. He suffers from right hemiplegia and mild expressive aphasia. He is awake and alert, and presents for right below the knee amputation (BKA) for vascular insufficiency. His chart carries a DNR order. In the holding area prior to surgery, the anesthesiologist discusses the DNR order with Mr. S, who appears depressed. Mr. S states unequivocally, that he does not wish CPR in the OR, regardless of its cause or positive prognosis. He tells his anesthesiologist that he is willing to go "so far, and no more." The patient agrees to subarachnoid anesthesia (spinal block) and sedation. He is not intubated. After about 20 minutes, the patient complains of weakness in his arms, and difficulty breathing. Within 3 minutes, his blood pressure and heart rate fall, and he abruptly arrests. Should the patient be intubated? Should CPR be commenced?

The probable cause of Mr. S's arrest is a cephalad migration of local anesthetic in the subarachnoid space, leading to a "high spinal block." As a result of migration of the local anesthetic from the lumbar segments to high thoracic or even cervical segments, weakness or paralysis of respiratory muscles, including intercostal muscles and diaphragmatic muscles can result. The effect of local anesthetic on segments contributing to the cardiac accelerator fibers can cause bradycardia, and even cardiac arrest. With cardiopulmonary support, prognosis for total recovery from this event is excellent, with only rare cases of central nervous system damage or death reported. CPR would not be futile from a medical standpoint. Intubation and institution of mechanical ventilation will not alone restore Mr. S's circulation, and these measures alone will be useless. Medications to treat blood pressure and bradycardia will require at least temporary artificial circulation. From the standpoint of medical futility, intubation and mechanical ventilation would be senseless unless accompanied by full CPR, if even briefly. It is hard to argue ethically for the institution of CPR in this patient, who while neurologically impaired, appeared to have full capacity to understand and make decisions regarding his own medical care. Despite preoperative discussion which included information about the good prognosis from CPR in the OR, the patient stated clearly his wishes to not be resuscitated if an arrest occurs. Instituting CPR in this patient because the cause of arrest is anesthetic-related, would be like justifying transfusion in a Jehovah's Witness against their will because the surgery was the cause of life-threatening hemorrhage, yet adhering to their wishes if hemorrhage was due to non-surgical injuries.

A mother brings her 18-month-old daughter to your office for a routine physical examination. The child has had no immunizations. Her mother says that they believe that vaccines weaken the immune system and have heard that vaccination can cause autism. What is your role in this situation? Can parents refuse to immunize their children?

The risk faced by unimmunized individuals is relatively low, and the mother's refusal to immunize does not pose a significant likelihood of serious harm to her child. The physician should be sure that the child's mother understands the risks of remaining unimmunized and attempt to correct any misconceptions about the degree of risk associated with getting immunized. If the mother persists in her request, the physician should respect her wishes.

What happens when an older child disagrees with her parents about a medical treatment?

The wishes of competent older children regarding their medical care should be taken seriously. If the medical caretaker judges a child competent to make the medical decision in question, she should first attempt to resolve the issue through further discussion. If that fails, the medical caretaker should assure that the child's voice has been heard and advocate for the child. In intractable cases, an ethics consultation or judicial hearing should be pursued.

When is it justified for me to withhold the truth from a patient?

There are two main situations in which it is justified to withhold the truth from a patient. As noted above, if the physicians has compelling evidence that disclosure will cause real and predictable harm, truthful disclosure may be withheld. Examples might include disclosure that would make a depressed patient actively suicidal. This judgment, often referred to as the "therapeutic privilege," is important but also subject to abuse. Hence it is important to invoke this only in those instances when the harm seems very likely, not merely hypothetical. The second circumstance is if the patient him- or herself states an informed preference not to be told the truth. Some patients might ask that the physician instead consult family members, for instance. In these cases, it is critical that the patient give thought to the implications of abdicating their role in decision making. If they chose to make an informed decision not to be informed, however, this preference should be respected.

What is public health?

There is no settled account or definition of public health. It has been defined numerous ways, reflecting the diverse contexts, activities, and interventions of public health. A sampling of definitions, from general to more specific, include: What we, as a society, do collectively to assure the conditions in which people can be healthy (Institute of Medicine 1988). The science and art of preventing disease, prolonging life and promoting health through organized efforts of society (Faculty of Public Health of the Royal Colleges of the United Kingdom, as cited in Nuffield Bioethics Council Report 2007, p. 6). A movement dedicated to the equitable improvement of health and wellbeing of communities with their full participation. (World Federation of Public Health Associations as cited in Nuffield Bioethics Council Report 2007, p. 6). Public health is the science of protecting and improving the health of communities through education, promotion of healthy lifestyles, and research for disease and injury prevention (Association of Schools of Public Health USA). The key functions of public health agencies are assessing community health needs and marshalling the resources for responding to them, developing health policy in response to specific community and national health needs, and assuring that conditions contribute to good health, including high quality medical services, safe water supplies, good nutrition, unpolluted atmospheres and environments that offer opportunities for exercise and recreation are available to individuals (World Medical Association 1995). [T]he promotion of health and the prevention of disease and disability; the collection and use of epidemiology data, population surveillance, and other forms of empirical quantitative assessment; a recognition of the multidimensional nature of the determinants of health; and a focus on the complex interactions of many factors-biological, behavioral, social and environmental-in developing effective interventions (Childress et al. 2002).

Is it justifiable to withhold or withdraw food or fluids?

This question underscores the importance of clarifying the goals of medical treatment. Any medical intervention can be withheld or withdrawn, including nutrition and IV fluids. At all times, patients must be given basic humane, compassionate care. They should be given a comfortable bed, human contact, warmth, and be kept as free from pain and suffering as possible. While some believe that food and fluids are part of the bare minimum of humane treatment, both are still considered medical treatments. Several court cases have established that it is justifiable to withhold or withdraw food and fluids.

A 3-month-old has been admitted to the hospital with a newly diagnosed ventricular septal defect. She is in early congestive heart failure and digoxin is indicated. After discussing the proper dose with the attending physician, you write an order for the drug. Thirty minutes later the baby vomits and then has a cardiac arrest and dies. You discover that in writing the digoxin order you misplaced the decimal point and the child got 10 times too much digoxin. What is your duty here? Will you get sued if you tell the truth?

This unfortunate event represents a serious error with profound implications for the patient and family. You owe this family an honest explanation. They need to hear you say that you're sorry. Any attempt to hide the details of the event would be dishonest, disrespectful, and wrong. Though a lawsuit may follow, these parents are less likely to litigate if you deal with them honestly and take responsibility for the error.

Does the patients have to be terminally ill to refuse treatment?

Though in most cases of withholding or withdrawing treatment the patient has a serious illness with limited life expectancy, the patient does not have to be "terminally ill" in order for treatment withdrawal or withholding to be justifiable. Most states, including Washington State, have laws that guarantee the right to refuse treatment to terminally ill patients, usually defined as those having less than 6 months to live. These laws do not forbid other patients from exercising the same right. Many court cases have affirmed the right of competent patient to refuse medical treatments.

People have rights. Does a fetus have rights?

Throughout the world, the legal status of fetuses is generally subordinated to that of pregnant women. In most countries, the legal status of the fetus is bolstered as gestation progresses. Viability signifies a change in its legal status. Still, although the human fetus has the potential for personhood, this does not imply that it is a person or that it has rights. So long as the fetus is attached to the pregnant woman, her body maintains its life, and bars access to it. US courts have ruled that a child has a legal right to begin life with a sound body and mind. Such a right may create a legal duty, on the part of a pregnant woman, to protect the health of her fetus. Failure to fulfill that duty could subject her to charges of fetal abuse, or render her liable for consequent damage to her child. Pregnant women's refusals of hospitalization, intrauterine transfusion, or surgical delivery have been legally challenged on the grounds of an obligation to the fetus.

Can I make allocation decisions based on judgments about "quality of life"?

Under conditions of scarcity, the question may arise whether a patient's quality of life seems so poor that use of extensive medical intervention appears unwarranted. When this question is raised, it is important to ask a few questions. First, who is making this quality of life judgment, the care team, the patient, or the patient's family? Several studies have shown that physicians often rate the patient's quality of life much lower than the patient himself does. If the patient is able to communicate, you should engage her in a discussion about her own assessment of her condition. When considering quality of life, you should also ask: What criteria are being used to make the judgment that the quality of life is unacceptable? These criteria are often unspoken and can be influenced by bias or prejudice. A dialogue between care givers and the patient can surface some underlying concerns that may be addressed in other ways. For example, residents on a medical floor in an urban public hospital may get discouraged with the return visits of a few chronically ill alcoholic patients and suggest that money is being wasted that could be used for prenatal care or other medically beneficial projects. While the residents' frustration is understandable, it would be helpful to consider other ways they might interrupt this vicious cycle of repeat admissions. How could this patient population be supported in ways that might improve health? Quality of life judgments based on prejudices against age, ethnicity, mental status, socioeconomic status, or sexual orientation generally are not relevant to considerations of diagnosis and treatment. Furthermore, they should not be used, explicitly or implicitly, as the basis for rationing medical services.

What other kinds of disclosures are inappropriate?

Unintended disclosures may occur in a variety of ways. For example, when pressed for time, providers may be tempted to discuss a patient in the elevator or other public place, but maintaining privacy may not be possible in these circumstances. Similarly, extra copies of handouts from teaching conferences that contain identifiable patient information should be removed at the conclusion of the session in order to protect patient privacy. And identifiable patient information should either be encrypted or should not be removed from the security of the health care institution. The patient's right to privacy is violated when lapses of this kind occur.

What if parents are unavailable and a child needs medical treatment?

When parents are not available to make decisions about a child's treatment, medical caretakers may provide treatment necessary to prevent harm to the child's health. In general, a child can be treated or transported without parental permission if the child has an emergency condition that places his or her life or health in danger, the legal guardian is unavailable or unable to provide permission for treatment or transport, and treatment or transport cannot be delayed without further endangering the child. Providers should administer only those treatments necessary to prevent harm to the child until parental permission can be obtained. Examining a child who presents to medical attention is always appropriate in order to establish whether a threat of life or health exists.

What role should my personal beliefs play in the physician-patient relationship?

Whether you are religious, or nonreligious, your beliefs may affect the physician-patient relationship. Care must be taken that the nonreligious physician does not underestimate the importance of the patient's belief system. Care must be taken that the religious physician who believes differently than the patient, does not impose his or her beliefs onto the patient at this vulnerable time. In both cases, the principle of respect for the patient should transcend the ideology of the physician. Our first concern is to listen to the patient. Physicians are autonomous agents who are free to hold their own beliefs and to follow their consciences. They may be atheists, agnostics, or believers. It is clear that religious beliefs are important to the lives of many physicians. Medicine is a secular vocation for some, while some physicians attest to a sense of being "called" by God to the profession of medicine. For example, the opening line from the Oath of Maimonides, a scholar of Torah and a physician (1135-1204) incorporates this concept: "The eternal providence has appointed me to watch over the life and health of Thy creatures" (Internet Sourcebook Project, 2011). In a much earlier time in the history of the world, the priest and the medicine man were one and the same in most cultures, until the development of scientific medicine led to a division between the professions. After Descartes and the French Revolution it was said that the body belongs to the physician and the soul to the priest. In our current culture of medicine, some physicians wonder whether, when and how to express themselves to patients regarding their own faith. The general consensus is that physicians should take their cues from the patient, with care not to impose their own beliefs. In one study reported in the Southern Medical Journal in 1995, physicians from a variety of religious backgrounds reported they would be comfortable discussing their beliefs if asked about them by patients (Olive, 1995). The study shows that physicians with spiritual beliefs that are important to them integrate their beliefs into their interactions with patients in a variety of ways. Some devout physicians shared their beliefs with patients, discussed patients' beliefs, and prayed either with or for patients who requested such. These interactions were more likely in the face of a serious or life-threatening illness and religious discussions did not take place with the majority of their patients (ibid).

After having completed a study that involved the collection of tissue from the subjects, an investigator wishes to perform additional analysis of the archived tissue samples. This nature of this analysis was not explicitly stated in the original consent form. Should the investigator be required to obtain explicit consent for the new research?

While de-identified tissues and data are currently permitted for secondary research uses under the regulations the Common Rule and such practices are under sharp debate given the inherent identifiability of genetic information and the requirements of respect for persons. Institutional Review Boards have increasingly required that explicit consent be obtained, if practical, before archived tissue can used for research. Even if participants may be willing in general to have surplus tissue used for research purposes, they should still be asked for their consent when specific new uses for their samples are identified.

A 28-year-old male is admitted with bacterial endocarditis and needs a replacement of his prosthetic heart valve. After his first replacement, he continued to abuse intravenous drugs. The medical team feels it would be "futile" and a waste of medical resources to replace this heart valve yet again. Is the team's judgment appropriate in this case?

While it is likely that this patient will require additional counseling and support services to improve his health outcomes, replacing the heart value is not "futile" in this case (see the topic page on Futility for further discussion). It is also likely that the medical team is using biased criteria to judge "wasted" vs. "properly used" medical resources. Thoughtful discussion may provide an opportunity for the team to voice their frustration and think through a treatment plan that will maximally support this patient's recovery.

What if a family member asks how the patient is doing?

While there may be cases where the physician feels naturally inclined to share information, such as responding to an inquiring spouse, the requirements for making an exception to confidentiality may not be met. If there is not explicit permission from the patient to share information with family member, it is generally not ethically justifiable to do so. Except in cases where the spouse is at specific risk of harm directly related to the diagnosis, it remains the patient's (and sometimes local public health officers'), rather than the physician's, obligation to inform the spouse.

How do teams work together?

Working together as a team, professionals must balance responsibilities, values, knowledge, skills, and even goals about patient care, against their role as a team member in shared decision-making. Because many physicians, in particular, are accustomed to a practice environment in which decisions are "made" by the doctor, and "carried out" by other professionals, it is difficult sometimes for physicians to adjust to a team approach, in which majority opinion, deference to more expert opinion, unanimity, or consensus may be more appropriate methods of decision-making than autocratic choice. Further, physicians who maintain a hierarchical concept of medical care may face serious problems when disagreements arise with other physicians of equal "stature" on the medical team. Interdisciplinary conflicts are seen in all areas of medical practice, but the operating room environment is particularly rich in examples in which patient care involves interdisciplinary cooperation, conflict, and compromise.

How do I recognize this type of relationship?

Your feelings are the first clue. Be alert when you feel anger, resentment, fear, dread, or excessive anxiety about seeing a patient, when you worry that the patient will transgress a professional or personal boundary, when you want to avoid the patient, and/or are unable to feel empathy for him/her.

Who is in charge in the operating room? Isn't the surgeon "Captain of the Ship"?

You will certainly hear at some point in your medical training that the surgeon is "captain of the ship" in the operating room. While recent legal decisions have essentially "sunk" the concept, it is important to understand the ethical and legal terrain. The phrase "captain of the ship" was first used by the Pennsylvania Supreme Court in 1949 in McConnell vs. Williams. In that case, an intern at a charity hospital was responsible for blinding a newborn by improperly applying silver nitrate drops to her eyes. Laws in widespread application at the time provided many hospitals with "charitable immunity" from legal damages, and the parents of the newborn were unable to get money from the intern because he acted as a hospital employee. They therefore brought suit against the obstetrician. The Pennsylvania Supreme Court allowed a finding of negligence against the obstetrician, despite the fact that the obstetrician had had no direct part in the negligent act, specifically so that someone would pay money to the parents. In its decision, the court used an analogy from maritime law, in which a captain can be held liable for the action of all members of the crew of his ship. Since 1949, several key changes have taken place. Hospitals are no longer immune from liability in most jurisdictions, in part because hospitals generally carry insurance against the negligent acts of their employees. Courts also recognize that the scope and complexity of medical practice is such that no single provider generally has complete control over a patient's medical care. The diversity of medical practice and the different forms of training and certifications required for specialty practice testify that different professionals have different expertise and therefore diverse levels of responsibility for individual acts in patient care. In this aspect the law is fair: the greater the authority and expertise asserted in a given act, the greater an individual's legal responsibility becomes. In recent years, many state Supreme Courts have specifically thrown out the "captain of the ship" doctrine in disgust. Cases in which the captain of the ship doctrine has been specifically discarded include those in which plaintiffs have asserted that the surgeon was responsible for the acts of nurses, nurse anesthetists, anesthesiologists, radiologists, and radiology technologists, and in which plaintiffs asserted that the anesthesiologist was responsible for the acts of surgeons, nurses, and nurse anesthetists. Ironically, some recent law suits have been successfully pursued against surgeons for the actions of other operating room personnel, only because the surgeon himself asserted that he had, or should have had, complete control over everyone in the room at the time of the negligent act!

Can we ethically qualify a "right to health care"?

Several ethical theories have been elaborated to formulate criteria for fair and just distribution and to examine the arguments for a "right to health care." At present, little agreement exists on any of these issues. Ideally, all persons should have access to a "decent minimum" of health care necessary to sustain life, prevent illness, relieve distress and disability, so that, in the words of one bioethicist, "each person may enjoy his or her fair share of the normal opportunity range for individuals in his or her society." Debates over this issue have been lengthy and serious. Many policy proposals have been considered: some implemented and others rejected. However, in the systems of managed health care now so common in American medicine, the question of fair and just allocation of resources must be raised and the various policies and criteria for allocating resources must be reviewed for their fairness and equitability. Some specific examples of public policy in devising an allocation system concentrate on the criteria of efficiency and cost-effectiveness. The state of Oregon is unique in having such a system for its Medicaid patients: a long list of medical procedures, ranked in terms of their cost/benefit ratio, determines the reimbursement policy. Even with such a system, ethical criteria must also be considered: what is to be done if life-saving and life-sustaining interventions rank low on cost-effectiveness? Is it ethical to omit the rescue of a person from death because their rescue by, say, bone marrow transplantation is less cost-effective than some preventive measures? How is cost-effectiveness to be applied to persons with shorter natural life expectancy, such as the elderly? These questions are not easily answered but they must be consistently raised whenever allocation systems are proposed. Some forms of allocation are obviously unethical in any society that values justice. For example, making the ability to pay the only way of obtaining medical care or distributing medical resources to the friends or political colleagues of those in power. Many other problems are less obviously unethical but still need to be evaluated and debated.

A 65-year-old man comes to his physicians with complaints of abdominal pain that is persistent but not extreme. Workup reveals that he has metastatic cancer of the pancreas. The man has just retired from a busy professional career, and he and his wife are about to leave on a round-the-world cruise that they've been planning for over a year. Should you tell him his diagnosis?

Several factors tempt one to withhold the diagnosis, and these should be recognized. One would be the concern that the patient would suffer psychological harm that would interfere with his planned trip. There is little empirical evidence that this occurs, and lacking some compelling reason to think it would occur with this man, it is insufficient grounds to withhold information. To the contrary, sensitive disclosure would allow the patient and his wife to decide if the trip is still important to them, versus seeing their grandchildren, for instance, and would spare the patient the inconvenience of suffering advancing symptoms while traveling, perhaps necessitating emergency care in a foreign locale. Finally, physicians should not confuse discomfort at giving bad news with justification for withholding the truth. In this case, the man should be told his diagnosis, prognosis, and treatment options.

During a visit to her family physician, a 35-year-old woman discloses that she suffers from anorexia nervosa. She complains of fatigue, dizziness, depression, headaches, irregular menses, and environmental allergies. Each day, she uses 15 to 60 laxatives, exercises for several hours, and eats a salad or half a sandwich. At 5'2", she weighs 88 pounds. She demonstrates a good understanding of the diagnosis and the recommended therapy for anorexia. Despite receiving a variety of resource information, the patient refuses any medical intervention. She continues to present to the family physician, offering a variety of somatic complaints. When a patient's preferences conflict with a physician's goal to restore health, which ethical principle should prevail, patient autonomy or physician beneficence? Does the patient's depression render her incompetent to refuse treatment for her anorexia?

Since this patient could rationally discuss her treatment options and her reasons for declining therapy, she could not be considered incompetent. Respect for autonomy is a central principle of bioethics, and it takes precedence in this case. Although the principle of beneficence could be used to argue for coercion towards treatment, compliance may be better improved by providing an ongoing partnership with the patient. Maintaining a therapeutic relationship with ongoing dialogue is more likely to provide this patient with the eventual ability to pursue therapy.

How pervasive is religiosity in the United States?

Religious belief and practice is pervasive in this country, although less pervasive within the medical profession. Surveys of the US public in the 2008 Gallup Report consistently show a high prevalence of belief in God, 78% and an additional 15% who believe in a higher power (Newport, 2009). In an aggregate of 2013 polls, 56% claim that religion is important in their own lives and 22% claim it is fairly important (Gallup, 2013). Washington State is one of ten states claiming the least importance of religion, at 52% (Newport, 2009). In 2010, approximately 43.1% of Americans reportedly attended religious services at least once a week (Newport, 2010). 77% of Americans identified themselves as Christian, 5% with a non-Christian tradition, and 18% did not have an explicit religious identity (Newport, 2012). One survey in Vermont involving family physicians showed that 91% of the patients reported belief in God as compared with 64% of the physicians (Maugans & Wadland, 1991). A 1975 survey of psychiatrists showed that an even lower number, 43%, professed a belief in God (American Psychiatric Association, 1975). These surveys remind us that there is a high incidence of belief in God in the US public. It also appears that physicians as a group are somewhat less inclined to believe in God. Whereas, up to 77 percent of patients would like to have their spiritual issues discussed as a part of their medical care, less than 20% of physicians currently discuss such issues with patients (King & Bushwick, 1994). Clearly, physicians are not inquiring about spirituality to nearly the degree that patients prefer.

What is a Do Not Attempt Resuscitation (DNAR) Order?

A Do Not Attempt Resuscitation (DNAR) Order, also known as a do not resuscitate (DNR) order, is written by a licensed physician in consultation with a patient or surrogate decision maker that indicates whether or not the patient will receive cardiopulmonary resuscitation (CPR) in the setting of cardiac and/or respiratory arrest. CPR is a series of specific medical procedures that attempt to maintain perfusion to vital organs while efforts are made to reverse the underlying cause for the cardiopulmonary arrest. Although a DNAR order may be a component of an advance directive or indicated through advance care planning, it is valid without an advance directive.

What about the patient whose decision making capacity varies from day to day?

A patient's decision-making capacity is variable as their medications or underlying disease processes ebb and flow. You should do what you can to catch a patient in a lucid state - even lightening up on the medications if necessary and safe - in order to include her in the decision making process. Delirious patients have waxing and waning abilities to understand information. However, if a careful assessment is done and documented at each contact, and during lucid periods the patient consistently and persistently makes the same decision over time, this may constitute adequate decisional capacity for the question at hand.

What should I do when my preceptor introduces me as "Dr. Miller"?

Some community preceptors prefer to introduce their medical students as "doctor" because they feel it encourages patient trust. However, it is important to recognize that by calling yourself "doctor," you are misrepresenting yourself to the patient. As with other truth-telling and informed consent issues, it is appropriate to disclose to the patient what he or she needs to know. In this case, the patient needs to know you are a physician-in-training! Students have found themselves in awkward situations once the patient begins asking questions that a physician should know how to answer. At this stage, even if you clarify, "Actually, I'm a medical student, not yet a physician," patient trust may be damaged. It can be a difficult conversation to have with your preceptors, but it is best to discuss this matter in advance. Find out what his or her expectations are. If they feel strongly about introducing you as "doctor," it remains your responsibility to explain tactfully that you cannot misrepresent yourself to patients. In the long run, patients' trust will be secured if they realize you are both being straightforward. If the preceptor insists, you may need to find polite ways to reintroduce yourself to the patient, modeling for the preceptor that direct communication is often the best foundation for a strong physician (and student-physician) patient relationship. For example, you might say, "Yes, I am a physician-in-training from University of Washington."

An 80-year-old Asian woman is hospitalized with weight loss, generalized weakness, and a pulmonary mass. Work-up reveals that she has pulmonary tuberculosis. Her family approaches the physician and asks that the patient not be told, stating that in her upbringing in mainland China tuberculosis was considered fatal and to tell her would be like giving her "a death sentence." Should you respect the family's concerns?

Some cultures hold different beliefs about truth-telling in the medical encounter. Some assert that in some Asian cultures, members of the family unit may withhold the truth about terminal illness from elders out of respect and a desire to protect them from harm. If a patient and their family members hold such beliefs, they should be respected, and a mechanism for informed decision making in collaboration with the family negotiated. One must not, however, assume that every patient of Asian ancestry holds the beliefs described here. The physician should make an attempt to explore the patient's belief system. If he finds that the patient does hold such beliefs about the harmful nature of truthful disclosure of the truth, then it would be justifiable to withhold the diagnosis of tuberculosis.

When does a fetus or a newborn become a person?

Some ethicists purport that viability entitles the fetus to "moral personhood". Viability is the physical capacity for life independent of maternal corporeal support. They argue that newborns and fetuses participate in the social matrix, and that this social role develops over time, beginning prior to birth. Others note that it is impossible to treat fetuses as persons without treating pregnant women as if they were less than persons. The birth of the fetus results in a distinct patient towards whom medical therapy can be individually directed. As such, many believe that the moral status of a developmentally younger newborn supersedes that of an older viable fetus.

Won't disclosing mistakes to patients undermine their trust in physicians and the medical system?

Some patients may experience a loss of trust in the medical system when informed that a mistake has been made. Many patients experience a loss of trust in the physician involved in the mistake. However, nearly all patients desire some acknowledgment of even minor errors. Loss of trust will be more serious when a patient feels that something is being hidden from them.

Is deception of subjects allowed when doing research?

As a general rule, deception is not acceptable when doing research with humans. Using deception jeopardizes the integrity of the informed consent process and could harm participants, as well as eroding trust between the public and researchers. In some instances deception is necessary to conduct the research: for example, a study of how the decision-making practices of physicians affect their practice of medicine might be presented as a study of "communication behaviors." The IRB will carefully review any proposal that suggests using deception or misrepresentation. They will require an in-depth justification of why deception is necessary for the study and the steps that will be taken to safeguard participants, including a plan to debrief subjects at the end of the research. A form of deception of subjects can occur if the terms of the informed consent are violated by the investigator or other scientists. For example, if a subject consents to have their blood sample evaluated for markers of diabetes and the investigator or other scientists use the blood sample for purposes unrelated to diabetes research, the subject has effectively been deceived about the nature of their research participation and they have been denied autonomous agency over their own actions. The recent court case involving Arizona State University v. the Havasupai tribe illustrates the ethical pitfalls and legal consequences of biological sample sharing without explicit prior consent

What are social determinants of health?

As these accounts suggest, public health is distinct from medicine in several ways. It is oriented toward the health of populations or groups rather than individuals; the prevention, rather than the treatment, of disease; the elimination of systematic inequalities in population health associated with social class, race/ethnicity, gender, and other markers of disadvantage; and the use of state power and collective resources to accomplish these ends. To pursue these goals, public health must address a broad array of social conditions that extend beyond medical services. The "social determinants of health"-sometimes simply referred to as the conditions in which people grow up, live, work, and play (Commission on the Social Determinants of Health 2008)-include the built and physical environments (e.g., sidewalks, highway safety, food access, water and air quality); conditions of housing, schools, neighborhoods, and the workplace; and broader social, economic, and cultural conditions related to education and employment, income and wealth, racism and sexism, and social support and security (Woolf, Braveman 2011; Jones et al. 2009). Since its inception, public health has recognized social conditions as basic causes of illness and disease. Upon investigating and reporting on the causes of typhus in 1848, public health pioneer Rudolf Virchow linked famine and poverty with the oppression of Polish peasants and argued that the "elimination of social inequality was the only way to prevent typhus epidemics in the future" (Mackenbach 2009). The late twentieth century has witnessed renewed interest in social determinants as root causes of significant and widening disparities in health-both within and between countries (Commission on the Social Determinants of Health, 2008). In the United States, disparities in life expectancy have worsened for men and women since 1980 (Marmot, Bell 2009), and examples of such disparities are often dramatic. For example, the gap in left expectancy between men in Washington D.C. and in suburban Maryland is 17 years (Marmot, Bell 2009). Such challenges in combination with the emergence and reemergence of infectious diseases in the late twentieth century (Armstrong et al. 1999) raise many complex ethical questions for international organizations, governments, communities, and individuals. While early ethical commentary focused on cases such as health promotion and HIV/AIDS (Wikler 1978; Pellegrino 1981; Bayer and Moreno 1986; Bayer 1993, 1989), systematic analyses of the ethical foundations and frameworks of public health are much more recent. A journal dedicated to the topic-Public Health Ethics-was not established until 2008.

When can CPR be withheld?

As with CPR on the medical wards, two general situations arise in which CPR can be withheld in the operating room: when CPR is judged to be of no benefit to the patient (See the main topic page, Do-Not-Resuscitate Orders.) when a patient with intact decision-making capacity (or in the case of those without decision making capacity, an appropriate surrogate decision-maker) indicates that they do no want CPR, even if the need arises.

What is Public Health Ethics?

As with the concept of public health, there is no settled account of the moral concepts and methods of public health ethics. Systematic efforts to articulate ethical principles and frameworks to guide ethical inquiry in public health (Kass 2001; Childress et al 2002; Roberts, Reich 2002; Upshur 2002; Powers, Faden 2006;The Nuffield Bioethics Council 2007; Daniels 2008; Arras, Fenton 2009; Bernheim et al. 2013) identify a number of general moral considerations that include: Producing benefits, often but not exclusively health benefits, and often interpreted in health policy as a utilitarian commitment to maximizing aggregate health benefits Preventing harms, often health harms, such as preventable morbidity and premature death Distributing health benefits fairly, or distributive justice (fair distribution of social goods) Procedural justice (fair process), participation, and transparency Respecting individual autonomy and liberty of action Respecting and fulfilling universal human rights Respecting privacy and confidentiality Protecting non-dominant subgroups from marginalization and stigmatization Building and maintaining trust Much work needs to be done to translate these general ethical considerations or some subset of them into guiding principles and frameworks for public health policy and practice. Such work entails defining them, determining their scope, specifying criteria for resolving conflicts among them, and so on. Such efforts have produced frameworks of unranked principles (e.g., Nuffield Bioethics Council 2007), theories of social justice (Powers, Faden 2006), among other more practical approaches (e.g., Kass 2001). Such frameworks have counterparts in medical ethics, whose methods reflect a similar diversity

Mrs. P is a 74-year-old woman presenting for emergent treatment of a fracture-dislocation of her right hip, suffered in a fall at her nursing home. She appears frail, but is alert and oriented. She is accompanied by her daughter, and both state that they want her to receive full medical care. On admission two hours earlier, the emergency room physician heard a loud systolic murmur, and echocardiogram revealed critical aortic stenosis, with a valve area of 0.3 cm2. The surgeon suggests that the patient, because of her cardiac status and age, should have a DNR order in the chart. Do you agree?

Cardiac arrest in the setting of critical aortic stenosis carries virtually zero chance of survival, since the tight stenosis of the left ventricular outflow tract makes generation of systemic blood pressures compatible with life virtually impossible. CPR in this setting can easily be termed "medically futile." Many hospital policies require that patient or family agreement accompany DNR orders. A frank discussion with Mrs. P and her daughter about the issue of CPR should be initiated with the hope of establishing understanding with the patient and her family about the question of resuscitation.

A young accident victim has been in a persistent vegetative state for several months and family members have insisted that "everything possible" be done to keep the patient alive. What are your professional obligations?

Case 1 illustrates the possible conflicts that can arise with patients or family members about withholding or withdrawing futile interventions. If you and other members of the health care team agree that the interventions in question would be futile, the goals of treatment should be clarified. It can be helpful to ask patients and family members to also articulate their goals, which may reveal some agreement among parties. Once goals are clear, physicians can discuss how various interventions help or frustrate these goals. This requires a process of working with the patient and/or family, and possibly drawing on other resources, such as social workers, palliative care services, hospital chaplains, and ethics consultants. If there is no professional consensus about the futility of a particular intervention, then there is no ethical basis for overriding the requests of patients and/or family members for that intervention.

On a busy night in the ER a member of the hospital board comes in with her sick child and asks that you see him right away. The child has a sore throat and red eye and he appears subdued, but alert. You have a full waiting room. What should you do?

Cases where famous or influential people are asking for special treatment ask that we review our ethical criteria for resource allocation. Do some people "deserve" special treatment over others? What would justify such a claim? In this case, the ER staff might be swayed by the powerful position the board member holds in their institution and want to do their best for her. However, the other people waiting in the ER have been subject to triage criteria based on medical need. It would be unjust to waive these criteria on the basis of social position. While this may seem unrealistic, one might also consider the effect on the hospital if the board member faces a long, tedious wait in the waiting room along with everyone else. A complaint voiced by this powerful person may enact change on staffing considerations more effectively than a number of patient complaints. To let her sail through would be to create an impression of smoothness that is most likely not part of the everyday ER experience.

How do I contact the ethics committee or request an ethics consultation?

Check with your hospital to identify the pager number to reach the ethics consultant. There should be an individual at each hospital that carries a pager for responding to ethics consultations. At UWMC, the ethics consultant can be reached through the hospital's paging operator at 206.598.6190. At Harborview Medical Center, you should call 206.744.3000, and at VA Puget Sound Medical Center, you should call the ethics consultation service pager, 206.762.1010.

What does an ethics committee or program do?

Historically, ethics committees involve individuals from diverse backgrounds who support health care institutions with three major functions: providing clinical ethics consultation, developing and/or revising policies pertaining to clinical ethics and hospital policy (e.g., advance directives, withholding and withdrawing life-sustaining treatments, informed consent, organ procurement), and facilitating education about topical issues in clinical ethics. The underlying goals of traditional ethics committees are: to promote the rights of patients; to promote shared decision making between patients (or their surrogates if decisionally incapacitated) and their clinicians; to promote fair policies and procedures that maximize the likelihood of achieving good, patient-centered outcomes; and to enhance the ethical environment for health care professionals in health care institutions. Ethics committees or select members often help resolve ethical conflicts and answer ethical questions through the provision of consultations. More recently, some ethics committees, particularly those affiliated with academic institutions and large health care systems, have expanded their traditional functions to become more comprehensive ethics programs. They address both clinical and organizational ethics issues. Ethics programs may provide ethics consultations in response to non-clinical ethics questions, identify and remedy systems-level factors that induce or exacerbate ethical problems and/or impede their resolution (often using quality improvement methods), and promote a positive ethics culture throughout the institution. Thus, the additional goals of expanded ethics programs include: integrating ethics throughout the health care institution from the bedside to the boardroom, ensuring that systems and processes contribute to/do not interfere with ethical practices, and promoting ethical leadership behaviors, such as explaining the values that underlie decisions, stressing the importance of ethics, and promoting transparency in decision making

If CPR is deemed "futile," should a DNAR order be written?

If health care providers unanimously agree that CPR would be medically futile, clinicians are not obligated to perform it. Nevertheless, the patient and/or their family still have a role in the decision about a Do Not Attempt Resuscitation (DNAR) order. As described earlier, involving the patient or surrogate decision maker is essential to demonstrate respect for all people to take part in important life decisions. In many cases, patients or surrogate decision makers will agree to forgo attempting CPR following a transparent and honest discussion regarding the clinical situation and the limitations of medicine. Under these circumstances, DNAR orders can be written. Each hospital has specific procedures for writing a valid DNAR order.

How much do patients need to be told?

In addition to fostering trust and demonstrating respect, giving patients truthful information helps them to become informed participants in important health care decision. Thus, patients should be told all relevant aspects of their illness, including the nature of the illness itself, expected outcomes with a reasonable range of treatment alternatives, risks and benefits of treatment, and other information deemed relevant to that patient's personal values and needs. Treatment alternatives that are not medically indicated or appropriate need not be revealed. Facts that are not important to the patients ability to be an informed participant in decision making, such as results of specific lab tests, need not be told to the patient. Also, complete and truthful disclosure need not be brutal; appropriate sensitivity to the patient's ability to digest complicated or bad news is important.

Would a physician ever be justified in breaking a law requiring mandatory reporting?

In general, mandatory reporting requirements supersede the obligation to protect confidentiality. While the physician has a moral obligation to obey the law, she must balance this against her responsibility to the patient. Reporting should be done in a manner that minimizes invasion of privacy, and with notification to the patient. If these conditions cannot be met, or present an intolerable burden to the patient, the physician may benefit from the counsel of peers or legal advisors in determining how best to proceed.

An elderly man who lives in a nursing home is admitted to the medical ward with pneumonia. He is awake but severely demented. He can only mumble, but interacts and acknowledges family members. The admitting resident says that treating his pneumonia with antibiotics would be "futile" and suggests approaching the family with this stance. Do you agree?

In many cases, "futility" is used inaccurately to describe situations that appear undesirable. For this patient, treating pneumonia with antibiotics stands a reasonable chance of success. The patient's quality of life, though low, is not unacceptably so. Unless the patient (or if found incapacitated, his surrogate) were to say that he would find this quality of life unacceptably low, there is neither quantitative nor qualitative grounds for calling antibiotics futile in this case. Unlike Cases 1 and 2, in Case 3 there is a treatment available that benefits the patient.

What is the basis for granting medical decision-making authority to parents?

In most cases, a child's parents are the persons who care the most about their child and know the most about him or her. As a result, parents are better situated than most others to understand the unique needs of their child and to make decisions that are in the child's interests. Furthermore, since many medical decisions will also affect the child's family, parents can factor family issues and values into medical decisions about their children.

When is it appropriate to question a patient's ability to participate in decision making?

In most cases, it is clear whether or not patients have capacity to make their own decisions. Occasionally, it is not so clear. Patients are under an unusual amount of stress during illness and can experience anxiety, fear, and depression. The stress associated with illness should not necessarily preclude one from participating in one's own care. However, precautions should be taken to ensure the patient does have the capacity to make good decisions. There are several different standards of decision-making capacity. Generally you should assess the patient's ability to: Understand his or her situation, Understand the risks associated with the decision at hand, and Communicate a decision based on that understanding. When this is unclear, a psychiatric consultation can be helpful. Of course, just because a patient refuses a treatment does not in itself mean the patient is incompetent. Competent patients have the right to refuse treatment, even those treatments that may be life-saving. Treatment refusal may, however, be an indication that it is necessary to pause to discuss further the patient's beliefs and understanding about the decision, as well as your own.

What if the patient is not competent?

In some cases, the patient is clearly unable to voice a wish to have treatment withheld or withdrawn. As with DNR orders, there are two general approaches to this dilemma: Advance Directives and surrogate decision makers. Advance Directive: This is a document which indicates with some specificity the kinds of decisions the patient would like made should he/she be unable to participate. In some cases, the document may spell out specific decisions (e.g. Living Will), while in others it will designate a specific person to make health care decisions for them (i.e. Durable Power of Attorney for Health Care). There is some controversy over how literally Living Wills should be interpreted. In some cases, the document may have been drafted in the distant past, and the patient's views may have changed. Similarly, some patients do change their minds about end-of-life decisions when they actually face them. In general, preferences expressed in a Living Will are most compelling when they reflect long held, consistently stable views of the patient. This can often be determined by conversations with family members, close friends, or health care providers with long term relationships with the patient. Surrogate decision maker: In the absence of a written document, people close to the patient and familiar with their wishes may be very helpful. (See Advance Care Planning.) The law recognizes a hierarchy of family relationships in determining which family member should be the official "spokesperson," though generally all close family members and significant others should be involved in the discussion and reach some consensus. The hierarchy is as follows: Legal guardian with health care decision-making authority Individual given durable power of attorney for health care decisions Spouse Adult children of patient (all in agreement) Parents of patient Adult siblings of patient (all in agreement)

How do physicians who care for the dying deal with their own feelings?

It is not hard to find physicians who are burned out - ask any nurse. What is difficult is to find for yourself a type of self-care that will enable you to develop your gifts as a physician, and continue to use them in practice. It helps to learn your strengths and weaknesses, and to actively seek whatever will nurture you - in or out of medicine. A strategy of detachment may not serve you well in the long run. There are indeed rewards for physicians who care for the dying, but as a Zen master once observed of a bingo game, "you must be present to win."

An elderly patient with irreversible respiratory disease is in the intensive care unit where repeated efforts to wean him from ventilator support have been unsuccessful. There is general agreement among the health care team that he could not survive outside of an intensive care setting. The patient has requested antibiotics should he develop an infection and CPR if he has a cardiac arrest. Should a distinction be made between the interventions requested by the patient? Should the patient's age be a factor?

Like Case 1, Case 2 illustrates possible conflicts that can arise with patients or family members about withholding or withdrawing futile interventions. Ventilator support clearly produces a physiologic effect by contracting and expanding the patient's lungs with oxygen. Yet this does not suffice to show that the treatment helps the patient, which is medicine's goal. If the patient will never leave the intensive care unit, does ventilator support constitute a benefit to the patient? A central issue this case raises concerns qualitative futility, namely: does the patient's quality of life fall well below a threshold considered minimal? If you and other members of the health care team agree that the interventions in question would be qualitatively futile, the goal should be to withdraw or withhold these interventions. Achieving this goal requires working in tandem with the patient and/or family, as well as drawing upon resources, such as social workers, hospital chaplains, and ethics committees. If there is no professional consensus about the futility of a particular intervention, then there is no ethical basis for overriding the requests of patients and/or family members for that intervention.

What are the limitations of living wills?

Living wills cannot cover all conceivable end-of-life decisions. There is too much variability in clinical decision making to make an all-encompassing living will possible. Persons who have written or are considering writing advance directives should be made aware of the fact that these documents are insufficient to ensure that all decisions regarding care at the end of life will be made in accordance with their written wishes. Moreover, the language that is often employed in advance directives is frequently imprecise. What seems clear to the author may seem cloudy to others when reviewed in a clinical situation. A partial remedy to this limitation is to strongly encourage patients to communicate preferences and values to both their medical providers and family/surrogate decision makers, and to encourage them to identify a health care agent (through a durable power of attorney for health care) where appropriate. Another potential limitation of advance directives is possible changes in the patient's preferences over time or circumstance. People often accommodate to disabilities and an old living will may become inconsistent with the patient's revised views about quality of life or other outcomes. This is yet another reason to recommend ongoing communication between patients and their physicians and family members.

Is there an increased morbidity associated with higher survival rates?

Long-term follow up studies of NICU very low birth weight survivors demonstrate that while many are normal(16%) , there are significant numbers of children with mild(39%), moderate(31%), and severe(14%)Â handicapping conditions. Major neurodevelopmental handicaps include cerebral palsy, mental retardation, blindness, and deafness. Less severe conditions include lower IQ, learning disabilities, mild movement disorders, etc. (See: "Neurodevelopmental disability through 11 years of age in children born before 26 weeks of gestation." Pediatrics. 2009; 124; e249-e257.) Data pertaining to the long-term impact of the survival of infants with complex congenital anomalies are less plentiful, but indicate an increased number of handicapped individuals who survive the neonatal/infancy period following successful life-prolonging treatment.

When can CPR be withheld?

Many hospitals have policies that describe circumstances under which CPR can be withheld based on the practical reality that CPR does not always provide direct medical benefit. Two general situations justify withholding CPR: When CPR will likely be ineffective and has minimal potential to provide direct medical benefit to the patient. When the patient with intact decision making capacity or a surrogate decision maker explicitly requests to forgo CPR.

How much of herself should the physician bring to the physician-patient relationship?

Many patients may feel more connected to a physician when they know something of the physician's life, and it may sometimes be appropriate to share information about family or personal matters. However, it is essential that the patient, and the patient's concerns, be the focus of every visit.

Is it possible to predict which infants will not survive despite aggressive medical/surgical care in the neonatal period?

Many researchers have evaluated population-based studies looking at mortality and morbidity (above data). Although general risks can be identified, there are no successful models to assess individualizable predictability of death or disability. Illness severity scores (SNAP-II, CRIB scores) and health care provider intuition scores exist, but none have been validated to predict individual outcomes. Decisions regarding treatment options must be based on population-based studies, physician recommendations, and parental perspectives.

Why do we agree to do surgery on patients with DNR orders?

Many types of surgery provide palliative benefits to patients who either will not survive long-term, or who do not wish resuscitation in the OR. A patient with an esophageal obstruction from cancer might benefit from gastrostomy placement through reduced pain and improved nutritional status, yet not want CPR if cardiac arrest happens in the OR. Requiring such a patient to suspend their DNR orders to be a candidate for surgery uses their discomfort, pain, and desire to benefit from surgery to coerce them into accepting medical care (CPR) they do not want. Patient refusal of some medical therapy, such as CPR, does not ethically justify physicians denying them other medical therapy, such as surgery, that might benefit them.

Mr. H is a 24-year-old man who resides in a skilled nursing facility, where he is undergoing rehabilitation from a cervical spine injury. The injury left him quadriplegic. He has normal cognitive function and no problems with respiration. He is admitted to your service for treatment of pneumonia. The resident suggests antibiotics, chest physiotherapy, and hydration. One day while signing out Mr. H to the cross covering intern, the intern says "he should be a DNR, based on medical futility." Do you agree? Is his case medically futile, and if so, why?

Medical futility means that an intervention, in this case CPR, offers no chance of meaningful medical benefit to the patient. Interventions can be considered futile if the probability of success (discharged alive from the hospital) is <1%, and/or if the CPR is successful, the quality of life is below the minimum acceptable to the patient. In this case, Mr. H would have a somewhat lower than normal chance of survival from CPR, based on his quadriplegia (homebound lifestyle is a poor prognostic factor). Furthermore, his quality of life, while not enviable, is not without value. Since he is fully awake and coherent, you could talk with Mr. H about his view of the quality of his life, particularly focusing on his goals and hopes for the future. You could share with him the likely scenarios should he have an arrest and the likely outcomes following CPR. After this discussion and clearly understanding Mr. H's goals, you can partner with Mr. H to determine whether or not CPR is indicated in the event of an arrest. In this case, CPR is not necessarily futile. A decision about resuscitation should occur only after talking with the patient about his situation, goals, and hopes in his life in order to make a shared and mutual decision.

What if the patient's family asks me to withhold the truth from the patient?

Often families will ask the physician to withhold a terminal or serious diagnosis or prognosis from the patient. Usually, the family's motive is laudable; they want to spare their loved one the potentially painful experience of hearing difficult or painful facts. These fears are usually unfounded, and a thoughtful discussion with family members, for instance reassuring them that disclosure will be done sensitively, will help allay these concerns. In unusual situations, family members may reveal something about the patient that causes the physician to worry that truthful disclosure may create real and predictable harm, in which case withholding may be appropriate. These occasions, however, are rare.

Are there ethical criteria for making triage decisions?

One common medical situation in which specific principles must be applied is called "triage." Triage (which means "choice" or "selection") is required when many patients simultaneously need medical attention and medical personnel cannot attend to all at the same time, such as in a disaster or in the crowded emergency department of an urban hospital. Again, the common sense rule is to serve persons whose condition requires immediate attention and, if this attention is not given, will progress to a more serious state. Others, whose condition is not as serious and who are stable, may be deferred. A second sort of triage is indicated in disasters, such as earthquakes, or in military action. The rules of military triage, developed centuries ago, direct the physician to attend first to those who can be quickly and successfully treated in view of a speedy return to the battlefield, or to treat commanders before troops in order to assure leadership. This sort of disaster triage is applied to civilian disasters by treating persons, such as firefighters or public safety officers, who can quickly return to duty and help others. Disaster triage implies that the most seriously injured may be relegated to the end of the line and left untreated, even at risk of death, if their care would absorb so much time and attention that the work of rescue would be compromised. This is one of the few places where a "utilitarian rule" governs medicine: the greater good of the greater number rather than the particular good of the patient at hand. This rule is justified only because of the clear necessity of general public welfare in a crisis.

What about patients with different specific religious or cultural beliefs??

Patient with certain religious beliefs or ethnic or cultural backgrounds may have different views on the appropriateness of truthful disclosure. For instance, Carrese and colleagues found that many people with traditional Navajo beliefs did not want to hear about potential risks of treatment, as their beliefs held that to hear such risks was to invite them to occur. Thus, dialogue must be sensitive to deeply held beliefs of the patient. One should not, however, assume that someone of a particular ethnic background holds different beliefs. Rather, a culturally sensitive dialogue about the patient's role in decision making should take place.

Does depression or other history of mental illness mean a patient has impaired decision making capacity?

Patients with active mental illness including depression should have their decision making capacity evaluated carefully. They should not be presumed to be unable to make treatment decision. In several studies, patients voiced similar preferences for life-sustaining treatments when depressed as they did after treatment of their depression. Depression and other mental disorders should prompt careful evaluation, which may often be helped by psychiatry consultation.

Mrs. H is a 62-year-old woman with metastatic breast cancer. She was admitted with dehydration and weakness. Her cancer treatments have failed, as she now has a recurrence. The oncologists are contemplating some new palliative chemotherapy. The nutrition team is concerned about her cachexia and recommends total parenteral nutrition (TPN). Should the patient be started on TPN?

Patients with metastatic cancer often suffer from profound cachexia, attributable to the metabolic effects of their cancer and their inability to get adequate caloric intake from eating alone. TPN is able to provide protein and nonprotein nutrients to reverse the catabolic effects of illness. TPN has a number of potential complications, such as those related to infection from the central line catheter site. In this case, you should carefully evaluate the goals of therapy as they relate to TPN. Is TPN likely to offer the patient any benefit? If her life expectancy can be prolonged with additional chemotherapy, it may be reasonable to give TPN to allow the patient to enjoy that benefit. If additional chemotherapy offers no substantial increase in quantity or quality of life, TPN could become another burden for the patient without any meaningful benefit, and ought to be withheld.

What can a physician do with a particularly frustrating patient?

Physicians will sometimes encounter a patient whose needs, or demands, strain the therapeutic alliance. Many times, an honest discussion with the patient about the boundaries of the relationship will resolve such misunderstandings. The physician can initiate a discussion by saying, "I see that you have a long list of health concerns. Unfortunately, our appointment today is only for fifteen minutes. Let's discuss your most urgent problem today and reschedule you for a longer appointment. That way, we can be sure to address everything on your list." Or, "I know that it has been hard to schedule this appointment with me, but using abusive language with the staff is not acceptable. What do you think we could do to meet everybody's needs?" There may be occasions when no agreeable compromise can be reached between the physician and the patient. And yet, physicians may not abandon patients. When the physician-patient relationship must be severed, the physician is obliged to provide the patient with resources to locate ongoing medical care.

What happens when the physician has a relationship with multiple members of a family?

Physicians with relationships with multiple family members must honor each individual's confidentiality. Difficult issues, such as domestic violence, sometimes challenge physicians to maintain impartiality. In many instances, physicians can help conflicted families towards healing. At times, physicians work with individual family members; other times, they may serve as a facilitator for a larger group. As always, when a risk for imminent harm is identified, the physician must break confidentiality. Physicians can be proactive about addressing the needs of changing family relationships. For example, a physician might tell a preteen and her family, "Soon you'll be a teenager. Sometimes teens have questions they would like to discuss with me. If that happens to you, it's okay to tell your parents that you'd like an appointment. You and I won't have to tell your parents what we talk about if you don't want to, but sometimes I might encourage you to talk things over with them." The physician-family relationship also holds considerable healing power. The potential exists to pursue options that can improve the quality of life and health for the entire family.

How should I respond when an intern asks if I want to practice a procedure on a patient who just died?

Practicing procedures on newly dead patients is a highly contentious issue. For some procedures, like intubation, students can benefit from practicing first on a cadaver. Weighing the risks and benefits, the student is more likely to harm a living patient were she to try to intubate without practicing first. No physical harm can occur to the cadaver. However, some are appropriately concerned about the disrespect that "practicing" procedures may show to the deceased patient or to the patient's family. It remains your responsibility to assure that your interactions with the cadaver are respectful and only as invasive as necessary. Most likely, the time you take to practice a simple procedure will not add significantly to the usual amount of time needed to prepare the patient's body for viewing by the family if they are waiting nearby. This is important to be cognizant of, however.

What is meant by "respectful" exchange of views?

Precisely because of the inequality of authority and responsibility in inter-professional, inter-physician, and student-teacher relationships, obligations of mutual respect are particularly important on the multidisciplinary team. Disagreements between professionals are common and expected, because of different knowledge, experience, values, and perspectives of the various team members. While disagreements might be settled in a number of ways, mutual respectful behavior is a mandatory feature of professionalism. Thus, while it is not only possible, but expected, that members of the patient care team will disagree at times, it is never acceptable for disagreements to be verbalized in an unprofessional manner. Respectful behavior begins with both listening to and considering the input of other professionals. Ask yourself whether your perception of whether you are respected depends more upon whether the other party agrees with you, or whether, despite disagreeing, they listened and acknowledged your point of view. Respect is demonstrated through language, gestures, and actions. Disagreement can and should be voiced without detrimental statements about other members of the team, and without gestures or words that impart disdain. Both actions and language should impart the message: "I acknowledge and respect your perspective in this matter, but for the following reasons. I disagree with your conclusions, and believe I should do something else..." It should go without saying that disrespectful behavior from a colleague does not justify disrespectful behavior in return.

How are DNAR Orders Written?

Prior to writing a DNAR Order, physicians should discuss resuscitation preferences with the patient or his/her surrogate decision maker (Blinderman et al., 2012; Quill et al., 2009). This conversation should be documented in the medical record, indicating who was present for the conversation, who was involved in the decision making process, the content of the conversation, and the details of any disagreement. These conversations are difficult and involve a careful consideration of the potential likelihood for clinical benefit within the context of the patient's preferences. Physicians can most effectively guide the conversation by addressing the likelihood of direct benefit from cardiopulmonary resuscitation within the context of the overall hopes and goals for the patient. They can then partner with the patient and his or her family to determine the clinical interventions that most effectively achieve these goals (Blinderman et al., 2012). This approach is described by the palliative care literature as a goal oriented approach to providing end of life care.

Do I have to do whatever I am told by the attending physician, even if I disagree with their plans?

Professional relationships not only exist between different professions, and specialties within similar professions, but between students and teachers as well. The student-teacher relationship is also an unequal one, not merely because teachers generally have more authority than students, based on their training and years of experience, but much greater responsibility as well. An attending physician, for example, may be held both morally and legally liable for the actions of students or residents, whether or not she approved of those actions. Ethically, teachers have obligations to observe and control the actions of junior members of the medical team, both to prevent harm to patients from inexperienced care-givers, and to educate students in appropriate care. Students and residents, conversely, have obligations to their patients and to their teachers, to not act recklessly or without the knowledge and approval of supervisors. Whenever a student or resident disagrees with an attending physician's plans, he should seek input from the attending, both about the reasoning to pursue the attending's plan, and about the reasoning for rejecting her own. A respectful exchange of views may provide both parties with new information, and certainly serves to further education.

What are the recognized obligations and values of a professional physician?

Professionalism requires that the practitioner strive for excellence in the following areas, which should be modeled by mentors and teachers and become part of the attitudes, behaviors, and skills integral to patient care: Altruism: A physician is obligated to attend to the best interest of patients, rather than self-interest. Accountability: Physicians are accountable to their patients, to society on issues of public health, and to their profession. Excellence: Physicians are obligated to make a commitment to life-long learning. Duty: A physician should be available and responsive when "on call," accepting a commitment to service within the profession and the community. Honor and integrity: Physicians should be committed to being fair, truthful and straightforward in their interactions with patients and the profession. Respect for others: A physician should demonstrate respect for patients and their families, other physicians and team members, medical students, residents and fellows. These values should provide guidance for promoting professional behavior and for making difficult ethical decisions. A Physician Charter: Medical Professionalism in the New Millennium was issued jointly by the American Board of Internal Medicine, the American College of Physicians and the European Federation of Internal Medicine in 2002. Subsequently, 90 professional associations, including most of the specialty and subspecialty groups in American medicine have endorsed the Charter. The fundamental principles of professionalism are stated as (1) the primacy of patient welfare; (2) patient autonomy; (3) social justice. Professional responsibilities that follow from these principles are commitment to competence, to honesty with patients, to confidentiality, to appropriate relationship with patients, to improving quality of care, to improving access to care, to a just distribution of finite resource, to scientific knowledge, to maintaining trust by managing conflicts of interests and to professional responsibilities.

What accounts for the rising awareness of maternal-fetal conflict?

Advances in medical technology have increased the physician's ability to direct medical procedures towards the fetus. Previously, physicians conceptualized the maternal-fetal dyad as one complex patient. Viewed as an organic whole, the combined maternal and fetal benefits of a proposed therapy could be weighed against the combined burdens. Distribution of benefits and burdens between the fetal and maternal components of the one patient was not ethically relevant. Over time, the medical model for the maternal-fetal relationship has shifted from unity to duality. When there are two individual patients, the physician must decide what is medically best for each patient separately.

During the conduct of a large clinical trial of an investigational drug, preliminary analysis of results show that there were three times as many participants in the experimental group who experienced severe nausea and vomiting compared to the control group. Two of the cases were severe enough to require that the participants be hospitalized. This is despite the fact that the preliminary analysis shows that there may be a moderate benefit with the drug. What should be done?

All adverse events are reported to the Data Safety Monitoring Board, and serious adverse events must be reported to the IRB. Investigators should consider reevaluating the balance of risks versus benefits. While these adverse events are probably not serious enough to suggest that the trial be terminated, these adverse events should be balanced by significant benefits. These findings may relate to each participant's willingness to continue in the trial, and according to 45 CFR 46.116(b)(5) the findings should be provided to all current and future participants as part of the informed consent process.

What sorts of interventions require informed consent?

All health care interventions require some kind of consent by the patient, following a discussion of the procedure with a health care provider. Patients fill out a general consent form when they are admitted or receive treatment from a health care institution. Most health care institutions, including UWMC, Harborview, and VAMC also have policies that state which health interventions require a signed consent form. For example, surgery, anesthesia, and other invasive procedures are usually in this category. These signed forms are the culmination of a dialogue required to foster the patient's informed participation in the clinical decision. For a wide range of decisions, explicit written consent is neither required nor needed, but some meaningful discussion is always needed. For instance, a man contemplating having a prostate-specific antigen screen for prostate cancer should know the relevant arguments for and against this screening test, discussed in lay terms.

What is the difference between an ethics committee and an ethics consultant?

An ethics consultant is an expert in ethics who provides ethics consultations and may also serve as an educator to the committee or program. In some health care institutions an ethics consultant provides ethics expertise to workgroups that are addressing systems issues and have the need to better understand the ethics and preferred practices from an ethics perspective. Sometimes in lieu of having an ethics consultant address ethics questions or concerns, the ethics committee will develop a subcommittee to handle these functions. The decision to have an ethics consultant versus a subcommittee rests with the available resources and the expertise of the committee members. In general, the strengths of having an ethics consultant is that she is a recognized expert, and the logistics of having someone perform a consultation is straight forward. The weaknesses are that clinicians can rely on this outside person for the answers to their questions and not develop their own expertise, and only one voice/perspective gets expressed. The major strength of having subcommittees or a consultation service (having 2-3 people per month) perform consultations is that this structure incorporates a diversity of views. The major weakness is the logistical difficulty of having more than one person respond to a consult request. Regardless of the ethics consultant versus subcommittee structure, peer-review of ethics consultations should routinely occur at a subsequent ethics committee meeting to ensure quality.

Childhood Obesity and Parental Responsibility (From BH509: When Life Makes You Sick: Ethics and the Social Determinants of Health, Erika Blacksher, instructor) Joe lives with his mother and aunt and her two children, both under age 6, in a small duplex south of Seattle. Average height for his age, Joe is 9 years old and weighs 184 pounds. His two cousins are not obese, but they are overweight. The mothers of the family work a number of jobs to make ends meet. They do not have much money for material things or extra curricular activities, and increasing gang activity in the area has made them reluctant to let the children play on their own in the nearby park and soccer field. What the family does not do in the way of 'fun', they make up for with food. The school nurse has brought Joe's obesity to his mother's attention twice, each time suggesting several approaches to helping him lose weight. They include buying healthier foods and healthier cooking. Joe's mom has been slow to take up these practices because the foods are more expensive than what she usually buys, the recipes seem strange to her, and she suspects they will not taste very good to Joe or her niece and nephew, who she often also cooks for. At a recent pediatric check up Joe's physician cautions the mother that she really needs to get Joe's weight under control. He is developing elevated blood sugar and has experienced difficulty breathing in his sleep. If she does not take action, he says he might be obligated to alert child protective services. As childhood obesity has become an epidemic problem in the United States, such cases increasingly are being reported. The rate of childhood obesity has more than tripled in the last three decades with serious short- and long-term health consequences. Obese children are significantly more likely to experience a wide range of poor health outcomes, including type II diabetes, cardiovascular conditions, asthma, sleep disordered breathing, anomalies in foot structure, low self-esteem, depression, and high-risk behaviors. Studies suggest that as many as half of all obese children remain obese as adults, leading to further health risks-elevated risk of heart disease, stroke, diabetes, osteoporosis, lower-body disability, some types of cancer, and premature mortality in general. Public health leaders emphasize structural and environmental interventions to remedy the U.S. obesity epidemic, but even they acknowledge the role parents play in contributing to children's healthfulness, including normal weight (Frieden et al. 2010). Increasingly, state intervention in severe cases of childhood obesity is being called for under certain circumstances (Murtagh, Ludwig 2011). Are clinicians obligated to report parents of obese children to child protective services?

Although parents have significant discretion to rear their children according to their own values and practices, society may justifiably intervene when it deems parental behaviors expose their children to serious harm. This "harm principle," discussed above, is often appealed to as the basis for intervention (Murtagh, Ludwig2011). There have been legal cases in which a child has been removed from the home in cases where parental practices are judged to have contributed severe morbid obesity, in states that include California, Indiana, Iowa, New Mexico, and New York. Similar cases have also been reported in the United Kingdom. Commentators on the subject, however, urge caution, as removing a child from the home can seriously harm a child in other ways (Black et al. 2011). Commentators encourage using intermediate options, such as in home support services, parent training, and financial assistance, and seeking a second medical opinion before any action is taken.

What is participation in health?

Recognition in this sense lends ethical support to a core value of public health-the notion of participation in health. Participation has long been a basic principle of public health practice and an essential component of health equity (Commission on the Social Determinants of Health 2008). Participation in health is variously defined but generally the idea refers to a collaborative process that equitably engages community members, organizational representatives, researchers, among others in knowledge creation and social change (Rifkin 1986; Robertson, Minkler 1994; Wallerstein, Bernstein 1994; Israel et al. 1998). Participatory processes are defended on grounds that the research and action that result will be more effective, fulfill an obligation of procedural justice, and produce more equitable outcomes (Wallerstein, Duran 2010; Dankwa-Mullen et al. 2010; Wallerstein 2006; Daniels, Sabin 1998). They may also protect non-dominant communities from health interventions that further marginalize and stigmatize them (Blacksher 2012) and may go some distance in cultivating trust between research institutions and community partners .

What cost-burden does the neonatal intensive care unit (specifically extremely low birthweight infants) place on the health care system?

As of 2001, newborn care cost the healthcare system over $12 billion. Of that, preterm or low birthweight infants generated 47% of this healthcare expense. On average, a preterm infant's hospital stay costs $15,000/patient, with extremely premature infants (<28 weeks) costing an average $65,000/patient (information adapted from Russell et al 2007). Although these costs are individually high, only 1-2% of preterm infants are born <28 weeks gestation. The largest financial burden to the healthcare system is generated by late-preterm infants (born at 34+ weeks gestation). In fact, if healthcare were rationed to exclude medical care to infants born <27 weeks gestation, it would only save the healthcare system 6% of its annual newborn care expenditure.

How should the patient's quality of life be considered?

CPR might appear to lack potential benefit when the patient's quality of life is so poor that no meaningful survival is expected even if CPR were successful at restoring circulatory stability. However, quality of life should be used with caution in determining whether or not CPR is indicated or has the potential to provide medical benefit, for there is substantial evidence that patients with chronic conditions often rate their quality of life much higher than would healthy people. Quality of life assessments have most credibility when the patient's values, preferences, and statements inform such assessments.

What are some examples of a decision that places a child a significant risk of serious harm?

Childhood vaccination provides an example of the kinds of factors that must be weighed in making this determination. While most physicians believe it is in a child's best interest to receive the routine childhood vaccinations and therefore recommend them to parents, they do not generally legally challenge parents who choose not to vaccinate their children. This is because in a well-vaccinated community the risk of contracting the vaccine-preventable illness and suffering harmful consequences from the infection are quite small. However, this calculation might shift if a clinician is faced with an unvaccinated child who has suffered a puncture would from a dirty nail. In the latter case, the risk of tetanus (a serious and almost always fatal disease if not prevented) has become significant, and the provider would be justified in seeking the power of the State (through a court order or involvement of child protective services) to assure that the child receives the vaccination and treatment necessary to prevent tetanus in a high risk situation.

Should children be involved in medical decisions even though their parents have final authority to make those decisions?

Children with the developmental ability to understand what is happening to them should be allowed to participate in discussions about their care. As children develop the capacity to make decisions for themselves, they should be given a voice in medical decisions. Most children and adolescents lack full capacity to make complex medical decisions, however, and final authority to make medical decisions will usually remain with their parents.

What is the role of confidentiality?

Confidentiality provides the foundation for the physician-patient relationship. In order to make accurate diagnoses and provide optimal treatment recommendations, the physician must have relevant information about the patient's illness or injury. This may require the discussion of sensitive information, which would be embarrassing or harmful if it were known to other persons. The promise of confidentiality permits the patient to trust that information revealed to the physician will not be further disseminated. The expectation of confidentiality derives from the public oath which the physician has taken, and from the accepted code of professional ethics. The physician's duty to maintain confidentiality extends from respect for the patient's autonomy.

Do patients want to know the truth about their condition?

Contrary to what many physicians have thought in the past, a number of studies have demonstrated that patients do want their physicians to tell them the truth about diagnosis, prognosis, and therapy. For instance, 90% of patients surveyed said they would want to be told of a diagnosis of cancer or Alzheimer's disease. Similarly, a number of studies of physician attitudes reveal support for truthful disclosure. For example, whereas in 1961 only 10% of physicians surveyed believed it was correct to tell a patient of a fatal cancer diagnosis, by 1979 97% felt that such disclosure was correct.

Defining Direct Medical Benefit

Determining the potential for direct medical benefit can be challenging, especially when there is great uncertainty in outcome. One approach to defining benefit examines the probability of an intervention leading to a desirable outcome. Outcomes following CPR have been evaluated in a wide variety of clinical situations. In general, survival rates in adults following in-hospital cardiac arrest range from 8-39% with favorable neurological outcomes in 7-14% of survivors (Meaney et al., 2010). In children, the survival rate following in-hospital cardiac arrest is closer to 27% with a favorable neurological outcome in up to one third of survivors (AHA, 2010). Out of hospital arrest is less successful, with survival rates in adults ranging from 7-14% and in infants and children approximately 3-9% (Meaney et al., 2010; Garza et al., 2009). In general, these statistics represent the population as a whole and do not necessarily reflect the chance of survival for an individual patient. Hence, multiple factors, including both the distal and proximal causes for cardiopulmonary arrest, must be considered to determine whether or not CPR has the potential to promote survival (Bishop et al., 2010).

What are the ethical obligations of members of the interdisciplinary team in patient care?

Ethically, every member of the operating room team has separate obligations, or duties, toward patients, which are based on the provider's profession, scope of practice and individual skills. Team members also have ethical obligations to treat each other in a respectful and professional manner. Relationships between professionals on the multidisciplinary team are by their nature unequal ones. Different knowledge and experience in specific issues both ethically and legally imparts unequal responsibility and authority to those care providers with the most knowledge and experience to handle them. But also because of differences in training and experience, each member of the team brings different strengths. Team members need to work together in order to best utilize the expertise and insights of each member.

What if the family disagrees with the DNAR order?

Ethicists and physicians are divided over how to proceed if the family disagrees with the recommendation to forgo attempting CPR. If there is disagreement, every reasonable effort should be made to clarify questions and communicate the risks and potential benefits of CPR with the patient or family. In many cases, this conversation will lead to resolution of the conflict. However, in difficult cases, an ethics consultation can prove helpful.

Who becomes a member of an ethics committee or program?

Ethics committee members usually represent major clinical services and other stakeholders in health care delivery. Thus, it is not uncommon for committee members to include clinicians (physicians and nurses) from medicine, surgery, and psychiatry, social workers, chaplains, and community representatives. These committees may also have a quality improvement manager, an individual responsible for the education program at the facility, a lawyer, and at least one individual with advanced training in ethics. This latter representative can come from a number of disciplines, including philosophy, law, medicine, theology, and anthropology. All members of the ethics committee take responsibility for learning techniques of ethical analysis (see Bioethics Tools) and the arguments surrounding most of the ethically charged issues in clinical practice. Some ethics committees allow guests. These can include health sciences students, philosophy graduate students, physician trainees, facilitators, and patient representatives. Guests need to maintain the confidentiality of the information discussed at the meetings, often signing oaths to that effect. In order to integrate ethics throughout the institution and effect cultural change, ethics programs may include senior leaders in disparate services, such as fiscal, human resources, patient safety, quality improvement, and compliance and business integrity. At some institutions a special ethics council is formed to address ethical issues across the institution, providing a more integrated approach to addressing ethical issues.

Is it ever acceptable to not have a full informed consent?

Exceptions to full informed consent are: If the patient does not have decision-making capacity, such as a person with dementia, in which case a proxy, or surrogate decision-maker, must be found. (See Surrogate Decision-Making/No Legal Next of Kin.) A lack of decision-making capacity with inadequate time to find an appropriate proxy without harming the patient, such as a life-threatening emergency where the patient is not conscious When the patient has waived consent. When a competent patient designates a trusted loved-one to make treatment decisions for him or her. In some cultures, family members make treatment decisions on behalf of their loved-ones. Provided the patient consents to this arrangement and is assured that any questions about his/her medical care will be answered, the physician may seek consent from a family member in lieu of the patient.

What happens when medical therapy is indicated for one patient, yet contraindicated for the other?

Fetal care becomes problematic when what is required to benefit one member of the dyad will cause an unacceptable harm to the other. When a fetal condition poses no health threat to the mother, caring for the fetal patient will always carry some degree of risk to the mother, without direct therapeutic benefit for her. The ethical principles of beneficence ("be of benefit") and nonmaleficence ("do no harm") can come into conflict. Because the patients are biologically linked, both, or neither, must be treated alike. It would be unethical to recommend fetal therapy as if it were medically indicated for both patients. Still, given a recommendation for fetal therapy, pregnant women, in most cases, will consent to treatment which promotes fetal health. When pregnant women refuse therapy, physicians must remember that the ethical injunction against harming one patient in order to benefit another is virtually absolute.

What is a fiduciary relationship?

Fiduciary derives from the Latin word for "confidence" or "trust". The bond of trust between the patient and the physician is vital to the diagnostic and therapeutic process. It forms the basis for the physician-patient relationship. In order for the physician to make accurate diagnoses and provide optimal treatment recommendations, the patient must be able to communicate all relevant information about an illness or injury. Physicians are obliged to refrain from divulging confidential information. This duty is based on accepted codes of professional ethics, which recognize the special nature of physician-patient relationships.

I just saw another caregiver tell something to my patient in a really insensitive way. What should I do?

First, examine what happened and ask yourself why the encounter went badly. If you see the patient later, you might consider acknowledging it to the patient in a way that doesn't slander the insensitive caregiver ("I thought you looked upset when we were talking earlier and I just thought I should follow up on that--was something bothering you?")

Skip is a 50-year-old man with metastatic nonsmall cell lung cancer. He decided to try palliative chemotherapy because "otherwise I might just as well roll over and give up." After the first cycle of carboplatin and taxol, he requires hospitalization for fever and neutropenia (a complication of the chemotherapy). You stop by for a visit, and he says he feels terrible, wonders "if the chemo is worth all this", but that he's too scared to stop. How would you respond?

For metastatic nonsmall cell lung cancer, palliative chemotherapy is an intervention providing, on average, a small benefit at considerable toxicity (a consideration for the Medical Indications box in a Clinical Ethics 4-box analysis). Yet for a patient who is well informed, understands the benefits and burdens, and wishes to proceed, a trial of palliative chemotherapy is justified. However, now Skip is voicing concern: the most important thing to do is hear him out. Find out what he is worried about, how he rates his quality of life, and what his goals are. This information will help you sort out what is going through his mind and help you guide him to a decision that will be the best for him. As Skip thinks through his situation, ask him if he wants you to describe what would happen if he decides to have more chemotherapy, or stops his chemo and starts hospice care. Eventually you might ask him what a good death would be for him--he may not be able to answer immediately, but it might help him (and you) shape a care plan later. When you talk with Skip, keep in mind the goals for a decent death.

IRB review of research: full-board, expedited, and exempt reviews.

Full board: Clinical trials that expose subjects to more than minimal risk must be reviewed by the IRB at a convened meeting of the full Board. Minimal risk is defined as those risks and discomforts to which a person is commonly exposed in the ordinary course of daily life, including routine visits to physicians. It is important to remember that loss of privacy, i.e unauthorized disclosure of personal information, is considered to be a risk of research. As technology and social media strip away more and more of our privacy, ethicists debate what constitutes a reasonable expectation of privacy (for more information, please see Haigh & Jones, 2005). Expedited: Studies that qualify for expedited review are those that present no more than minimal risk to subjects, and involve only procedures commonly done in clinical settings, such as taking hair, saliva, excreta or small amounts of blood. A study that qualifies for expedited review is held to the same ethical standards of autonomy, beneficence and justice that are used in full board review, but the approval process may take less time. Exempt: Some research with humans can be designated as exempt from IRB review. Research that is part of a routine educational experience, or in which participants will be anonymous or effectively de-identified falls into this category and may be granted a certificate of exemption. The proposal must still be reviewed by a member of the IRB to assign exempt status, but the application process may be considerably shorter.

Is an intervention more likely to be futile if a patient is elderly?

Futility has no necessary correlation with a patient's age. What determines whether a treatment is futile is whether or not the treatment benefits the patient. In cases where evidence clearly shows that older patients have poorer outcomes than younger patients, age may be a reliable indicator of patient benefit, but it is benefit, not age, that supports a judgment of medical futility. For patients of all ages, health care professionals should advocate for medically beneficial care, and refrain from treatments that do not help the patient.

What is the difference between futility and rationing?

Futility refers to the benefit of a particular intervention for a particular patient. With futility, the central question is not, "How much money does this treatment cost?" or, "Who else might benefit from it?" but instead, "Does the intervention have any reasonable prospect of helping this patient?"

Who decides when a particular treatment is futile?

Generally the term medical futility applies when, based on medical data and professional experience, a treating health care provider determines that an intervention is no longer beneficial. Because health professionals may reasonably disagree about when an intervention is futile, all members of the health care team would ideally reach consensus. While physicians have the ethical authority to withhold or withdraw medically futile interventions, communicating with professional colleagues involved in a patient's care, and with patients and family, greatly improves the experience and outcome for all.

What are the physician's professional obligations with respect to CAM?

Given the number of Americans who use CAM in combination with allopathic medicine, the issue of interactions between therapies is a pressing one, particularly in the case of botanical compounds. The demonstration that garlic, used therapeutically in AIDS patients in the 1990s, interfered with the activity of HAART drugs provided a sobering reminder that herb-drug interactions should be studied as rigorously as drug-drug interactions. This is also an example of an "herb" also being a "food". The dose may be all that separates medicinal from food use, with potentially large ramifications, thus it is important to know more about food/nutrient-drug interactions. For the allopathic clinician, it is especially important to track the patient's use of all CAM therapies. Offhand dismissal or ridicule of CAM will impair communication and the therapeutic relationship with the patient; harmful herb-drug interactions could be missed or the patient may break entirely with the allopathic system. The CAM physician has obligations to adhere to the best practices of EBM, understanding that research in CAM lags practice. There is a danger for clinicians as well as for patients to believe the "if natural, then safe" fallacy, and there is danger in the tendency, however innocently formed, to use natural products as pharmaceuticals. The emerging Vitamin D story illustrates that overprescribing or overdosing a "natural" compound can have unintended negative consequences. CAM therapies provide useful tools to improve human health. As CAM research proceeds, the effectiveness or ineffectiveness of individual therapies will be established, and the effective CAM therapies will add significantly to the armamentarium of modern medicine. As concerns about the costs of health care grow, it will be particularly important to perform cost-benefit analyses on CAM therapies and determine whether they, used in informed combination with conventional care, might serve to reduce the economic burden of our national health care.

What obligations do clinicians have in these relationships?

Health care providers have professional and ethical obligations to care for these patients because of the fiduciary nature of the clinician-patient relationship. The provider has knowledge, influence, and power in the relationship, which entail special responsibilities. It may be difficult to see some of these patients as vulnerable, but without a clinician's help their vulnerability would only be compounded. While their medical, social, and psychiatric conditions may be complex, patients benefit both from a therapeutic relationship and from medical treatments and advice. Because clinicians often find these relationships exhausting and frustrating, they should identify trusted colleagues with whom they can share their frustrations, employ strategies that allow the best in the relationship to prevail, and use a team approach. Clinicians should also try to address or manage their own attitudes and behaviors that contribute to the problem, recognizing that the patient's behaviors and attitudes may not change. The clinicians should do everything they can to maintain a therapeutic relationship (even one that is not ideal), however in some circumstances they may need to transfer care to another provider. This transfer can be done without threatening the patient. The treating provider can help to maintain the patient's trust in the health system, while also setting clear boundaries.

How much information is considered "adequate"?

How do you know when you have provided enough information about a proposed intervention? Most of the literature and law in this area suggest one of three approaches: Reasonable physician standard: what would a typical physician say about this intervention? This standard allows the physician to determine what information is appropriate to disclose. However, this standard is often inadequate, since most research shows that the typical physician tells the patient very little. This standard is also generally considered inconsistent with the goals of informed consent, as the focus is on the physician rather than on what the patient needs to know. Reasonable patient standard: what would the average patient need to know in order to be an informed participant in the decision? This standard focuses on considering what a typical patient would need to know in order to understand the decision at hand. Subjective standard: what would this particular patient need to know and understand in order to make an informed decision? This standard is the most challenging to incorporate into practice, since it requires tailoring information to each patient. Most states have legislation or legal cases that determine the required standard for informed consent. In the state of Washington, we use the "reasonable patient standard." The best approach to the question of how much information is enough is one that meets both your professional obligation to provide the best care and respects the patient as a person, with the right to a voice in health care decisions. (See also Truth-Telling and Law and Medicine.)

Other students have (unauthorized) access to last year's killer exam. Should I look at it?

In a survey of students from the late 1980's. 58% reported cheating at least once during medical school. There have been disincentives for reporting cheating, and perhaps a general sense that "this is just the way it is." On the contrary, cheating in class is an example of unprofessional behavior. It represents a lack of fairness, lack of integrity, and can foreshadow lying in other contexts during your medical training. As a member of a profession, you are accountable for your own behavior and for the behavior of your colleagues. The Assistant Dean for Student Affairs or the Medical Student Association (MSA) representative can field your concerns and help you develop a plan for confronting your classmates.

How do I decide whether to tell a patient about an error?

In general, even trivial medical errors should be disclosed to patients. Any decision to withhold information about mistakes requires ethical justification. If a physician believes there is justification for withholding information about medical error from a patient, his judgment should be reviewed by another physician and possibly by an institutional ethics committee. The physician should be prepared to publicly defend a decision to withhold information about a mistake from the patient.

What are the confidentiality standards regarding adolescents?

In many states adolescents may seek treatment without the permission of their parents for certain conditions, such as treatment for pregnancy, sexually transmitted infections, mental health concerns, and substance abuse. Familiarize yourself with state and local laws, as well as institutional policies, regarding adolescents and healthcare.

How can disagreements on the multidisciplinary team be handled?

In the best situations, disagreement leads to a more complete interprofessional discussion of the patient's care, resulting in a new consensus about the best course of action. The new consensus may require compromises from each individual. When members of a team cannot arrive at a consensus of what should be done, it may be helpful to consult other professionals who are not directly involved in the patient's care team for objective input. If the disagreement still cannot be resolved, another resource may be the hospital's ethics committee, which can listen to disagreements and help suggest solutions.

What is the primary justification for the existence of neonatal intensive care units?

Increased survival in all birthweight categories from extreme prematurity to term infants with complex congenital anomalies.

A 29-year-old woman had an obstetrical ultrasound at 33 weeks to follow-up a previous finding of a low-lying placenta. Although the placental location was now acceptable, the amniotic fluid index (AFI) was noted to be 8.9 cm. Subsequent monitoring remained reassuring until 38.5 weeks, when the AFI was 6 cm. The patient declined the recommendation to induce labor, and also refused to present for any further monitoring. She stated that she did not believe in medical interventions. Nevertheless, she continued with her prenatal visits. At 41 weeks, she submitted to a further AFI, which was found to be 1.8 cm. She and her husband continued to decline the recommendation for induced labor. Which ethical duty takes precedence, the duty to respect the patient's autonomous decision, or the duty to benefit a viable fetus? Is induction of labor a harmful intervention, subject to the principle of nonmaleficence?

Induction of labor at term is an intervention with demonstrated efficacy and carries low risk to the mother. In this case, it could prevent serious damage to a viable fetus. Informed discussion and persuasive efforts should be continued towards this goal. However, deliberate disregard of maternal refusal for therapy could constitute assault. So long as the fetus is attached to the pregnant woman, her body maintains its life, and bars access to it.

When should I refer to a patient's advance directive?

It is best to ask a patient early in his care if he has a living will or other form of advance directive. Not only does this information get included in the patient's chart, but by raising the issue, the patient has an opportunity to clarify his/her wishes with the care providers and loved ones. However, advance directives take effect only in situations where a patient is unable to participate directly in medical decision making. Appeals to living wills and surrogate decision makers are ethically and legally inappropriate when individuals remain competent to guide their own care. The assessment of decisional incapacity may be difficult at times, is thought to be a responsibility of most physicians, but sometimes may require a psychiatric evaluation. Some directives are written to apply only in particular clinical situations, such as when the patient has a "terminal" condition or an "incurable" illness. These ambiguous terms mean that directives must be interpreted by caregivers. Some more recent types of instructive directives have attempted to overcome this ambiguity by addressing specific interventions (e.g. blood transfusions or CPR) that are to be prohibited in specified clinical contexts.

Case 1: Jose is a 62-year-old man who just had a needle biopsy of the pancreas showing adenocarcinoma. You run into his brother in the hall, and he begs you not to tell Jose because the knowledge would kill him even faster. A family conference to discuss the prognosis is already scheduled for later that afternoon. How should you handle this?

It is common for family members to want to protect their loved ones from bad news, but this is not always what the patient himself would want. It would be reasonable to tell Jose's brother that withholding information can be very bad because it creates a climate of dishonesty between the patient and family and medical caregivers; also, that the only way for Jose to have a voice in the decision making is for him to understand the medical situation. Ask Jose how he wants to handle the information in front of the rest of the family, and allow for some family discussion time for this matter. In some cultures it is considered dangerous to talk about prognoses and to name illnesses (e.g., the Navajo). If you suspect a cultural issue it is better to find someone who knows how to handle the issue in a culturally sensitive way than to assume that you should simply refrain from providing medical information. For many invasive medical interventions which require a patient to critically weigh burdens and benefits, a patient will need to have some direct knowledge of their disease in Western terms in order to consider treatment options.

How should I interpret a patient's advance directive?

Living wills generally are written in ambiguous terms and demand interpretation by providers. Terms like "extraordinary means" and "unnaturally prolonging my life" need to be placed in context of the present patient's values in order to be meaningfully understood. The health care agent or a close family member often can help the care team reach an understanding about what the patient would have wanted. Of course, physician-patient dialogue is the best guide for developing a personalized advance directive.

Obstacles to discussing Spirituality with Patients

Some physicians find a number of reasons to avoid discussions revolving around the spiritual beliefs, needs and interests of their patients. Reasons for not opening this subject include the scarcity of time in office visits, lack of familiarity with the subject matter of spirituality, or the lack of knowledge and experience with the varieties of religious expressions in our pluralistic culture. Many admit to having had no training in managing such discussions. Others are wary of violating ethical and professional boundaries by appearing to impose their views on patients. Nonreligious physicians have expressed anxiety that a religious patient may ask them to pray. In such instances, one could invite the patient to speak the prayer while the physician joins in reverent silence. On the other hand, some physicians regularly incorporate spiritual history taking into the bio-psycho-social-spiritual interview, and others find opportunities where sharing their own beliefs or praying with a particular patient in special circumstances has a unique value to that patient. Certainly issues in modern medicine raise a host of value-laden questions such as whether or not to prolong life through artificial means, whether it is licit to shorten life through the use of pain medications in the provision of palliative care or to pursue "death with dignity" in the final weeks of one's life. These and a myriad of other questions have religious and spiritual significance for a wide spectrum of our society and deserve a sensitive dialogue with physicians who attend to patients facing these troubling issues. Often, such questions are initiated in doctor-patient discussions and may trigger a referral to the chaplain.

What about "macro-allocation" concerns?

Some situations involve what is called "macro-allocation," that is, broad policies to distribute resources across populations, as distinguished from "micro-allocation" decisions, such as in the above triage examples, to give priority to one patient over another. Many of these reasons for shortage are the result of deliberate decisions to ration. Even such shortages as vital organs result from social policies that favor voluntary donation over routine salvaging or a commercial market in organs. Other shortages result from broad social and cultural institutions: our country has left health care largely in the private sphere and the availability of care for individuals is conditioned by their ability to pay or their employment status, the scarcity of flu vaccine in a given year may result from budgetary decisions rather than an estimate of need in the population. The social "safety net" that acknowledges a moral duty to assure health care to those unable to pay is strengthened or weakened according to prevailing societal commitments. The theoretical ethical question is: can a fair and just way of allocating health care resources be devised? The practical ethical question is: can a fair and just allocation be actually implemented in a particular social, economic and medical climate?

Can patients refuse to undergo routine preventive health measures?

Sometimes there are preventive health interventions that provide minimal or no benefit to the individual, yet provide substantial collective benefit to the public's health. Immunization is a good example. If most of the people in a region are vaccinated, those who are not vaccinated are also protected from the contagious disease because the likelihood of an outbreak is low. This is known as herd or community immunity. Hence vaccination protects others, especially infants and those with compromised immunity (due to chemotherapy, for example). If a patient refuses a legally required immunization (e.g., in jurisdictions where immunization is legally mandated), this becomes a legal matter. This issue arises more often as an ethical (not necessarily a legal) matter, however. An effective response is to maintain a therapeutic alliance with a child's parent in order to address fears and concerns about vaccination and to gently persuade them of the importance and safety of vaccination (Opel 2012). If not legally mandated, an adequately informed refusal, expressing compelling personal or religious beliefs, may be respected.

Will the patient trust me if I am a student?

Students may feel uncertain about their role in patient care. Building trust requires honesty: students must be honest about their role, letting the patient know s/he is a physician-in-training. In some settings, an attending physician or resident can introduce the student to initiate a trusting relationship. In other settings, students may need to introduce themselves. One form of introduction would be "Hello, I am Mary Jones. I'm a third year medical student who is part of the team that will be caring for you during your hospitalization. I'd like to hear about what brought you into the hospital." (For further discussion of this issue, see Student Issues.) Many patients appreciate the opportunity to work with the student on the team. Students usually have more time to spend with a patient, listening to the patient's history and health concerns, and may become more aware of personal concerns than other team members. Patients notice and appreciate this extra attention.

What constitutes a fair distribution of health?

Systematic analyses of social justice and health equity are relatively recent (Marchand et al. 1998; Powers, Faden 2006; Asada 2007; Daniels 2008). Such analyses have taken up a number of basic questions, including: Are social inequalities in health unjust? Which inequalities are most urgent? What constitutes a fair distribution of health? A fundamental question concerns the place of social justice as an ethical construct in public health. Is social justice the moral foundation of public health? This view has its defenders (Powers, Faden 2006; Robertson 1998); however, it has not been a standard view. Rather, it is widely assumed that public health and health policy should aim to produce the most health benefits, or aggregate health, in a population (Brock 2000). Similarly, it has been suggested that maximizing benefits and removing harms "provide a prima facie warrant for many activities in pursuit of the goal of public health" (Childress et al. 2002). This utilitarian goal, which aims to maximize aggregate health, is distinct from the distributive goal to produce a fair distribution of health. Although public health activities and health policies may produce more health and reduce health inequalities, health initiatives very often reproduce or exacerbate disparities because, as already noted, better off people tend to benefit sooner and disproportionately from many health promotion interventions (Mechanic 2002; Frohlich 2008). One common strategy to address this problem is implementing interventions that target vulnerable subgroups. Such targeting may take a variety of forms, such as focused media campaigns, cash stipends, and social and health care services. But targeting also carries an ethical risk-the further marginalization and stigmatization of minority and socio-economically marginalized groups. As commentators have noted, the pursuit of social justice entails not only distributive goals, such as reducing health disparities, but also recognition goals (Fraser 2003, Young 1990, Anderson 1999). This demand of justice has been variously described but generally refers to a form of basic respect for persons that, at the very least, protects non-dominant groups from discrimination and marginalization based on group membership, such as race, ethnicity, gender, sexual orientation, nationality, social class, and so on. Racism and sexism are paradigmatic examples of transgression of basic respect in this sense. Recognition may be further elaborated to include the idea that all persons have an equal opportunity to participate as peers in public life and to have a voice in important public issues, particularly those that directly affect them (Fraser 2003, Anderson 1999).

General ethical principles applied to research with human subjects.

The Belmont Report, which provides the ethical foundation for research regulations and guides IRB deliberations, was generated by a federally commissioned group of scientists, physicians, ethicists, and philosophers and published in 1979. The three primary ethical principles cited in Belmont are: autonomy, beneficence, and justice. Autonomy refers to the right of an individual to determine what activities they will or will not participate in. Implicitly, full autonomy requires that an individual be able to understand what they are being asked to do, make a reasoned judgment about the effect participation will have on them, and make a choice to participate free from coercive influence. The cornerstone of protecting autonomy is the informed consent process, whereby an investigator provides a potential research participant with full disclosure about the nature of the study, the risks, benefits and alternatives, and an extended opportunity to ask questions before deciding whether or not to participate. Populations presumed to have diminished autonomy, by virtue of impaired cognition (for example, children, cognitively-impaired elderly, or mentally ill subjects) or of circumstance (for example prisoners or seriously ill people) are considered to be vulnerable populations. In some of these cases (children and prisoners) special safeguards to protect their autonomy are required by regulation. Beneficence refers to the obligation on the part of the investigator to maximize benefits for the individual participant and/or society, while minimizing risk of harm to the individual. Maximizing potential benefits is predicated on sound experimental design, thus research proposals must undergo rigorous scientific review before proceeding to the IRB for ethical review. An honest enumeration of reasonably anticipated risks must be followed by a thorough risk/benefit calculation. Justice demands equitable selection of participants, i.e., avoiding participant populations that may be unfairly coerced into participating, such as prisoners and institutionalized children. The principle of justice requires that those who undertake the burdens of research must be likely to benefit from the research, and is a principle often violated by the export of clinical trials to underdeveloped countries.

Angela is a 72-year-old woman with end stage congestive heart failure from coronary artery disease--she has had two myocardial infarctions. When her medical management is optimal, she is just able to take care of herself in her own apartment, but with any small decompensation, she ends up in the hospital. She comes in for a clinic visit, and her weight is up 2 kilograms and she is complaining of paroxysmal nocturnal dyspnea, even though she has been taking her meds as prescribed. Exasperated and discouraged, she asks, "Am I dying?". The cardiologist replies: "Well, no--this is all reversible." What would you say?

The SUPPORT study has shown us that the clinical course of dying from congestive heart failure is quite different from dying of lung cancer. Patients with lung cancer begin a visible, predictable decline several weeks before death that usually evident to experienced clinicians. Patients with congestive heart failure, however, experience periods of fairly good function alternating with decompensation right up until death, and the terminal event for these patients is often sudden. This pattern of decline is not usually labeled by patients or physicians as "dying." The unpredictable course has resulted in very few hospice referrals for patients with end-stage congestive heart failure. The best care plan in this situation would be based on a discussion with Angela about what kinds of contingency plans should be in place if she has a severe, possibly fatal decompensation (see Advance Care Planning). Some medical centers are developing Palliative Care or Comfort Care services to try to better match the needs of patients with less predictable end-stage illnesses.

A 28-year-old woman presents for diagnostic laparoscopy for pelvic pain. During laparoscopy, the surgeon announces that she intends to proceed to hysterectomy for multiple uterine myomata. The anesthesiologist then declares that he will "wake the patient up" rather than allow the surgeon to proceed, due to lack of consent for the procedure, and questionable medical necessity. Can the anesthesiologist "tell" the surgeon what to do? Who is in charge when two physicians on the team disagree?

The anesthesiologist can stop the surgery, and may even have an ethical obligation to the patient to do so, but should take such action only after discussing several issues with the surgeon. Is the surgery in fact included in the consent? If not, is the surgery medically necessary at this moment (i.e., would delay place the patient's life in significant danger) or can it be postponed until the patient can be awakened and asked for consent? If the surgery is not emergent, and there is no consent, the anesthesiologist is morally obliged to protect the patient's autonomy and right to give consent. Anesthesiologists have been also held legally liable for harm done to patients during elective surgery for which they did not consent, because the anesthesiologist renders the patient insensate and unable to protect themselves from unwanted intrusion. Often, in a case like this one, consensus can be obtained from the health care team, which in this case could consult the hospital legal counsel and the hospital ethics committee prior to proceeding.

Mrs. Franklin, an 81-year-old Alzheimer's patient hospitalized under your care has been asked to participate in a clinical trial testing a new drug designed to help improve memory. You were present when the clinical investigator obtained a signed informed consent from Mrs. Franklin a few days ago. However, when you visit Mrs. Franklin today and ask her if she is ready to begin the study tomorrow, she looks at you blankly and seems to have no idea what you are talking about. What should you do?

The competence of Mrs. Franklin to give an ethically valid informed consent is in doubt. You should contact the primary investigator to discuss Mrs. Franklin's participation in the trial. There may be a surrogate who can give consent for her participation if it is deemed to be in her best interests. Although she may be considered a vulnerable research subject because of her mental status, Mrs. Franklin does belong to the population the intervention is designed to assist, and the study may lead to beneficial interventions for Alzheimer's patients in the future. However, a careful balancing of risks and benefits should occur.

What will the ethics consultant do if I page her or him?

The consultant will usually ask you to specify the nature of the perceived ethical problem, and the question that you would like answered. She will meet with you and the other people involved in the situation. In clinical consultations she will review the medical record and meet with the patient (or surrogate decision maker) and family members. Often the consultant will arrange an interdisciplinary meeting to review the specifics of the case and to facilitate communication across disciplines or between clinicians and the patient (and/or the family). The consultant will write a note and attempt to answer the proposed question(s). In the Seattle area, the recommendations may be anchored to the 4 box analysis of relevant case information, utilization of principles of clinical ethics, rigorous analysis of similar and dissimilar cases, and supporting arguments and data from the literature. If definitive recommendations cannot be made because there is disagreement among the ethics consult team, a clear explication of the ethics standards (e.g., laws, consensus statements, policies), and analytic arguments (i.e., claims and counterclaims, such as reliance on established codes, consideration of short- and long-term consequences, analysis of comparable features of the case) will be presented in the consultant's note. If several ethically permissible options exist, the consultant will present them in a note including their justifications.

Is CPR always beneficial?

The general rule of attempting universal CPR needs careful consideration (Blinderman et al., 2012). Even though including patients and families in decisions regarding resuscitation respects patient autonomy, providing patients and families with accurate information regarding the risks and potential medical benefit of cardiopulmonary resuscitation is also critical. Under certain circumstances, CPR may not offer the patient direct clinical benefit, either because the resuscitation will not be successful or because surviving the resuscitation will lead to co-morbidities that will merely prolong suffering without reversing the underlying disease. Some physicians and ethicists define CPR under these circumstances as medically inappropriate or "futile" (Burns & Truog, 2007). Hence, evaluating both the proximal and distal causes of the cardiac arrest is important when determining the likelihood of successful resuscitation (Bishop et al., 2010; Blinderman et al. 2012). When CPR does not have the potential to provide direct medical benefit, physicians may be ethically justified in writing a DNAR order and forgoing resuscitation.

A 22-year-old woman in her first pregnancy with an unremarkable prenatal course presents with preterm labor at 28 weeks gestation. Her contractions were successfully stopped with terbutaline. Discharge planning was reviewed with her, and she was instructed to follow a regimen of bedrest and oral terbutaline. She reported that she did not intend to comply with these instructions. She believed that God would not allow her to labor unless it was time for the baby to deliver, and she indicated that He had communicated this to her. How can the physician ensure nonmaleficence towards the mother and still promote beneficence towards her fetus? Is the mother competent? Should maternal autonomy prevail over other ethical concerns?

The gestational age of this pregnancy places the fetus on the border of viability. Extensive, non-coercive discussions are essential to ensure that this patient understands the implications of refusing therapy. You may suggest that she invite her spiritual advisor to meet with both of you to talk together about her beliefs and the impact on her fetus. If her refusal persists, in light of her unconventional religious claims, a psychiatric consultation might be requested to evaluate her competency. If competency is documented, the ethical principle of nonmaleficence would support refraining from performing any unwanted interventions. If the patient is found to be incompetent, judicial intervention could be considered.

What is a "good death"? A medical perspective

The good death is not a familiar idea in American culture. Some experts in palliative care describe the United States as a "death-defying" culture, with a mass media that spotlights only youth and beauty. Yet public interest in care of the dying is currently high. The striking public interest in physician aid-in-dying is one obvious reason. But there are other reasons: over the past 100 years, there has been a epidemiologic shift in the reasons people die. In the pre-antibiotic era, people most often died young, of infectious diseases; now, thanks to medical technology, most Americans (and others with access to this technology) live much longer, to die of degenerative, neoplastic, and even man-made diseases. Finally, there is a marked public fear that a medical death, depicted in TV shows like "ER" as an unresponsive, uncommunicative body hooked up to an array of flashing monitors, represents an irresponsible use of technology and a dishonorable way to treat a person. Interestingly, contemporary medical literature contains little that might characterize what makes a death "good." Recently, a large, expensive empirical study of intensive care unit deaths suggested that medical care for a common type of in-hospital death is "bad" (the SUPPORT study, referenced below). In this study of dying patients, severe pain was common, decisions to withhold invasive treatments were made at the last minute, and physicians often had no knowledge of patient preferences not to have CPR. Even worse, an intervention designed to provide physicians with better prognostic information had no effect on medical decision making prior to death. While the SUPPORT authors did not actually describe these deaths as "bad," we could certainly agree that they were not "good deaths." In caring for a person who is dying, knowing what would make the experience of dying "good" is an important goal for physicians and other members of the care team. I find it doesn't take fancy techniques-you just need to be sincere and patient and interested. Listen more and talk less. Try asking something like, "Knowing that all of us have to think about dying at some point, what would be a good death for you?" What people choose when they think about a good death for themselves is often beyond what medicine can provide-for instance, an affirmation of love, a completion of important work, or a last visit with an important person. As a physician, I can't always make those things happen. But I can help the dying person get ready-and in this way, contribute to a death that is decent.

How has the physician-patient relationship evolved?

The historical model for the physician-patient relationship involved patient dependence on the physician's professional authority. Believing that the patient would benefit from the physician's actions, a paternalistic model of care developed. Patient's preferences were generally not elicited, and were over-ridden if they conflicted with the physician's convictions about appropriate care. During the second half of the twentieth century, the physician-patient relationship has evolved towards shared decision making. This model respects the patient as an autonomous agent with a right to hold views, to make choices, and to take actions based on personal values and beliefs. Patients are acknowledged to be entitled to weigh the benefits and risks of alternative treatments, including the alternative of no treatment, and to select the alternative that best promotes their own values.

History of Cardiopulmonary Resuscitation and Do Not Attempt Resuscitation Orders

The history of CPR and DNAR orders is extensively reviewed in the literature (Bishop et al., 2010; Burns et al., 2003). In the 1960s, CPR was initially performed by anesthesiologists on adults and children who suffered from witnessed cardiac arrest following reversible illnesses and injuries. Based on the success of this intervention, CPR became the standard of care for all etiologies of cardiopulmonary arrest and the universal presumptive consent to resuscitation evolved (Burns et al., 2003). However, in 1974, the American Heart Association (AHA) recognized that many patients who received CPR survived with significant morbidities and recommended that physicians document in the chart when CPR is not indicated after obtaining patient or surrogate consent (ibid). This documentation formally became known as the DNR order. Recent medical literature encourages reference to this documentation as do-not-attempt-resuscitation (DNAR) and allow a natural death (AND) based on the practical reality that performing CPR is an attempt to save life rather than a guarantee (Venneman et al., 2008).

A 17 year old young woman is diagnosed with acute lymphocytic leukemia. The patient and her family are practicing Jehovah's Witnesses. Based on their religious beliefs, the patient and her parents do not want the medical treatment to include any blood transfusions or blood products. All non-blood alternatives had been attempted or deemed inadequate. The standard of care would require the use of blood products, to which the patient and her parents will not consent. There is sub-optimal treatment available which does not include transfusion support that the patient and her parents are willing to consent to receive. The physicians estimate that the difference in receiving sub-optimal treatment is that the minor's chances for cure are probably diminished by at least 50%. Can the 17 year old patient be deemed sufficiently mature to make her own medical treatment choices? Who has authority to make this determination? Are the patient's parents, as her legally authorized surrogate decision-makers, entitled to make a choice for their daughter? If so, would the parents be bound to use a "substituted judgment" or a "best interest" standard when making a decision on behalf of their minor child?

The legal and clinical ethics parameters in Case 3 concern: (a) whether a minor can provide informed consent for her own treatment (which includes non-treatment) that may have fatal consequences; (b) what surrogate decision-making authority do parents have for a mature minor; and (c) the health care providers' ability to accept non-treatment choices that greatly diminish the likelihood of successful treatment. The patient's ability to direct her care is expressed in law as a liberty or privacy right and in clinical ethics as respect for patient autonomy. Case 3 implicates a number of clinical ethics and legal issues. Specific clinical ethics and legal issues: The 17 year old patient is a minor under state law. 26 A key question in this case is whether or not the patient should be treated as capable of providing her own informed consent. The treatment team held a series of meetings with the patient, her parents, and her younger sibling to discuss the patient's diagnosis, its implications and treatment availability. The patient was also separately counseled by medical staff to ascertain whether she was freely and voluntarily expressing her preferences or if she may have felt pressured by her family or church members. Jehovah's Witnesses have beliefs which forbid the acceptance of blood transfusions and blood products due to an interpretation of certain biblical text. With the exception of this treatment modality, Jehovah's Witnesses are generally willing to accept medical treatment. This belief is not generally shared by others in our society. As such, this belief may appear unacceptable and irrational to others, including health care providers. Even if the patient is deemed competent to make medical treatment decisions, if the patient rejects potentially life-saving treatment based on an uncommon belief, this may cause distress to the patient's health care providers who may view such an act as not being in the patient's best interest. The patient was capable of articulating her personal beliefs and preferences and was believed to be mature by the hospital staff. In some instances, physicians have documented clinical observations that support a conclusion that the minor was mature and capable of making medical decisions in light of the nature of the condition and treatment choices... This type of clinical determination of ability to provide mature reasoned decision-making for health care has been recognized in state law, e.g., in Washington State. The court has identified certain factors as relevant in considering whether a minor is "emancipated" including: age, maturity, intelligence, training, experience, economic independence, and freedom from parental control.27 Additional state laws have expanded the ability of minors to provide consent for particular types of health care, such as abortion, birth control, sexually transmitted diseases and mental health treatment. 28 In this particular situation, since the refusal of the blood transfusion had potential fatal consequences, from an institutional risk management perspective, the option of a court review and judicial determination of emancipation was a preferred choice. The hospital's legal counsel initiated a legal process to allow the patient and her family to request the local superior court for a court determination of emancipation so that the patient would be deemed an adult for making her own treatment choices. In this particular case, ultimately a court proceeding was held at the hospital. The patient, her parents, her sibling, church members of the family, the church's attorney, the treating physician, the hospital's attorney, and the judge were present. Testimony was taken and the judge also spoke with the patient in private (the judge later gave a summary of the conversation for the record). There was also evidence in the form of an affidavit signed by Children's Protective Services that this would not be a situation in which that state agency would file a petition and seek a court order for treatment of the minor. The physician supported that the patient was emancipated and should be permitted to make her own informed consent. The court entered an order of emancipation. The effect of this court order of emancipation put the minor patient on equal consent footing as an adult. Emancipation, in and of itself, does not alter the requirement that the patient provide informed consent, i.e. be able to understand and weigh the risks and benefits of the recommended medical treatment and other treatment options, including non-treatment. While this particular set of facts resolved with minimal or no conflict, other situations involving patients with differing social or religious beliefs regarding specific treatments may have greater conflict.

What are the elements of full informed consent?

The most important goal of informed consent is that the patient has an opportunity to be an informed participant in her health care decisions. It is generally accepted that informed consent includes a discussion of the following elements: The nature of the decision/procedure Reasonable alternatives to the proposed intervention The relevant risks, benefits, and uncertainties related to each alternative Assessment of patient understanding The acceptance of the intervention by the patient In order for the patient's consent to be valid, she must be considered competent to make the decision at hand and her consent must be voluntary. It is easy for coercive situations to arise in medicine. Patients often feel powerless and vulnerable. To encourage voluntariness, the physician can make clear to the patient that she is participating in a decision-making process, not merely signing a form. With this understanding, the informed consent process should be seen as an invitation for the patient to participate in health care decisions. The physician is also generally obligated to provide a recommendation and share his reasoning process with the patient. Comprehension on the part of the patient is equally as important as the information provided. Consequently, the discussion should be carried on in layperson's terms and the patient's understanding should be assessed along the way. Basic or simple consent entails letting the patient know what you would like to do; giving basic information about the procedure; and ensuring that the patient assents or consents to the intervention. Assent refers to a patient's willing acceptance of a treatment, intervention, or clinical care. Basic consent is appropriate, for example, when drawing blood in a patient who has given blood before. Sometimes consent to the procedure is implied (e.g. the patient came in to have blood drawn), but an explanation of the elements of the procedure remain necessary. Decisions that merit this sort of basic informed consent process require a low-level of patient involvement because there is a high-level of community consensus that the treatment being offered is the only or best option and/or there is low risk involved in the treatment If a patient does not consent under the paradigm of basic consent, then a fuller informed consent discussion is warranted.

What does the duty of confidentiality require?

The obligation of confidentiality prohibits the health care provider from disclosing information about the patient's case to others without permission and encourages the providers and health care systems to take precautions to ensure that only authorized access occurs. Appropriate care often requires that information about patients be discussed among members of a health care team; all team members have authorized access to confidential information about the patients they care for and assume the duty of protecting that information from others who do not have access. Electronic medical records can pose challenges to confidentiality. In accordance with the Health Information Portability and Accountability Act of 1997 (HIPAA), institutions are required to have policies to protect the privacy of patients' electronic information, including procedures for computer access and security.

Is there such a thing as presumed/implied consent?

The patient's consent should only be "presumed," rather than obtained, in emergency situations when the patient is unconscious or incompetent and no surrogate decision maker is available, and the emergency interventions will prevent death or disability. In general, the patient's presence in the hospital ward, ICU or clinic does not represent implied consent to all treatment and procedures. The patient's wishes and values may be quite different from the values of the physician. While the principle of respect for person obligates you to do your best to include the patient in the health care decisions that affect her life and body, the principle of beneficence may require you to act on the patient's behalf when her life is at stake.

A patient who has coronary artery disease and congestive heart failure shows his physician his advance directive that states he wants to receive cardiopulmonary resuscitation and other forms of life-sustaining treatment. What should the doctor say to the patient in response to this?

The patient's expression of a preference should be explored to understand its origins. It is possible that the patient believes, based on television shows, that CPR is usually effective. If this is the case, the doctor should educate the patient about the near futility of CPR under these circumstances. However, the physician may learn that the patient has deeply held beliefs that suggest that not trying to live is tantamount to committing suicide which he perceives as morally wrong. In this situation, the doctor might want to ask the patient to explore this further with him and perhaps the chaplain.

A 3-year-old presents to the emergency department. She was diagnosed with pyelonephritis by her physician yesterday, treated with an intramuscular injection of antibiotic and sent home on an oral antibiotic. She is vomiting today and unable to keep the antibiotic down. As you prepare to admit her, you feel she should have been admitted yesterday. Should you tell the parents that their physician made a mistake? How should you handle this disagreement?

The practice of medicine is not an exact science. Frequently physicians will disagree about what constitutes the most appropriate management in a given case. Often these are legitimate disagreements with more than one acceptable course of action. Simply because you would have managed a patient differently does not mean the other physician made a mistake. In this case, you may wish to discuss the case with the other physician and explain why you manage children with pyelonephritis differently. However, in situations where standard practice varies, the parents should not be told that a mistake has been made.

Designing ethical research studies.

The primary concern of the investigator should be the safety of the research participant. Protecting subject safety requires the investigator to use all available information to identify potential risks to the subject, to establish means of minimizing those risks, and to continually monitor the ongoing research for adverse events experienced by subjects. The investigator must be prepared to stop the study if serious unanticipated risks are manifest. The scientific investigator must obtain informed consent from each research participant. This should be obtained in writing (although oral consents are sometimes acceptable) after the participant has had the opportunity to carefully consider the risks and benefits and to ask any pertinent questions. Informed consent should be seen as an ongoing process, not a single event or a mere formality. There are regulations and guidance documents that govern exceptions to the requirement to obtain informed consent, for example in cases of emergency or if the subject is unconscious and thereby unable to give consent. The investigator must protect the subjects' privacy and confidentiality. Researchers must have mechanisms in place to prevent the disclosure of, or unauthorized access to, data that can be linked to a subject's individual identity. The investigator must consider how adverse events will be handled. In particular, it must be established a priori who will provide care for a participant injured in a study and who will pay for that care. The investigator must strive for clinical equipoise. A true null hypothesis should exist at the onset regarding the outcome of the trial, that is, if a new intervention is being tested against the currently accepted treatment, the investigator should be genuinely uncertain which approach is superior.

How can respect for persons involve a spiritual perspective?

The principle of respect for persons undergirds our duties as health care professionals to treat all persons fairly, to safeguard the autonomy of patients, and to limit the risks of harm by calculating the burdens and benefits of the care plan. Such respect for persons is a guiding principle of the healing profession and flows from the profession's fundamental ethical commitment in serving the sick and injured. Although respect for persons is a secular principle it may have a deeper meaning for physicians who hold a religious perspective as in most religions, the patient is seen as a part of God's creation, or as created in the likeness of God (imago Dei). Likewise, it is reinforced in religious hospitals whose mission is to care for persons as "children of God," regardless of socio-economic standing. Such caring implies care for the whole person, physically, emotionally, socially and spiritually. Thus, our concern for the patient's spiritual well-being is an integral part of health care and is a way of showing respect for the person who comes as a patient-supplicant.

Mr. S is a 70-year-old man with end-stage COPD, admitted last month with pneumonia. His course was complicated by respiratory failure needing mechanical ventilation, and multiple efforts to wean him have been unsuccessful. Awake and alert, he now communicates through written notes that he wants the ventilator taken off. What do you think his prognosis is? What else do you want to know before making this decision? If he is competent, will you honor his request?

The prognosis of full recovery from long-term mechanical ventilation is poor, particularly in patients like Mr. S with minimal pulmonary reserve. The approach to his request should start with an evaluation of his decision making capacity. Even though he is awake and alert, you should carefully probe the reasons for his request, with particular attention to making sure he understands the consequences of his decision. If you're concerned about depression or other mental illness affecting his thinking about this decision, you might request a psychiatry consultation. You should ask Mr. S if he's discussed this with his spouse or family. If his decision making capacity is intact, you should honor his request.

What does it mean to be a member of a profession?

The words "profession" and "professional" come from the Latin word "professio," which means a public declaration with the force of a promise. Professions are groups which declare in a public way that their members promise to act in certain ways and that the group and the society may discipline those who fail to do so. The profession presents itself to society as a social benefit and society accepts the profession, expecting it to serve some important social goal. The profession usually issues a code of ethics stating the standards by which its members can be judged. The traditional professions are medicine, law, education and clergy. The marks of a profession are: Competence in a specialized body of knowledge and skill; An acknowledgment of specific duties and responsibilities toward the individuals it serves and toward society; The right to train, admit, discipline and dismiss its members for failure to sustain competence or observe the duties and responsibilities.

What if the truth could be harmful?

There are many physicians who worry about the harmful effects of disclosing too much information to patients. Assuming that such disclosure is done with appropriate sensitivity and tact, there is little empirical evidence to support such a fear. If the physician has some compelling reason to think that disclosure would create a real and predictable harmful effect on the patient, it may be justified to withhold truthful information.

Under what circumstances can minors make medical decisions for themselves?

There are three situations in which minors (those who have not reached the age of majority in their state of residence) have the legal authority to make decisions about their health care. First, every state has emancipated minor laws which designate minors who meet certain criteria as having the authority to make decisions (including medical decisions) for themselves. Although emancipated minor laws vary from state to state, most states recognize an emancipated minor as a person who meets one of the following criteria: Economically self-supporting and not living at home Married A parent On active duty in the armed services Second, most states recognize some minors as sufficiently mature to make medical decisions on their own behalf. A determination that a minor is mature usually requires that the minor be older than 14 years of age and have demonstrated a level of understanding and decision-making ability that approximates that of an adult. While some states allow physicians to make this determination, most require a judicial determination of mature minor status. Third, all states make condition-specific exceptions to the requirement of parental consent. These laws may allow an adolescent to seek treatment without parental consent for sexually transmitted diseases, pregnancy, contraception, psychiatric disorders, and drug or alcohol abuse.

What can hinder physician-patient communication?

There may be many barriers to effective physician-patient communication. Patients may feel that they are wasting the physician's valuable time; omit details of their history which they deem unimportant; be embarrassed to mention things they think will place them in an unfavorable light; not understand medical terminology; or believe the physician has not really listened and, therefore, does not have the information needed to make good treatment decisions. Several approaches can be used to facilitate open communication with a patient. Physicians should: sit down attend to patient comfort establish eye contact listen without interrupting show attention with nonverbal cues, such as nodding allow silences while patients search for words acknowledge and legitimize feelings explain and reassure during examinations ask explicitly if there are other areas of concern

Health Care Disparities and Priority for Treatment (From BH509: When Life Makes You Sick: Ethics and the Social Determinants of Health, Erika Blacksher, instructor) Samuel Banks is a 54-year-old accountant with a college degree and health insurance. He was the first of four children and a first generation college graduate. Having worked hard all of his life, Mr. Banks has been exposed to a variety of chronic stresses, including racism. As an African American growing up in the southeast United States, Mr. Banks was exposed to a steady stream of slights and insults as well as institutional forms of racism. Despite these challenges, he had been healthy much of his life even though he put on an extra 25 pounds in his late 40s. Over this past year he began to have heart issues. Initially, he thought he was just having heartburn, but the discomfort persisted. Mr. Banks visited his physician, who was new to him because his employer had just switched insurers. He had waited an hour in the waiting room and was agitated by the time he got to see Dr. Susan Mott. The visit did not last long. Dr. Mott was polite but hurried, facing a full waiting room of patients, and Mr. Banks perceived the doctor as rushed and rude. He left Dr. Mott's office frustrated and determined to find a new physician. Before he could find one, he had a heart attack. He was rushed to the emergency room where he received medical treatment that included a physical exam, ECG, chewable aspirin, supplemental oxygen, nitroglycerin, and monitoring through the night. What he needed and did not receive, however, was coronary artery bypass graft surgery (CABG). This is not an uncommon experience for minorities. Members of racial and ethnic groups receive less and lower quality health care than their white counterparts. The Institute of Medicine report, Unequal Treatment: Confronting Racial and Ethnic Disparities in Health Care documents the extent of these disparities in health care and their potential sources (Geiger HJ et al. 2003). Many people argue that health care disparities constitute an injustice. The argument can be made on a number of moral grounds, including the idea that people should be treated equally, regardless of race or ethnicity, receiving treatment only on the basis of medical need. Some commentators, however, have raised the question of whether disadvantaged patients should be given priority at the point of care. Should the health care system prioritize care for members of disadvantaged groups over that of other patients? Should clinicians attempt to redress injustices in the social determinants of health by prioritizing the disadvantaged when they enter the health care system?

These allocation questions have received scant attention in bioethics, despite considerable attention to other resource allocation problems (Daniels 1994). There are arguments for and against giving priority to socially disadvantaged patients at the point of treatment. Some of the reasons in support of prioritizing members of socially disadvantaged groups include the view that it would constitute a form of compensation for the unjust disadvantages, including poor health, that poor and minority groups suffer, and, in doing so, promote social cohesion; and that better off groups are to some degree responsible for the conditions of worst off groups, given their contribution to the institutions and policies that unfairly distribute the social determinants (Hurst 2009). Reasons against priority include the view widely held in medicine that social attributes should not be taken into consideration within medicine, that everyone should be treated equally, based only on medical need, and that doing otherwise would demonstrate disregard and disrespect for the needs of those who are better off; and that such a practice would alienate more powerful and privileged groups and risk strong support for health system funding. Another important consideration is that disparities in health should be addressed at their source, which many locate in the non-medical determinants of health

M.S. is a married 35-year-old pregnant childless woman who has lost four previous pregnancies between 16 and 23 weeks gestation. She currently has reached 23 weeks and 3 days of gestation, her fetus is seemingly healthy, and has an estimated weight of 550 grams (+/-1.2 lbs). She has ruptured her bag of waters and is now having labor that seems unstoppable with tocolytics. Delivery seems inevitable. What are the management options and who decides what form of care should be instituted following delivery?

This gestational age and estimated birth weight represent the "gray zone" in terms of viability vs. non-viability. Accordingly, the parents have a choice to make. They can choose a palliative care mode of treatment (with non-survival being a virtual certainty) or alternatively, assisted ventilation, pressors, antibiotics, parenteral nutrition, etc. The role of the physician is to provide continuity, regularly updated information, and guide the parents through the decision-making process throughout the infant's clinical course, whatever it may be. This situation 40 years ago would have presented no ethical dilemma. Indeed, the 1972 Roe v. Wade Supreme Court case defined the limit of viability as 28 weeks gestation. Any form of aggressive care involving newborn infants below this gestational was thought to be medically futile. Today, however, aggressive measures at birth are sometimes initiated with a modest degree of success achieved in terms of promoting survival (nearing 50% for 23-24 week infants). Notably, survival is accompanied by a long stay in the hospital following delivery (3-4 months), enormous costs (see above), considerable suffering, and morbidity (in at least 50% of the cases there is significant handicap).

Forced Treatment for Multidrug-Resistant Tuberculosis MW is a 33-year-old man with multidrug-resistant tuberculosis (MDR-TB). He is homeless, and has a pattern of missing many of his scheduled clinic visits. Upon starting a multi-drug regimen for his condition, MW initially comes to his scheduled clinic visits, but after a few weeks begins missing them. The provider contacts the social work case manager, who arranges supervised drug administration (also known as "directly observed therapy"). Nevertheless, MW often cannot be found and this approach is deemed to be failing. Should MW be forced into treatment against his will?

This is a case in which the health of the public is clearly and seriously threatened. Multidrug-resistant tuberculosis has the potential of causing substantial morbidity and mortality for the population, particularly in large urban areas. Thus the need for the individual patient to be treated for the good of the public is high. Similarly, the patient himself stands to benefit from the treatment. Ordinarily, patients have the right to refuse potentially beneficial treatment, provided they are competent and make an informed decision to do so. The tension created in this case is that the patient's refusal to follow the medication regimen puts others at substantial risk of harm. Hence it may be justifiable to compromise his autonomy to protect the health of others. In such cases, every effort should be exhausted to enlist the patient's cooperation with the medical regimen. Interventions such as directly observed therapy are often effective ways to achieve the desired result without compromising the patient's autonomy. Failing this, it would be justifiable to seek court permission to confine and treat the patient against his will. In the legal process that ensues, considerations will include the magnitude of harm, the degree to which specific individuals are exposed to harm, and the probability of harm.

Can medical treatment ever be provided against a patient's will?

This is a controversial area in law, and the law varies by state. The ethical justification for treatment of a patient against his will is based on balancing the risk to the public versus respecting the patient's personal freedom (see discussion of the harm principle above). If the magnitude of risk to the public is great, many states allow for involuntary treatment. Health care providers have obligations, however, to use the least restrictive means to achieve the therapeutic goal. An involuntary hospital stay is the most restrictive, necessary only when there is high risk of infecting others as well as evidence of non-adherence to outpatient treatment. A less restrictive approach utilizes directly observed therapy, in which a designated provider or case worker observes the patient taking his medications at home or in another outpatient setting. Case 2 below discusses this question in the context of a patient with active multi-drug resistant pulmonary tuberculosis.

One of your clinic patients is a 35-year-old man with AIDS on Medicare who is an active intravenous drug user. He uses heroin and cocaine, but he never shares needles and is reliably present at all his clinic visits. He admits that he is often unable to take his medicines regularly when he is using drugs. He is asking about antiretroviral therapy with protease inhibitors. You have just read that HIV viral resistance to protease inhibitors occurs rapidly when patients are unable to take their medicines reliably. Should you prescribe protease inhibitors to this patient?

This is a difficult and ongoing debate in the care of patients with HIV. Protease inhibitors used in combination with nucleoside analogues have proven a powerful weapon in the fight against HIV. The problem of resistance is a real concern in a patient who cannot take his medicines reliably. Many public health advocates feel that these medicines should not be offered to patients who are admittedly noncompliant because they would be creating resistant clones of virus which could then be passed on to others, or make the individual unable to benefit later if they were able to become compliant. They also argue that the cost of these medications on the health care system is so extreme that they should only be used by those who can fully benefit from them. Others argue the principle of justice which espouses equitable distribution of resources amongst all available people in need, and if the patient wants the medications he should have equal access to them. There is no answer to this debate at this time. The only clear principle that should be followed here is that of non-abandonment. Whatever your choice is with the patient, the physician's responsibility is to remain available to the patient and continue an ongoing therapeutic relationship and encourage him with information and guidance about his HIV disease and issues of addiction.

A 55-year-old man has a 3-month history of chest pain and fainting spells. You feel his symptoms merit cardiac catheterization. You explain the risks and potential benefits to him, and include your assessment of his likely prognosis without the intervention. He is able to demonstrate that he understands all of this, but refuses the intervention. Can he do that, legally? Should you leave it at that?

This patient understands what is at stake with his treatment refusal. As he is competent to make this decision, you have a duty to respect his choice. However, you should also be sure to explore his reasons for refusing treatment and continue to discuss your recommendations. A treatment refusal should be honored, but it should also not be treated as the end of a discussion.

A 64-year-old woman with MS is hospitalized. The team feels she may need to be placed on a feeding tube soon to assure adequate nourishment. They ask the patient about this in the morning and she agrees. However, in the evening (before the tube has been placed), the patient becomes disoriented and seems confused about her decision to have the feeding tube placed. She tells the team she doesn't want it in. They revisit the question in the morning, when the patient is again lucid. Unable to recall her state of mind from the previous evening, the patient again agrees to the procedure. Is this patient competent to decide? Which preference should be honored?

This patient's underlying disease is impairing her decision-making capacity. If her wishes are consistent during her lucid periods, this choice may be considered her real preference and followed accordingly. However, as her decision-making capacity is questionable, getting a surrogate decision maker involved can help determine what her real wishes are.

How do you know when someone is dying?

This question is not as simple as it might sound. The SUPPORT study demonstrated that even for patients with a high probability of dying, it is still difficult for a clinician to predict that a particular patient is about to die. Thus it may be more useful for clinicians to give up relying on their predictive skills, and look at the common clinical paths (or trajectories) taken by dying patients, and design medical care that includes "contingency plans" for clinical problems that a person with incurable lung cancer (for example) is likely to experience. Such contingency plans might include advance directives and perhaps DNAR orders, as well as lines such as: "You will probably die from this, although we can't predict exactly when. What is really important for you in the time you have left?"

What about obtaining court orders to force pregnant women to comply?

US courts have demonstrated a willingness to force treatment on a mother for the sake of her viable fetus. However, the decisions of lower-court judges do not necessarily reflect settled law. Of the two court orders that have been reviewed by the highest courts of their respective states, one was upheld and the other was overruled. The use of court orders to force treatment on pregnant women raises many ethical concerns. Court orders force pregnant women to forfeit their autonomy in ways not required of competent men or nonpregnant women. There is an inconsistency in allowing competent adults to refuse therapy in all cases but pregnancy. Hospital administrators, lawyers and judges have little warning of impending conflicts and little time for deliberation; this time pressure makes it unlikely that pregnant women will have adequate legal representation. Furthermore, forced obstetrical interventions have the potential to adversely affect the physician-patient relationship. The American College of Obstetrics and Gynecology has stated that "Obstetricians should refrain from performing procedures that are unwanted by pregnant woman....The use of the courts to resolve these conflicts is almost never warranted."

When is it appropriate for a physician to recommend a specific course of action or override patient preferences?

Under certain conditions, a physician should strongly encourage specific actions. When there is a high likelihood of harm without therapy, and treatment carries little risk, the physician should attempt, without coercion or manipulation, to persuade the patient of the harmful nature of choosing to avoid treatment. Court orders may be invoked to override a patient's preferences. However, such disregard for the patient's right to noninterference is rarely indicated. Court orders may have a role in the case of a minor; during pregnancy; if harm is threatened towards oneself or others; in the context of cognitive or psychological impairment; or when the patient is a sole surviving parent of dependent children. However, the use of such compulsory powers is inherently time-limited, and often alienates the patient, making him less likely to comply once he is no longer subject to the sanctions.

Who should I approach for advance care planning?

Unlike health promotional activities that are targeted to select populations based on cost-effectiveness research, asking about interest in advance directives and advance care planning has been recommended for newly admitted patients to health care organizations. This is recommended by the Patient Self-Determination Act of 1991 and The Joint Commission, and it has led to including a standard set of questions at hospital admission. When not systematically incorporated into a routine we often think of initiating advance care planning with patients who are at higher risk for decisional incapacity. If you practice in a setting that does not require routine advance care planning discussions with patients (e.g., on hospital admission, biyearly), you should consider having a conversation about this with patients with the following conditions: At risk for strokes (e.g., those with hypertension) Early dementia Engaging in risky behavior that is associated with head trauma and coma (e.g., riding a motorcycle without a helmet, riding in cars without seat belts) Debility/frailty (e.g., those at risk for delirium associated with an acute illness) Severe, recurrent psychiatric illnesses Chronic progressive and terminal illness

You are the ICU attending physician taking care of a 40-year-old gay man with AIDS who is intubated with his third bout of pneumocystis pneumonia. His condition is worsening steadily and he has not responded to appropriate antibiotic therapy. The patient's longtime partner, Richard, has a signed durable power of attorney (DPOA) and states that if the patient's condition becomes futile the patient would not want ongoing ventilation. As the ICU attending you decide that ongoing intubation is futile. You consult with Richard and decide to remove the patient from the ventilator to allow him to die in the morning. The patient's Roman Catholic parents arrive from Kansas and threaten a lawsuit if the ventilator is withdrawn. There are several key questions which come out of this case: Who is the legal decision maker here? What are some of the pertinent social influences in this case? Who are some other staff members who may be able to help? How should the physician deal with any prejudices they have in this case?

What is the legal decision making status of a long-term partner? Richard, the durable power of attorney is the legal decision maker in this case. The document is a legally binding agreement that states Richard is the final arbiter of all medical decisions once the patient becomes incapacitated. This creates a legal foundation for Richard to keep his role as the final medical decision maker in conjunction with the attending physician while allowing room for discussion with the family on this difficult topic. How should I facilitate communication between family members? This is an unfortunate situation for everybody involved. The physician can help diffuse this situation by trying to understand the different perspectives that each of the involved individuals brings to the situation. The family arrives to see their dying son and may be confronted with multiple issues for the first time. First they may be finding out that their son is gay, that he has AIDS, and that he is immanently dying all at the same time. Any of these issues may be a shock to the family, so it is important to keep this perspective in mind when making difficult care decisions and to communicate clearly and honestly with them. Communication regarding the patient's care should be consented to by the patient whenever possible. Alternatively, individuals in the gay communities in metropolitan areas that have been severely affected by AIDS have watched many of their friends die of their disease and are very well educated about end of life issues. It is likely that Richard as your patient's DPOA has spent significant time considering these issues with the patient before becoming the patient's surrogate. His role as the patient's significant other is not legally defined in many areas of the United States at this time. This relationship is often the equivalent of marriage in the gay community and should be respected by the hospital personnel in all points of medical care. Who are some other staff members who may be able to help? This is a case where several members may help with the decision. ICU nurses often have experience and perspective in dealing with grieving families of terminally ill patients as do staff social workers or grief counselors. Another invaluable resource in this case is a hospital chaplain or spiritual counselor who may be able to provide spiritual support and guidance to the family. It is important here to find out what resources are available in the hospital for Richard and the patient's family and after discussing the case with them, seek help from these other skilled professionals. If you as a physician have cultivated a relationship with these services it is often appropriate to invite them to a family meeting so that they can help you focus the discussion on the care of the patient, who is always your first priority as a physician. How should I deal with any prejudices I may have in this case? Much has been written on the responsibility of the physician in taking care of the patient with AIDS. The AMA position is "A physician may not ethically refuse to treat a patient whose condition is within the physician's realm of competence.... neither those who have the disease or are infected by the virus should be subject to discrimination based on fear or prejudice, least of all from members of the health care community." From this quote it is safe to say that the physician has a fiduciary responsibility toward the care of the HIV infected patient and there is no room within the profession for prejudice for people with AIDS. This stand on prejudice should cover not only gay men with AIDS, but also all other patients that a physician takes care of, even the next two cases (Case 2 and Case 3). (See also Personal Beliefs.)

Why is medical futility controversial?

While medical futility is a well-established basis for withdrawing and withholding treatment, it has also been the source of ongoing debate. One source of controversy centers on the exact definition of medical futility, which continues to be debated in the scholarly literature. Second, an appeal to medical futility is sometimes understood as giving unilateral decision-making authority to physicians at the bedside. Proponents of medical futility reject this interpretation, and argue that properly understood futility should reflect a professional consensus, which ultimately is accepted by the wider society that physicians serve. Third, in the clinical setting, an appeal to "futility" can sometimes function as a conversation stopper. Thus, some clinicians find that even when the concept applies, the language of "futility" is best avoided in discussions with patients and families. Likewise, some professionals have dispensed with the term "medical futility" and replaced it with other language, such as "medically inappropriate." Finally, an appeal to medical futility can create the false impression that medical decisions are value-neutral and based solely on the physician's scientific expertise. Yet clearly this is not the case. The physician's goal of helping the sick is itself a value stance, and all medical decision making incorporates values.

A 16-year-old female presents to a family physician to obtain a referral for family therapy. She is estranged from her mother and stepfather, who see the same physician. For many years, this patient responsibly cared for her four younger siblings while their single mother worked. Since her mother's marriage, the family has become involved in a fundamentalist church. The patient moved out when she felt the social and moral restrictions of the family's religion were too burdensome for her. The patient seemed quite mature; she maintained a 3.5 GPA, along with a part-time job. She demonstrated a genuine desire for reconciliation, and the therapy referral was provided. She also requested and obtained a prescription for contraceptives during the visit, with the assurance that her sexual activity would be kept confidential. In follow-up, she reported that the therapist had informed her that if she mentioned anything about being sexually active with her adult partner, he would be obliged to report her to the state. The patient was very concerned about the conflict between this statement and the family physician's prior assurance of confidentiality. Should this patient's confidentiality be broken?

While the physician has a moral obligation to obey the law, he must balance this against his responsibility to the patient. In researching the Criminal Code of Washington, the physician learned that sexual intercourse with a minor, at least 16, but under 18, is a class C felony, and a reportable offense, if the offender is at least 90 months older than the victim. This patient's relationship did not actually meet the criteria for mandatory reporting. Had this not been the case however, the physician could be justified in weighing the balance of harms arising from the filing of such a report. There is little justification for informing the family of the young woman's sexual activity. Due to the family's strong fundamentalist beliefs, significant damage would have occurred in the family reconciliation process with this discovery. Although they would clearly disapprove of the patient's actions, her choices carry no risk of harm to them.

How do I know if the treatment is no longer "of benefit?"

Write Us When is it justifiable to discontinue life-sustaining treatments? How do I know if the treatment is no longer "of benefit?" Do different standards apply to withholding and withdrawing care? Do patients have to be terminally ill to refuse treatment? What if the patient is not competent? What if I'm not sure if the patient is competent? Is a psychiatry consult required to determine decision making capacity? Does depression or other history of mental illness mean a patient has impaired decision making capacity? Is it justifiable to withhold or withdraw food or fluids? Is it justifiable to withhold or withdraw care because of costs? On the medicine wards, you will have patients who are receiving treatments or interventions that keep them alive, and you will face the decision to discontinue these treatments. Examples include dialysis for acute or chronic renal failure and mechanical ventilation for respiratory failure. In some circumstances, these treatments are no longer of benefit, while in others the patient or family no longer want them. When is it justifiable to discontinue life-sustaining treatments? If the patient has the ability to make decisions, fully understands the consequences of their decision, and states they no longer want a treatment, it is justifiable to withdraw the treatment. Treatment withdrawal is also justifiable if the treatment no longer offers benefit to the patient. How do I know if the treatment is no longer "of benefit?" In some cases, the treatment may be "futile"; that is, it may no longer fulfill any of the goals of medicine. In general, these goals are to cure if possible, or to palliate symptoms, prevent disease or disease complications, or improve functional status. For example, patients with severe head trauma judged to have no chance for recovery of brain function can no longer benefit from being maintained on a mechanical ventilator. All that continuation would achieve in such a case is maintenance of biologic function. In such a case, it would be justifiable to withdraw mechanical ventilation.

Sexually Transmitted Diseases and Contact Tracing MG is a 27-year-old graduate student, recently married, who comes into the student health clinic for a routine pelvic exam and Pap smear. During the course of the exam, the gynecology resident performing the exam obtains the Pap smear, but also obtains cervical cultures for gonorrhea and chlamydia. The examination concludes uneventfully. Several weeks later, MG receives a postcard indicating that the Pap smear was normal, with no evidence of dysplasia, but that the cervical culture for gonorrhea was positive. The card instructs her to come into the clinic to discuss treatment, and that "public health authorities" have been notified for contact tracing, which refers to the identification and diagnosis of sexual partners, as required by law. The young woman is terrified that her husband will be contacted. Is contact tracing ethically justified?

Yes. Her sexual partners have a right to know that they were exposed to gonorrhea. Notification has positive public health benefits that outweigh the young woman's concerns about violation of privacy. First, those notified can be tested and treated. Second, they can take precautions to protect others from contracting gonorrhea (like using condoms). Third, their sexual partners can be notified to further reduce the spread of infection. To address the young woman's concerns, she can seek advice from her doctor and public health officials and may choose to tell her husband herself. Should she fail to inform her husband, public health officials will be obligated to do so.

What if the patient or family requests an intervention that the health care team considers futile?

You have a duty as a physician to communicate openly with the patient or family members about interventions that are being withheld or withdrawn and to explain the rationale for such decisions. The aim of respectful communication should be to elicit the patient's goals, explain the goals of treatment, and help patients and families understand how particular medical interventions would help or hinder their goals and the goals of treatment. It is important to approach such conversations with compassion. For example, rather than saying to a patient or family, "there is nothing I can do for you," it is important to emphasize that "everything possible will be done to ensure the patient's comfort and dignity." In some instances, it may be appropriate to continue temporarily to make a futile intervention available in order to assist the patient or family in coming to terms with the gravity of their situation and reaching closure. For example, a futile intervention for a terminally ill patient may in some instances be continued temporarily in order to allow time for a loved one arriving from another state to see the patient for the last time. However, futile interventions should not be used for the benefit of family members if this is likely to cause the patient substantial suffering, or if the family's interests are clearly at odds with those of the patient. If intractable conflict arises, a fair process for conflict resolution should occur. Involvement of an ethics consultation service is desirable in such situations. The 1999 Texas Advance Directives Act provides one model for designing a fair process for conflict resolution.

Under what circumstances should I call the ethics consultant/service?

You should consider asking for a case consultation when two conditions are met: you perceive that there is an ethical problem in the care of patients, and health care providers have not been able to establish a resolution that is agreed upon by the patient/surrogate and the clinicians caring for the patient While a communication breakdown is often a factor in ethics consultation, an ethical dilemma occurs when two courses of action may both be ethically defensible. These dilemmas are often due to a conflict between principles (e.g., autonomy, beneficence, justice) or between principles and outcomes. At most hospitals, anyone may request an ethics consultation including the patient or family. Please check the hospital's policies to learn how to request an ethics consultation. You should consider asking for an ethics consultation even when patient care is not the central issue if you believe that there is a lack of clarity about an ethics policy or concept that would benefit from education. The following are examples: a question about professionalism, a general question about patient privacy or confidentiality, and a question about ethical practices in the workplace, human resources, resource allocation, or the business practices of the institution.

How should I advise a patient if she doesn't have anyone to name as a proxy?

You should inform the patient that the best course of action under these circumstances is to write down her wishes and give a copy to her health care providers. She should fill out a legal form, such as a living will, with as much detail as possible, and then include more detailed values and preferences to provide a better understanding of her wishes. Copies of completed documents should be provided to every health care clinic/facility where the patient receives care

What if I see my resident or attending doing something "unethical"?

You will encounter many positive role models during your medical training. However, you will also see some behaviors and actions that are downright troubling or offensive. Because of the "team hierarchy," you may feel unable to confront someone who is "above" you or, more concerning, in control of your evaluation. However, you do still have several duties in this case. Ideally, you could talk with your resident about what you observed. Everyone has a unique perspective and your resident may have a rationale for his behavior that was unknown to you. Approaching him honestly, with simple questions, may allow him the benefit of the doubt and open up a dialogue between you. The nature of the observed 'unethical' act determines what your obligations are. In simpler cases, it can be a matter of treating it as a negative lesson in how NOT to be a physician. In more complex instances, patient care may be in jeopardy and you may have an obligation to report the resident's behavior if he refuses to discuss it with you directly. Your attending physician or clerkship coordinator can be valuable resources as you make these judgment calls. Discussing these instances with your peers can also be helpful.

Examples of helpful communication strategies:

"What I hear from you is that . . . Did I get that right?" "How do you feel about the care you are receiving from me? It seems to me that we sometimes don't work together very well." "It's difficult for me to listen to you when you use that kind of language." "You seem quite upset. Could you help me understand what you are going through right now?" "What's your understanding of what I am recommending, and how does this fit with your ideas about how to solve your problems?" "I wish I (or a medical miracle) could solve this problem for you, but the power to make the important changes is really yours."

Strategies for maintaining a therapeutic relationship

1) Be compassionate and empathic. Keep in mind that most patients whom you find frustrating to deal with have experienced significant adversity in their lives. 2) Acknowledge and address underlying mental health issues early in the relationship. 3) Prioritize the patient's immediate concerns and elicit the patient's expectations of the visit and their relationship with you. 4) Set clear expectations, ground rules, and boundaries and stick to them. Have regular visits, which helps convey confidence that the patient can deal with transient flare-ups without an emergency visit. 5) Be aware that strong negative emotions directed at you are often misplaced. The patient may be imposing feelings and attitudes onto you that they have had toward other doctors, friends, family members in the past. This is known as transference. Acknowledge the patient's feelings and set behavioral expectations. 6) Be aware of your own emotional reactions and attempt to remove yourself so you can objectively reflect on the situation. Involve colleagues. Vent your feelings or debrief confidentially with a trusted colleague so that your negative emotions are kept at bay during patient encounters. 7) Recognize your own biases. For example, patients with addictions genuinely need medical care, but the behaviors associated with addiction are vexing for health care providers. These patients are often both vulnerable and manipulative. Be sure that you are attentive to their vulnerability, rather than focusing exclusively on their manipulative behaviors. 8) Avoid being very directive with these patients. A tentative style tends to work better. Remember that you provide something many of these patients do not have-a steady relationship with someone who genuinely wants to help them. This in itself can improve the patient's health, even in the absence of medical treatment. 9) Prepare for these visits. Keep in mind your goals of care and make a strategy for the encounter before it occurs.

Are advance directives legally binding?

Advance directives are recognized in one form or another by legislative action in all 50 states (in Washington, see RCW 70.122). If the directive is constructed according to the outlines provided by pertinent state legislation, they can be considered legally binding. In questionable cases the medical center's attorney or ethics advisory committee can provide guidance on how to proceed

What types of advance directives are currently available?

Advance directives were officially inaugurated in 1977 with passage of the Natural Death Act in California. The law allowed patients to direct their physicians to withhold or withdraw life-sustaining treatment when they were terminally ill and death was imminent. In 1992, Washington State passed a similar Natural Death Act. Originally, it only applied to persons who were terminally ill and for whom death was imminent. However, it was later amended to include persons in permanent unconscious conditions and expanded to apply explicitly to discontinuation of artificial hydration and nutrition. A 1991 federal law, the Patient Self-Determination Act, requires that patients are informed about their right to participate in health care decisions, including their right to have an advance directive. Advance directives fall into two broad categories: instructive and proxy. Instructive directives allow for preferences regarding the provision of particular therapies or classes of therapies. Living wills are the most common examples of instructive directives, but other types of instructive directives, such as no transfusion and no CPR directives are also employed. The proxy directive, generally a Durable Power of Attorney for Health Care (DPAHC), allows for the designation of a spokesperson or surrogate medical decision maker of the patient's choosing. This surrogate decision maker (often referred to as a health care agent) makes health care decisions for the patient in the event she is incapacitated. Although not actually an advance directive, many states have adopted the use of life-sustaining treatment order forms to facilitate communication about end-of-life wishes with regard to the use of treatments. Most prominent and a model for many states is the POLST, physicians orders for life-sustaining treatment. These orders stay with the patient and are designed to direct care both in the home and outpatient settings, and are used sometimes to facilitate transfers within the hospital. Although advance directives are usually focused on end-of-life decision making, there is another type of directive for patients with recurrent severe mental health problems. This is the mental health directive or psychiatric advance directive. In this type of directive the patient informs health care providers about their preferences for future mental health care if s/he were to become decisionally incapacitated.

When should CPR be administered?

Anesthesiologists and surgeons may be reluctant to accept DNR orders on patients undergoing surgery because of the scope of medical practice which constitutes "normal care" in the surgical environment. Many surgeries require intubation and mechanical control of respiration for the duration of surgery, to protect the airway from aspiration, prevent anesthetic-induced hypoventilation, to allow the administration of paralytic agents to prevent muscle contraction during surgery, and for many other reasons. Yet intubation and ventilatory assistance are mainstays of CPR. It is inaccurate to call anesthesia "ongoing resuscitation," yet the administration of anesthetic agents frequently causes initial changes in the autonomic nervous system, such that hypotension, tachycardia, bradycardia, and temporary cardiac dysrhythmias can result. It is not rare to administer vasopressive medications and antiarrhythmic agents during the course of "normal" anesthetic management. Such medications are often also considered a vital part of effective administration of CPR. Finally, both invasive and noninvasive technology in the OR permits easy application of therapeutic measures which might seem extreme on the medical ward, such as external or transvenous pacing and defibrillation. Under most other circumstances, such measures would fall almost exclusively within the realm of CPR. So where do we draw the line between "normal" and "usual" procedures in the operating room, and "extraordinary" procedures which constitute CPR? Many authorities have suggested that the application of chest compressions is an unusual enough occurrence even in the OR setting, that it provides an medical and ethical boundary between CPR and normal anesthetic care.

What you need to understand to care for the dying?

Another useful framework was outlined by Joanne Lynn, who was one of the principal investigators of SUPPORT. She suggests that there are four things clinicians must know to care for the dying. The patient's story - including how that person has viewed her life, the other persons important to her, and how she could bring her life to a close in a way that would be true to herself. The body - which covers the biomedical understanding of disease, and what limits and possibilities exist for that person. The medical care system available for this particular patient - knowing how you can make the system work for the patient, as well as the relevant law and ethics. Finally, you must understand yourself - because you, as a physician, can be an instrument of healing, or an instrument that does damage. Obviously, learning how to do all this is beyond the scope of this web page--these are goals that guide a career of learning and reflection. But this framework provides guidelines for you as you develop your own approach to caring for dying patients.

Mrs. W is an 81-year-old woman with recurrent colon cancer with liver metastases admitted to the hospital for chemotherapy. Because of her poor prognosis, you approach her about a DNR order, but she requests to be "a full code." Can you write a DNR order anyway?

As a competent adult, this patient has the right to make decisions about her medical care. You must respect her wish not to be treated until she gives you permission to do so. However, it is especially important under these circumstances to clarify with Mrs. W her understanding of what CPR means and what her likely outcomes will be. To ensure that there is a clear understanding, addressing Mrs. W's hopes and goals is essential. Perhaps she wants to live to see her granddaughter graduate from high school in two months, knowing that she will die soon thereafter, however she does not want heroic measures to prolong her life forever. Additionally, she may not want to live on machines for a prolonged period of time, and hence, if she survives cardiopulmonary arrest yet is dependent on a ventilator to breathe, her decision may change. Understanding Mrs. W's goals may help you partner with her to make meaningful medical decisions that address her concerns and wishes throughout the duration of her illness.

A 23-year-old Navajo man has injured his leg after a fall. He presents to the emergency room of the reservation hospital where he is complaining of pain. His leg appears to be broken. The man requests that you call a medicine man before doing anything further. Should you find a medicine man? Should you proceed with treatment?

As a competent adult, this patient has the right to make decisions about his medical care. You must respect his wish not to be treated until he gives you permission to do so. Calling the local medicine man will show your respect for the patient and strengthen the patient's trust in you and your abilities.

What should be included in a discussion of DNR orders in the OR with the patient or patient's surrogates?

As discussed above, surgery and anesthesia may require the administration of medical therapies, which under other circumstances might be considered resuscitation. It is an ongoing source of discussion about what constitutes appropriate information and choices to present to patients about to undergo surgery who have DNR orders on their charts. Since the goal of medical therapy is to provide meaningful benefits to the patient, discussion of DNR orders in the OR should center around the patient's goals for surgical therapy. Patients may have fears of "ending up a vegetable" on a ventilator after surgery, for example. In those cases, discussion should center around the positive prognosis for patients who have CPR in the OR, together with reassurance that the patient's stated wishes in their advanced directive regarding ventilatory support would be followed postoperatively after anesthetic effects are ruled out as a cause of ventilatory depression. Most authorities now agree that a "smorgasbord" or checklist "yes-or-no" approach to the various procedures in the operating room is confusing and counterproductive to the purpose of DNR discussions. Anesthesiologists in particular need to be aware that studies indicate that many patients with DNR orders in their charts (up to 46%) may be unaware that the order exists, even when they are competent. While policies at the University of Washington Medical Center require documentation of discussion of DNR orders with the patient or appropriate surrogates, anesthesiologists and surgeons should nevertheless approach the patient about to undergo surgery with sensitivity to the fact that they may be unaware of their DNR order. If this proves to be the case, a full discussion of the DNR order should be undertaken prior to proceeding.

Is the outcome from CPR different in the OR than on the medical ward?

CPR in the OR carries a very different medical prognosis than CPR administered in other hospital areas. While only 4 to 14% of all patients resuscitated in the hospital survive to discharge, 50 to 80% of patients resuscitated in the OR return to their former level of functioning. This is probably due to several differences between arrest on the medical ward and arrest in the OR. In the OR, the event of arrest is always witnessed, and the proximate cause usually known, allowing rapid, effective intervention which is directed toward the specific cause of arrest. Also, causes of arrest in the OR are often reversible effects of anesthesia or hemorrhage, and not usually due primarily to the patient's underlying disease. Patient and physicians may require correction of the perception that CPR in the OR is just as futile as CPR on the general medical ward.

When should CPR be administered?

CPR should be administered in the absence of a valid physician's order to withhold it.

What about emergencies?

Even in emergencies, physicians have an ethical obligation to recognize and respect patient autonomy. Whenever possible, physicians should obtain input from the patient, or when the patient is incapacitated, from appropriate surrogates, regarding the status of the patient's DNR orders in the OR. In the absence of such input, consensus should be reached among the caregivers about the medical benefits or futility of CPR. In any case, medical care of the patient in the absence of patient input should be directed toward realizing, to the best of the physician's ability and knowledge, the patient's goals.

What if the family disagrees with a patient's living will?

If there is a disagreement about either the interpretation or the authority of a patient's living will, the medical team should meet with the family and clarify what is at issue. The team should explore the family's rationale for disagreeing with the living will. Do they have a different idea of what should be done (e.g., based on other communication from the patient)? Do they have a different impression of what would be in the patient's best interests, given her values and commitments? Or does the family disagree with the physician's interpretation of the living will? Is there a conflict of interest that may be fueling disagreement between the patient and family members? These are complex and sensitive situations and a careful dialogue can usually surface many other fears and concerns. However, if the family merely does not like what the patient has requested, they do not have much ethical authority to sway the team. If the disagreement is based on new knowledge, substituted judgment, or recognition that the medical team has misinterpreted the living will, the family has much more say in the situation and most hospitals would defer to the family in these situations. If no agreement is reached, the hospital's Ethics Committee or Ethics Consultation Service should be consulted.

What should we do with DNR orders in the OR?

In 1992, the American Society of Anesthesiologists (ASA) produced Guidelines for the Ethical Care of Patients with Do Not Resuscitate Orders, and Other Orders Limiting Care in the Operating Room. Out of respect for patient autonomy, or the right of competent, adult patients, to determine their own medical care, no specific definition of CPR was provided in the document. Instead, it requires a discussion with the patient to define medical procedures under anesthesia to which the patient would consent. Shortly after the ASA adopted its guidelines, the American College of Surgeons, and the Association of Operating Room Nurses (AORN) adopted guidelines which drew directly from the ASA's document. All acknowledged that patients do not check their rights to self-determination at the OR doors, that policies automatically suspending or upholding DNR orders in the OR were ethically suspect, and that rediscussion of the DNR order should occur, whenever possible, prior to undertaking surgery and anesthesia.

How extensively is CAM used?

In 1993, David Eisenberg and colleagues at Harvard University published a systematic survey on the use of "unconventional medicine" in the United States. They determined then that one-third of respondents (study n=1539 adults) used CAM, generally to treat chronic (as opposed to life-threatening) conditions, and the highest use was reported by nonblack persons between 25-49 years of age who were well educated and of a relatively high socioeconomic status. Total expenditures for CAM therapies were $13.7 billion, three quarters of which was paid out of pocket. Subsequent, more exhaustive, surveys demonstrated that CAM use has steadily increased. A follow-up survey by Eisenberg in 1998 and a 2008 report from DHHS place the percentage of adult Americans who use CAM at around 40%, and the percentage of children at 12%. CAM expenditures in 1998 were conservatively estimated at over $21 billion. According to the 2008 report the most commonly used CAM therapies are nonvitamin, nonmineral natural products (for example, herbal products such as Echinacea) and deep breathing/ meditation practices. The most common presenting complaints among people seeking CAM treatment are back pain and other musculoskeletal complaints (National Health Statistics Reports ). At its founding in 1847, the AMA declared itself and its allopathic members to be sharply opposed to homeopathy, which they later decried as a "cult". The antipathy of allopaths toward many forms of CAM persisted for much of the 20th century. However in recent decades significant progress has been made in establishing integrative medical centers at which CAM and allopathic practitioners work together to co-manage patients. In the Seattle area each of the major hospitals now includes CAM in their curricula, houses integrated medicine clinics, and engages in collaborative research with CAM researchers. Nationwide, this trend holds also holds.

What if the patient starts to cry while I am talking?

In general, it is better simply to wait for the person to stop crying. If it seems appropriate, you can acknowledge it ("Let's just take a break now until you're ready to start again") but do not assume you know the reason for the tears (you may want to explore the reasons now or later). Most patients are somewhat embarrassed if they begin to cry and will not continue for long. It is nice to offer tissues if they are readily available (something to plan ahead); but try not to act as if tears are an emergency that must be stopped, and don't run out of the room--you want to show that you're willing to deal with anything that comes up.

What should I know about the hospice approach?

In order to help someone towards a decent, or even good, death, the hospice framework is very helpful. Hospice started as a grassroots effort, as a view of dying that lets go of the possibility of cure. Instead, hospices emphasize symptom control and attention to psychological and spiritual issues. Pathophysiology becomes less important and personal meaning becomes more important. Thus this framework analyzes a person's medical care into four major topics, and this can be used to outline day-to-day care plans for a patient: Pain - one of the things most feared by patients with life-threatening illness. Symptom control - including dyspnea, nausea, confusion, delirium, skin problems, and oral care. Psychological issues - especially depression, sadness, anxiety, fear, loneliness. Spiritual or existential issues - including religious or non-religious beliefs about the nature of existence, the possibility of some type of afterlife. Hospice care in Washington State is most often provided by multidisciplinary teams who go to patients' homes. This care is covered by Medicaid for patients judged to have less than six months to live. Hospice care is generally underutilized, and even though most hospice teams feel that at least six weeks of hospice care is optimal, most patients receive much less because they are either referred very late or have not wanted hospice. A major problem in connecting hospice care to acute medical care is that referral implies a "switch" from curative to palliative medicine-a model that does not fit comfortably in many illnesses.

Describe the difference between physician-assisted suicide and euthanasia. Do you think either should be legal/illegal? Why?

In physician assisted suicide, the physician provides the patient with the means to end his or her own life. In euthanasia, the physician deliberately and directly intervenes to end the patient's life; this is sometimes called "active euthanasia" to distinguish it from withholding or withdrawing treatment needed to sustain life. I think physician assisted suicide should be legal, but euthanasia should be illegal. It is my opinion that patients have the right to decide and obtain methods to end their life. However, I believe physicians have the ethical obligation to not perform a procedure that deliberately ends the life of a patient. This would break the hypocratic oath. Types of factors that should be considered: 1) type of suffering (pain or loss of dignity); 2) life expectancy (short, medium, or long), 3) the existence of an explicit request (yes, no, not mentioned).

What if patients are unable to express what their wishes are?

In some cases, patients are unable to participate in decision-making, and hence cannot voice their preferences regarding cardiopulmonary resuscitation. Under these circumstances, two approaches are used to ensure that the best attempt is made to provide the patient with the medical care they would desire if they were able to express their voice. These approaches include Advance Care Planning and the use of surrogate decision makers. (See Advance Care Planning and Advance Directives , and Surrogate Decision Makers) Not all patients have Advance Care Plans. Under these circumstances, a surrogate decision maker who is close to the patient and familiar with the patient's wishes may be identified. Washington state recognizes a legal hierarchy of surrogate decision-makers, though generally close family members and significant others should be involved in the discussion and ideally reach some consensus. Not all states specify a hierarchy, so check your state law. Washington's hierarchy is as follows: Legal guardian with health care decision-making authority Individual given durable power of attorney for health care decisions Spouse Adult children of patient (all in agreement) Parents of patient Adult siblings of patient (all in agreement) The surrogate decision maker is expected to make decisions using a substituted judgment standard, which is based on what the patient would want if she could express her wishes. In certain circumstances, such as in children who have not yet developed decisional capacity, parents are expected to make decisions based on the best of the patient, called a best interest standard.

What if CPR is not futile, but the patient wants a DNAR order?

In some cases, patients may request their desire to forgo attempting CPR at the time of admission. Some of these patients may have an advanced care directive that indicates their preferences to forgo attempting CPR. In other cases, a patient may explicitly request CPR not to be performed. If the patient understands her condition and possesses intact decision making capacity, her request should be honored. This position stems from respect for autonomy, and is supported by law in many states that recognize a competent patient's right to refuse treatment.

When should CPR be administered?

In the absence of a valid physician's order to forgo CPR, if a patient experiences cardiac or respiratory arrest, the standard of care is to attempt CPR. Paramedics responding to an arrest are required to administer CPR. Since 1994 in Washington, patients may wear a bracelet or carry paperwork that allows a responding paramedic to honor a physician's order to forgo CPR. In the state of Washington, the POLST form is a portable physician order sheet that enables any individual with an advanced life-limiting illness to effectively communicate his or her wishes to limit life-sustaining medical treatment in a variety of health care settings, including the outpatient setting

What ethical issues are associated with research in CAM?

In the context of CAM research, different ethical issues arise. The following is a small sampling: Informed consent The regulations stipulate that the informed consent document and process must accurately describe reasonably anticipated risks and potential benefits. Yet in many instances CAM therapies have not had systematic safety or efficacy data collected, and are ratified by historical anecdotal evidence. The informed consent document should state that although a therapy might have been in widespread use, rigorous safety data may not be available. Misconceptions A prospective research subject must weigh for themselves the balance of risks and benefits that will accrue from their participation. The therapeutic misconception is the (documented) belief held by many research subjects that they will benefit from participating in a research study (allopathic or CAM), irrespective of disclaiming language in the informed consent form. This misconception weighs in favor of participation. Additionally, there is a widely held notion that if something is "natural" it must be safe, or beneficial. This misconception also weighs in favor of participation by reducing the perceived risks associated with the study. In CAM research it is imperative to impress upon potential subjects that the risks and benefits of participation are more difficult to anticipate than they are for better-studied interventions. Study design The Belmont principle of beneficence dictates that there be a reasonable likelihood of obtaining useful data from a study. As discussed above, the difficulties inherent in generating sound designs in CAM research challenge adherence to this principle. Research study designs must undergo rigorous review by scientists and clinicians well versed in the CAM modality being tested.

What ethical issues are associated with CAM in clinical practice?

In the context of clinical practice, the ethical issues pertain to providing optimal medical care to an individual. Any physician, allopathic or otherwise, is bound by oath to do no harm and to provide the most efficacious therapies to their patient. The precepts of evidence-based medicine (EBM) and the accessibility of medical research literature provide clinicians with powerful tools to identify such therapies. In evaluating the risk of harm and the potential benefits of any therapy, weight must be given to the amount and quality of research that has been done on the intervention, known risks and side effects of the therapy, the credential and competence of the practitioner, the seriousness of the condition being treated, and the belief system and wishes of the patient. Given the relative dearth of research literature on many CAM therapies, the clinician must use best judgment to decide which therapies are unlikely to do harm, either directly or by reducing the effectiveness of other therapies, and which may offer some, if not great, benefit. A CAM therapy that is neither harmful nor effective can become damaging if it precludes the patient obtaining effective treatment. The advantage of integrative medical clinics is to take much of the guesswork out of this algorithm. Well-trained, licensed CAM providers working alongside conventional clinicians create an environment in which patient care can utilize current best practices in each discipline. CAM therapies for some medical conditions require close supervision by a CAM professional. Just as a patient with diabetes (or hypertension, depression, etc) should receive ongoing medical supervision for his/her diabetes (hypertension, depression, etc) management, the same holds for CAM therapies. Many warnings about the risks associated with CAM therapy use are grounded in an assumption that the patient may be self-treating and/or not receive adequate monitoring by a trained CAM professional.

As a trainee, should I do advance care planning with my patients?

In the ideal doctor-patient situation the primary care physician should initiate discussion when the patient is not acutely ill. However, this often is not the case and therefore these discussions frequently occur in the hospital setting. Regardless of the setting, good medical practice includes having these discussions. Thus, medical students and residents should engage the patients they are caring for in these discussions. If the patient has been recently diagnosed with a terminal or life changing condition, has severe depression, demonstrates paranoid ideation, or is suicidal, you should ask the responsible attending physician whether this is an appropriate time to raise these issues. Otherwise, you should initiate the discussions and request faculty support (such as role modeling or mentoring) if needed. You should review the framing of the discussion and the patient's views with the attending physician responsible for the patient's overall care.

A 75-year-old woman shows signs of abuse that appears to be inflicted by her husband. As he is her primary caregiver, she feels dependent on him and pleads with you not to say anything to him about it. How is this case different from Case 1? How would you handle this situation?

In this case, the doctor may feel conflicted. A vulnerable elderly woman needs protection and may be fearful of her abusive husband. Reporting to Adult Protective Services, required if the physician suspects elder abuse (http://www.aasa.dshs.wa.gov/aps/), may also make the patient fear that she will be exposed to further abuse by her enraged husband. The therapeutic alliance with the patient provides a unique opportunity to support the patient's health and safety. The physician should discuss services that might be available to assist her; encourage the woman to seek help; provide medical care and counseling; and explain the physician's duty to report. It should also be noted that anyone who suspects elder abuse is required to report it - neighbors, social workers, family members, etc. Whenever the safety of an individual is at stake, confidentiality may be breeched and there are usually specific agencies (public health, Child Protective Services, Adult Protective Services) designated for reporting. These agencies then assume responsibility for investigating the reports and taking appropriate steps to ensure the person's safety.

A young mother has just been informed that her 2-year-old son has leukemia. The mother refuses permission to begin chemotherapy and informs the team that their family physician (a naturopath) will follow the child's illness. What should you do?

Of utmost importance are the child's best interests, which include getting good medical care and maintaining a close connection with his mother. One way to achieve both is by requesting a care conference with both the mother and the family's naturopathic physician. It is best not to assume that the family's naturopathic provider is in agreement with the mother. The potential therapeutic value of chemotherapy is part of licensed naturopathic physician training, as is the specialist nature of cancer treatment. The family ND may fully support chemotherapy as part of the child's care plan. If the mother refuses this meeting and you remain convinced that chemotherapy is the only hope this child has, you are professionally obligated to seek a court order to appoint a guardian for the child. If chemotherapy offers a clear and compelling survival benefit, the justification for seeking legal intervention increases.

What patient characteristics and behaviors make a clinician-patient relationship difficult?

Patients are labeled 'difficult' based on the feelings they invoke in clinicians, such as anger, frustration, anxiety, dread, and guilt. (Wasan et al., 2005)Patients who, for medical or non-medical reasons, appear ungrateful or frivolously utilize medical care are most likely to be described as 'difficult'. They may continue to seek medical attention but not heed the advice they are given. Patients may have multiple medical complaints, psychiatric conditions (helplessness, depression, anxiety, self-loathing), abrasive personality traits (expressing rage, inflexibility), personality disorders, addictions, and multiple physical symptoms of unknown or ambiguous etiology. They often make requests that clinicians think are inappropriate, such as requests for additional pain medicine, increased phone contact or clinic appointments, etc. Worried well patients, patients with poorly controlled chronic pain, who are non-compliant with medical regimens, seductive or manipulative, consume a lot of clinician-time and health care resources, somatisize, or are self-destructive or attention-seeking may also be labeled 'difficult'. (Krebs et al., 2006; Elder et al., 2006)

Why is confidentiality important?

Patients routinely share personal information with health care providers. If the confidentiality of this information were not protected, trust in the physician-patient relationship would be diminished. Patients would be less likely to share sensitive information, which could negatively impact their care. Why is confidentiality important? Creating a trusting environment by respecting patient privacy encourages the patient to seek care and to be as honest as possible during the course of a health care visit. (See also Physician-Patient Relationship.) It may also increase the patient's willingness to seek care. For conditions that might be stigmatizing, such as reproductive, sexual, public health, and psychiatric health concerns, confidentiality assures that private information will not be disclosed to family or employers without their consent.

What physician characteristics and behaviors make the clinician-patient relationship difficult?

Physician attitudes, biases, fatigue, stress, burn-out, as well as language and cultural barriers may negatively impact the relationship (Hull & Broquet, 2007). Angry, defensive, fatigued, harried, dogmatic, or arrogant physicians are more likely to see patient encounters as difficult (ibid). Data suggests that some physicians are more likely to describe difficult patient encounters when they have "lower job satisfaction, less experience, [and] poorer psychosocial attitudes" (Elder et al., 2006). Also, physicians who have low tolerance for illnesses that are incurable or untreatable can find patients with these illnesses difficult to manage. Also susceptible are physicians who have a hard time adjusting their practice to accommodate patients who seem overly dependent or physicians who feel helpless or annoyed when the patient's ailments are exacerbated by social factors (family conflict, poverty, addiction, etc.). It can be particularly frustrating or infuriating when patients appear ungrateful or even resentful for the care they receive. Recognize this source of frustration and remember that you are likely a vital part of the patient's support system even if the patient appears ungrateful or aggressive. Physicians who tend to emphasize the patient's autonomy in medical decision-making may have a harder time with patients who abdicate responsibility for their health or are comfortable in a more vulnerable role. Highly frustrated physicians tended to be younger, more likely to practice subspecialty internal medicine, and to experience higher stress (Krebs et al., 2006).

Mr. B is a 46-year-old man who comes into clinic to follow-up on multiple uncontrolled medical problems including diabetes, hypertension, obesity, depression and sleep apnea. He is unemployed and is currently homeless, though had been sleeping regularly on a friend's couch. He and the friend have been getting into some arguments and he does not feel comfortable staying there anymore, so is sleeping in his car. On further discussion the patient shares that he lost his last job due to recurrent conflict with a co-worker. He shares his frustration that "everyone is out for me because they're racist." On several occasions he has yelled at the personnel at the front desk when he was unable to get the assistance or appointment he needed. He was seen by a counselor in clinic for his depression, but this also ended in an episode of Mr. B becoming angry and security being called. He has been banned from one of the specialty clinics where he had been referred after a confrontation with staff there. He wonders why he is always getting kicked out of places and feels he has cause and a "right" to be angry. Though he seeks ongoing care, he does not have insight into how his anger is perceived by others.

Prepare for your visit with Mr. B by 'venting' and strategizing with trusted colleagues. Establish and review ground rules for interactions with you and staff. "I understand that you are angry and I would like to spend some time talking with you about that, but I will immediately end our appointment if there is any threatening speech or behavior." Validate some of his feelings by agreeing that racism and other forms of discrimination are wrong and he has every reason to be angry, but that learning different ways to express his anger may help minimize the kinds of social interactions that he dislikes. Discuss treatment modalities that may be beneficial to him in this regard. If possible, schedule his visit before your lunch or a break so you attend to yourself after the visit.

Mr. S is a 48-year-old man with a long history of drug and alcohol use. He has continues to drink alcohol regularly though is regularly seeing a substance abuse counselor. He also uses cocaine with some regularity and buys pain pills on the street. Mr. S has chronic pain due to back, shoulder and neck injuries from the past. When he takes a narcotic he feels better, so would like to be on a regular dose of pain medicine. He is seen frequently in clinic, often as a walk-in seeing a variety of providers, and he invariably asks for pain medicine. Mr. S is often so focused on getting a narcotic prescription that the majority of the visit is consumed and little other health care is provided.

Prepare for your visit with Mr. S by thinking through your goals of care, and determining how you will meet his pain management needs. In patients with active addictions, prescribing or increasing the dosages raises patient safety concerns and tends not to lead to positive outcomes for the patient. Consider talking with colleagues about how they might manage his requests and seek advice from or referral to pain management and/or addiction specialists. If you've noticed that Mr. S has not always been truthful in the past (a common component of addiction), expect that to be the case again. Establish boundaries at the beginning of the visit. Acknowledge his request and share with him what you would like to accomplish before you will discuss pain prescriptions. Consider putting your expectations and agreements about pain management in writing. Make sure he knows that you believe he has legitimate pain issues, but that the therapies you prescribe may not be exactly what he wants. Be firm with the boundaries you set with him. For example, you might tell him "We can schedule monthly visits, but I will not refill prescriptions in between appointments."Â Create a plan for pain crises.

What is the definition and scope of CAM?

The answer to this question continues to invite debate; looking to the federal government for a definition provides at least an historical perspective on the question. In 1991, NIH formed the Office of Alternative Medicine (OAM), with funding of $2 million and the directive to "investigate and evaluate promising unconventional medical practices". At the time, "alternative" simply designated practices considered to be outside of mainstream medicine, or, put another way, practices not commonly taught at conventional medical schools. In 1998 this office was re-named NCCAM, or the National Center for Complementary and Alternative Medicine and in FY 2010 it had a budget of $128.8 million. The addition of the word "complementary" to the title reflected the fact that these practices are increasingly used in conjunction with, and adjunctive to, conventional or "allopathic" medicine. The term "integrative medicine" refers to an approach to patient care that fully utilizes both conventional and alternative methodologies. CAM whole medical systems are naturopathic medicine, homeopathy, Ayurveda and traditional Chinese medicine (TCM) including acupuncture; these systems have evolved over centuries to millennia. Mind-body medicine and energy medicine encompass a broad spectrum of techniques including meditation, QiGong, and hypnotherapy. Somatic therapies include massage, craniosacral, and chiropractic. This is a fractional list of the many therapies and modalities that may reside under the general rubric of CAM. The NCCAM website is an excellent source of information pertaining to CAM.

An elderly woman with chronic kidney disease told her daughters that if she ever ended up with dementia she wouldn't want to live like that. Years later she developed senile dementia and moved into a nursing home. Although she did not recognize family or friends, she enjoyed the company of others and the nursing home's cat. When her kidneys stopped functioning, staff at the nursing home expressed ambivalence about the value of kidney dialysis, yet asked her daughters whether their mother should be started on dialysis. Should the daughters consider her previously stated wishes as an advance directive? What questions should health care providers and family members ask to clarify patient values and preferences so that they can be more easily applied in the future?

The daughters should consider her previously stated wishes as well as her current best interests. The daughters don't know how to proceed because they did not have the advance care planning conversation that clarified what their mother meant when she said that she wouldn't want to live with dementia. Was it the cognitive problems, the problems with self care, living in an institution, or the sense that living with dementia would not bring any joy? Without knowing this, the daughters are unprepared to step into her mother's shoes. However, asking the daughters to describe their mother and what brought her enjoyment and meaning throughout her life might help clarify the meaning of her words. Without really knowing their mother's wishes or feeling comfortable about what she would want as a goal of care under the present circumstances, the decision about dialysis is difficult. The daughters may choose to approve dialysis with the proviso that future triggers could lead to its discontinuation. For example, if her current quality of life deteriorates to the point where she is no longer experiencing joy, or if her behavior on dialysis requires early termination of the dialysis sessions, it may be appropriate to discontinue dialysis at that time.

A 72 year old woman was admitted to the Neurological Intensive Care Unit following a cerebral hemorrhage which left her with severe brain damage and ventilator dependent. One year before this event, the patient and her husband had drawn up "living wills" with an attorney. She was diagnosed by her treating physician as being in a permanent unconscious condition. The patient's living will specified that the patient did not want ventilator support or other artificial life support in the event of a permanent unconscious condition or terminal condition. The patient's husband is her legal next of kin and the person with surrogate decision-making authority. When the living will was discussed with him, he insisted that the patient had not intended for the document to be used in a situation like the present one. Further discussion with him revealed that he understood that the patient would not be able to recover any meaningful brain function but he argued that the living will did not apply because her condition was not imminently terminal. He further indicated that he did not consider his wife to be in a permanent unconscious condition. The immediate family members (the couple's adult children) disagreed with their father's refusal to withdraw life support. The treatment team allowed a week to pass to allow the husband more time to be supported in his grief and to appreciate the gravity of his wife's situation. Nevertheless, at the end of this time, the husband was unwilling to authorize withdrawal of life support measures consistent with the patient's wishes as expressed in her living will.

The ethical and legal parameters in Case 1 are informed consent, surrogate decision-making and the patient's ability to direct her care - expressed in law as a liberty or privacy right and in clinical ethics as respect for patient autonomy. While the details of each case will determine the advice provided, the difficult issues raised in Case 1 prompt consideration of a number clinical ethics and legal issues. Specific clinical ethics and legal issues: The patient is unable to provide informed consent for medical care. Informed consent means making a medical treatment choice and includes the choice of non-treatment. What is known about the patient's wishes for continued medical treatment under her current circumstances? Her providers, referencing intuitional policy, thought ventilator support and CPR were medically futile. A provider's determination of medical futility means that treatment is highly unlikely to provide overall benefit to the patient. Such determinations are case-specific, and should be thoroughly discussed with surrogate decision-makers. While providers may not be obligated to provide medically futile interventions, depending upon circumstances, the patient may be transferred to another facility. Institutional policies are crafted to provide guidance to providers within the context of clinical ethics and the relevant laws and should guide decision-making in this area. In Washington State for example, decisions to withdraw or withhold medical treatment are partially governed by the Washington Natural Death Act which currently requires that the patient be in a permanent unconscious or terminal condition. 21 There are other circumstances in which a surrogate will be able to make choices on behalf of the patient. The patient's advance directive is strong evidence and significant in determining what the patient would want for substituted judgment. Since the patient's husband (her legal surrogate) only made vague statements as to why he thought she would want continued care under these circumstances and the husband's perspective was contradicted by their adult children, it appears the situation requires further communication efforts, e.g., patient care conference, clinical ethics consultation, potential consult with institutional risk manager and/or attorney. The services of a hospital chaplain may also be helpful since the husband had indicated that his religious beliefs played some role in his perspective of his wife's situation. If these additional communication efforts fail to resolve the impasse, one possible legal/risk management approach would be to consider pursuing withdrawal of life support after multiple steps and ongoing consultation. Possible actions might include the following. The content of the patient's advance directive should be verified to be consistent with a decision to forego further life-sustaining measures. Those persons who were present when she prepared and signed the document should be contacted to gather further information about the patient's intentions. The requisite clinical determination(s) ("terminal" or "permanent unconscious" conditions) to activate the patient's advance directive should be confirmed and documented in the patient's chart. Consensus among the medical team should be confirmed regarding: the clinical determinations; the appropriateness of withdrawing life support as in the patient's best interests; and, that withdrawal is consistent with her advance directive. The applicability of the institutional futility or withholding and withdrawal policy should be reviewed and, if applicable, documented in the patient's chart. A patient care conference with the family members should be scheduled to review the patient's prognosis with the family once again. Assuming that the medical team is in consensus about withdrawal, they can communicate their decision to withdraw care at a specific future date and time. With this advance notice of planned future action, the patient's husband is provided an opportunity to seek judicial review or arrange for a transfer of care to another medical facility before the withdrawal of care. At any time throughout this process, it may be possible to break the stalemate of the patient's situation and allow a resolution. It is anticipated that in such a complex medical and emotionally charged circumstance that there would be ongoing communications and multiple opportunities with hospital staff, care providers, and the patient's surrogate and immediate family members about what the patient would want and or what is in her best interest. This situation underscores the importance of communication with the surrogate the throughout the resolution process. A clinical ethicist or palliative care consultant can assist in this process.

Why are advance directives important to medical care?

The major ethical argument for the use of instructive directives, such as a living will, appeals to the ethical principle of autonomy. The principle of autonomy requires physicians to respect the wishes of competent adult patients. Even after a patient loses the capacity to be autonomous, we can continue to respect autonomy by abiding by the patient's prior expressed wishes. In this way, patients can continue to participate (indirectly) in their medical care decisions even if they become decisionally incapacitated, i.e., unable to make informed decisions. Instructive directives may extend individual autonomy and help ensure that future care is consistent with previous desires. The living will was created initially to help prevent unwanted, invasive medical care at the end of life. However, the living will also attempts to ensure that patients receive the treatment they want, which may be invasive, end-of-life care. Most importantly, the living will tries to promote patient-centered care. When a patient becomes incapacitated someone else will be required to make health care decisions. In most states, the patient's spouse is the legal surrogate. Domestic partnership laws in some states empower non-spouse, registered domestic partners to act as surrogate decision makers. If no spouse is available, state law usually designates the order of surrogate decision makers, usually other family members. In Washington State the order of hierarchy is codified in RCW7.70.065. By designating a health care agent through a DPAHC, the patient's choice of a surrogate decision maker supersedes that of the state. A legal surrogate is particularly valuable for persons in non-traditional relationships or without close family. The health care agent need not be a relative of the patient, though this person should have close knowledge of the patient's wishes and views.

A 3-year-old child is brought to your clinic with a fever and stiff neck. You are quite certain the child has meningitis. When you discuss the need for a spinal tap and antibiotic treatment, the parents refuse permission, saying, "We'd prefer to take him home and have our minister pray over him." Can the parents refuse treatment in this case? How should you handle this?

The physician has a duty to challenge the decision of parents when their refusal of treatment would pose a significant risk of substantial harm. Failure to diagnose and treat bacterial meningitis would seriously threaten the health and even life of this child. The physician should share his or her view with the family and seek to elicit their cooperation through respectful discussion. The physicians should be open to alternatives that satisfy the parents' concerns and achieve the goal of keeping the child safe. Inviting the family's religious leader to the hospital while also providing standard medical therapy may prove to be an acceptable compromise. Should these efforts not result in parental permission, the physician is justified in seeking legal authority (in the form of a court order of authorization from a state child protection agency) to proceed with the procedure and treatment of the child. In most states a physician is legally authorized to provide emergency treatment to a child without a court order when delay would likely result in harm.

What are some ways to discover well known sets of beliefs?

There are many groups that share common sets of beliefs. These belief systems may be based on shared religion, ethnicity, or ideology. Knowledge of these beliefs and the reasonable range of interpretation of doctrine can be very helpful in deciding if unusual beliefs should be respected. Good resources for guidance in this area include patients and family members themselves, staff members with personal knowledge or experience, hospital chaplains, social workers, and interpreters. Unusual beliefs that fall outside known belief systems should prompt more in-depth discussions to insure they are reasonable. It is important to explore each individual's beliefs, as shared membership in a particular religious or cultural group does not necessarily entail identical belief systems.

Are there some questions or aids that I could use for advance care planning?

When having a discussion about advance care planning, the following questions are recommended: Who should speak on your behalf if you become so sick you can't speak for yourself? Are there any circumstances that you've heard about through the news or TV where you've said to yourself, "I hope that never happens to me" or "I would never want to live like that?" If so, what are they and why do you feel this way about them? For each of the circumstances that you just identified, what do you think should be the goals for your care? For example, should the goal of care be to prolong your life, improve or maintain your function and/or quality of life, provide comfort care, or something else? For other situations in which you would not be able to communicate your preferences, such as [to be provided by the clinician], what do you think should be the goals for your care? For example, should the goal of care be to prolong your life, improve or maintain your function and/or quality of life, provide comfort care, or something else? Are there any life-sustaining treatments that you know you would want to receive regardless of the circumstances, or would not want to receive under any circumstances? If so, what are they and why do you feel this way about them? Some people have more concerns about the way they will die or dying than death itself. Do you have any fears or concerns about this? In the event that you are dying, where do you want to receive your health care? Should your current preferences be strictly applied to future situations or serve as a general guide to your spokesperson or family member(s)? An alternative approach after introducing the topic of advance care planning is to suggest that the patient review an existing advance care planning educational resource and utilize issues presented in its content to guide discussion and/or worksheets when available to append to an advance directive.

How can I raise these issues without scaring patients?

You can raise advance care planning as one of many health promotion activities. These discussions are aimed at avoiding harms (over- and under-treatment), and promoting benefits (treatments tailored to the patient's goals). You should reassure the patient that raising this issue does not mean that there is something unspoken to worry about. You also may tell the patient that this topic is difficult for many patients and that you will understand if she does not want to come to any conclusions during this discussion. You may want to refer the patient to one of several reputable internet based advance care planning sites to stimulate further thought and preliminary steps in advance care planning


संबंधित स्टडी सेट्स

Chapter 13: Labor and Birth Process

View Set

Supply Chain Management Midterm - T/F

View Set

CIS3361 Information security Management CH7

View Set

Focus on Personal Finance: Chapter 4

View Set

ACT English Section Practice: Semicolons, Dashes, and Commas

View Set